Você está na página 1de 241

Espacios Lp

Rene Erln Castillo

Departamento de Matematicas
Universidad Nacional de Colombia
Bogota - Colombia
ii

A la memoria de Julieta mi mama-abuela


A mi esposa Hilcia del Carmen
A mis hijos: Rene Jose
Manuel Alejandro
Irene Gabriela y
Renzo Rafael
Indice general

Introducci
on 1

1. Funciones Convexas y Desigualdades 5


1.1. Funciones Convexas . . . . . . . . . . . . . . . . . . . . . . . . 5
1.2. Desigualdad de Young . . . . . . . . . . . . . . . . . . . . . . 16

2. Los espacios Lp 21
2.1. Funciones en Lp y sus normas . . . . . . . . . . . . . . . . . . 21
2.2. Supremo esencial . . . . . . . . . . . . . . . . . . . . . . . . . 22
2.3. Lp no es un espacio normado . . . . . . . . . . . . . . . . . . 27
2.4. Los espacios lp . . . . . . . . . . . . . . . . . . . . . . . . . . . 38
2.4.1. Desigualdad de Holder y Minkowski version discreta . . 38
2.4.2. El espacio lp y su norma . . . . . . . . . . . . . . . . . 40
2.4.3. Desigualdad de Hardy en lp . . . . . . . . . . . . . . . 45
2.4.4. Desigualdad de Hilbert en lp . . . . . . . . . . . . . . . 47
2.4.5. El espacio l . . . . . . . . . . . . . . . . . . . . . . . 50
2.5. Aproximaciones en Lp () . . . . . . . . . . . . . . . . . . . . . 57
2.6. Funcionales Lineales y Acotados sobre el Espacio Lp () . . . . 67
2.7. Espacios duales . . . . . . . . . . . . . . . . . . . . . . . . . . 86
2.8. Convergencia debil en Lp . . . . . . . . . . . . . . . . . . . . . 89
2.9. Continuidad de la traslacion en Lp () para 1 p < . . . . . 103

3. Operadores Integrales 107


3.1. Desigualdades basicas y algunos operadores importantes . . . 107
3.2. Lp es un espacio reflexivo para 1 < p < . . . . . . . . . . . 115
3.3. El espacio L2 . . . . . . . . . . . . . . . . . . . . . . . . . . . 118
3.3.1. Teorema de Radon-Nikodym . . . . . . . . . . . . . . . 121
3.4. Espectro de un Operador . . . . . . . . . . . . . . . . . . . . . 127

4. Operador Maximal 137


4.1. Funciones localmente integrables . . . . . . . . . . . . . . . . . 137

iii
iv INDICE GENERAL

4.2. El teorema de cubrimiento de Vitali . . . . . . . . . . . . . . . 138


4.3. Funcion Maximal de Hardy-Littlewood . . . . . . . . . . . . . 138

5. Convoluci on 157
5.1. Resultados basicos . . . . . . . . . . . . . . . . . . . . . . . . 157
5.2. El soporte y la convolucion . . . . . . . . . . . . . . . . . . . . 168
5.3. Convolucion con funciones suaves . . . . . . . . . . . . . . . . 170
5.4. Aproximaciones de la identidad . . . . . . . . . . . . . . . . . 172

6. Potenciales 179
6.1. Potencial de Riesz . . . . . . . . . . . . . . . . . . . . . . . . . 179
6.2. Potenciales en Lp . . . . . . . . . . . . . . . . . . . . . . . . . 186

7. Espacios Lp () con 0 < p < 1 197


7.1. Nociones basicas . . . . . . . . . . . . . . . . . . . . . . . . . 197

8. Lp es un espacio uniformemente convexo 209


8.1. Convexidad Uniforme . . . . . . . . . . . . . . . . . . . . . . . 209

9. Isometra en Lp 217

A. El teorema del residuo 225

B. -homomorfismo 229

C. Alfabeto griego 231

Bibliografa 232
Indice alfab
etico 232
INDICE GENERAL 1

Introducci
on
El presente texto no pretende ser un tratado en espacios Lp (0 < p ),
lo cual sera una utopa, el objetivo del mismo es estudiar (en la medida de
lo posible) de manera exhaustiva los espacios Lp , dado que de alguna manera
podemos ver el espacio Lp como el padre de todos los espacios de funciones
integrables, por ejemplo, los espacios de Sobolev se definen va los espacios Lp .

A proposito de esto, en la literatura podemos encontrar tratados, manua-


les y textos dedicados al estudio de los espacios de Sobolev, en los cuales se
destina un captulo para estudiar aspectos generales de los espacios Lp . En
mi opinion, esto es estudiar el fruto sin importar la tierra de donde proviene.
Basado en esta opinion, decid preparar un texto dedicado solo al estudio de
los espacios Lp , el cual luce, sino u
nico, como uno de los pocos en su genero,
tanto en lengua castellana como inglesa. Esta afirmacion surge despues de
una exhaustiva b usqueda en todas las fuentes disponibles para el autor.

En matematicas los espacios Lp son espacios de funciones que se definen


usualmente usando una generalizacion natural de la norma p para espacios
vectoriales de dimension finita, estos suelen llamarse de Lebesgue en honor a
Henri Lebesgue (Dunford Schwartz 1958 III.3), sin embargo seg un Bourba-
ki (1987) los espacios Lp fueron introducidos por Riesz (1910). Los espacios
Lp forman una clase importante de espacios de Banach, los cuales se estu-
dian en analisis funcional y en espacios vectoriales topologicos. Ademas, estos
espacios tienen aplicaciones en fsica, probabilidad y estadstica, finanzas e
ingeniera, as como en otras disciplinas. Otra manera de ver la importancia
de los espacios Lp es mirarlos como una generalizacion parcial de los espacios
L2 , este ultimo tiene un origen independiente basado en hechos basicos de
analisis de Fourier.

Cronologicamente, los espacios L1 representan el espacio de todas las


funciones Lebesgue integrables, ademas L1 esta conectado va dualidad al
espacio de las funciones esencialmente acotadas L . Quiero destacar que la
multiplicacion de dos funciones Lebesgue integrables no necesariamente es
Lebesgue integrable; sin embargo, si tomamos dos funciones una p-integrable
y otra q-integrable, la desigualdad de Holder nos garantiza que el producto
de estas funciones resulta Lebesgue integrable.

En el primer semestre de 2012, el departamento de matematicas de la


Universidad Nacional de Colombia me dio la oportunidad de dictar el curso
topicos avanzados en analisis, oportunidad propicia e ideal para desarrollar
2 INDICE GENERAL

el presente texto. Es oportuno aclarar que el contenido de este texto no es


original del autor, este esta basado en diversos tratados, textos y artculos
diseminados en la literatura. Sin embargo, en el texto se incluyen demos-
traciones de resultados clasicos de manera novedosa, as como resultados no
populares tal como el espectro del operador de Hardy, amen de un n umero
de ejercicios (125) que aportaran una luz en el fascinante mundo de la inves-
tigacion en el campo de las desigualdades matematicas.

En la bibliografa se citan algunos de los textos en los cuales el autor se


baso para organizar el presente trabajo. Este texto esta dirigido a estudian-
tes que hayan cursado teora de la medida e integracion de Lebesgue, analisis
funcional y analisis complejo, as como a los profesionales que requieren de
estas herramientas para el desarrollo de sus trabajos de investigacion.

El lector pudiera extranar una seccion donde se incluyan resultados como


el principio de acotacion uniforme (Banach-Steinhauss), Teorema de Hanh-
Banach (version norma), Teorema de Egoroff, Teorema de Fubini, etc. A
juicio del autor, incluir estos resultados como una seccion o apendice sera
engrosar de manera innecesaria el presente texto, en tal sentido es suficiente
consultar los textos clasicos en el tema tales como Real Analysis por H.L.
Royden [?] y Real Analysis, Modern Tecniques and Their Applications por
Gerald Folland [?] para comprender todo el material expuesto en el presen-
te texto. Sin embargo, incluimos al final del texto un apendice dividido en
secciones A, B y C donde se incluyen algunos resultados no populares pero
relevantes, los cuales se utilizan en algunas demostraciones.

Este texto esta dividido en captulos y a su vez cada captulo esta sub-
dividido en secciones. La mayora de los captulos finaliza con una seccion
de ejercicios. El objetivo de esta seccion es que el lector, a traves de la re-
solucion de los mismos, pueda comprobar el grado de dominio alcanzado de
las ideas y tecnicas presentadas en cada captulo. Por supuesto, cada seccion
de ejercicios contiene problemas de diferentes grados de complejidad, existen
algunos de rutina y otros que pueden verse como un reto. Todas las crticas
constructivas que redunden en beneficio de este trabajo, seran bienvenidas.
Cualquier error de transcripcion o de calculo es absoluta responsabilidad del
autor.

Quiero expresar mi mas sincera gratitud a Oscar Pe na, Oscar Guzman,


Dionisio Dallos, Fabio Vallejo y Camilo Chaparro, quienes mostraron du-
rante el desarrollo del curso topicos avanzados en analisis su entusiasmo en
aprender las ideas expuestas por el autor. Para m fue placentero tenerlos
INDICE GENERAL 3

como estudiantes, en virtud de la disimilitud en la formacion matematica


de cada uno, aunado a la particularidad de sus intereses en la matematica.
Finalmente quiero reconocer y agradecer a Oscar Pe na quien inicio la trans-
cripcion de la primera version de este texto, Fabio Vallejo y Camilo Chaparro
quienes continuaron la labor de Pe na. A todos ellos un millon de gracias. A
Humberto Rafeiro, mi colega y colaborador quien hizo posible esta version
final que ahora usted amigo lector tiene en sus manos.

Erln Castillo
Rene
Bogota, Colombia

Algunas veces hay que decir cosas difciles,


pero se deben decir de la forma mas simple posible.
G.H. Hardy
4 INDICE GENERAL
Captulo 1

Funciones Convexas y
Desigualdades

1.1. Funciones Convexas


Definicion 1.1. Sea f una funcion definida en un intervalo (a, b) se dice
que es convexa en (a, b) si para todo x, y (a, b) satisface

f (tx + (1 t) y) tf (x) + (1 t) f (y)

para 0 < t < 1.


Teorema 1.2. f es convexa en (a, b) si y solo si
f (x) f (a) f (b) f (a)

xa ba
con a < x < b.
Demostracion. Sea a < x < b, entonces 0 < x a < b a de aqu
xa
0< <1
ba
xa
haciendo t = ba
, tendremos que

xa bx
1t=1 =
ba ba
por otra parte notese que
x (b a) xb xa xb ab + ba xa
x= = =
ba ba ba

5
6 Funciones Convexas y Desigualdades

xa bx
x= b+ a,
ba ba
ahora bien de la convexidad de f resulta

xa bx
f (x) f (b) + f (a)
ba ba
(b a) f (x) (x a) f (b) + (b x) f (a)
(b a) f (x) (x a) f (b) + af (a) af (a) + (b x) f (a)
(x a) f (b) (x a) f (a) + (b a) f (a)
f (x) f (a) f (b) f (a)
.
xa ba
Recprocamente, sea a < x < y < b y 0 < t < 1, luego 0 < 1 t, entonces
es claro que tx < ty y (1 t) x < (1 t) y as

x = tx + (1 t) x tx + (1 t) y < ty + (1 t) y = y

es decir,
x < tx + (1 t) y < y,
entonces por hipotesis

f (tx + (1 t) y) f (x) f (y) f (x)



tx + (1 t) y x yx

(y x) [f (tx + (1 t) y) f (x)] (tx + (1 t) y x) (f (y) f (x))


(y x) (1 t) (f (y) f (x))
f (tx + (1 t) y) f (x) f (x) + tf (x) + (1 t) f (y)
f (tx + (1 t) y) tf (x) + (1 t) f (y) .

Corolario 1.3. Sea f convexa en (a, b) y a < x < y < z < b, entonces

f (y) f (x) f (z) f (x) f (z) f (y)


.
yx zx zy

Demostracion. Sean x, y, z tales que x < y < z, entonces por el Teorema 1.2

f (y) f (x) f (z) f (x)



yx zx
1.1 Funciones Convexas 7

ahora bien, por otra parte x < y < z implica que z < y < x de
aqu obtenemos 0 < z y < z x, luego
zy
0< <1
zx
hacemos
zy
t=
zx
as
yx
1t= .
zx
Notese que
zx yz yx yx zy
y=y = = z+ x
zx zx zx zx
luego  
yx zy
f (y) = f z+ x
zx zx
de la convexidad de f tenemos que
yx zy
f (y) f (z) + f (x)
zx zx
(z x) f (y) (z y) f (x) + (y x) f (z)
(z x) f (y) (z y) f (x) + zf (z) zf (z) + (y x) f (z)
(z x) f (y) (z y) f (x) (z y) f (z) + (z x) f (z)
(z y) (f (z) f (x)) (z x) (f (z) f (y))
f (z) f (x) f (z) f (y)

zx zy
por lo tanto
f (y) f (x) f (z) f (x) f (z) f (y)
.
yx zx zy

Teorema 1.4. Sea f : (a, b) R continua tal que


 
x+y 1 1
f f (x) + f (y) ,
2 2 2

entonces f es convexa en (a, b).


8 Funciones Convexas y Desigualdades

Demostracion. Procedamos por induccion, sea n N, para n = 1, tendremos


que
       
1 1 x+y 1 1
f x+ 1 y =f f (x) + 1 f (y) .
2 2 2 2 2
Supongamos que para 0 < k < 2n se cumple
     
k k k k
f x + 1 n y n f (x) + 1 n f (y)
2n 2 2 2
ahora consideremos  
k k
x0 = n x + 1 n y
2 2
 
k+1 k+1
y0 = x+ 1 n y
2n 2
 
2k + 1 2k + 1
x0 + y 0 = x+ 2 y
2n 2n
 
x0 + y 0 2k + 1 2k + 1
= n+1 x + 1 n+1 y
2 2 2
y escojamos h = 2k + 1 2n+1 , entonces
     
h h x0 + y 0 1 1
f n+1
x + 1 n+1 y = f f (x0 ) + f (y0 )
2 2 2 2 2
 
h h
n+1 f (x) + 1 n+1 f (y) .
2 2
Sea [0, 1], n N entonces existe kn Z + tal que
kn kn + 1
n

2 2n
de aqu obtenemos que
kn
lm =
n 2n

de la continuidad de f tenemos
   
kn kn
f (x + (1 ) y) = f lm n x + 1 lm n y
n 2 n 2

   
kn kn
f (x + (1 ) y) = lm f x+ 1 n y
n 2n 2
1.1 Funciones Convexas 9

 
kn kn
lm n f (x) + lm 1 n f (y)
n 2 n 2
f (x + (1 ) y) f (x) + (1 ) f (y) .

Teorema 1.5. Sea f una funcion convexa definida de (a, b) en R, entonces


f es continua sobre (a, b).
Demostracion. Sea x0 un punto cualquiera de (a, b) y un n umero real po-
sitivo tal que la bola cerrada de centro x0 y radio esta contenida en (a, b),
es decir
B (x0 ) = {x R : |x x0 | < } (a, b) .
Sea M = max {f (x0 ) , f (x0 + )}, ahora bien cualquier x B (x0 ) =
(y1 , y2 ) donde y1 = x0 y y2 = x0 + se puede escribir en la manera
siguiente
y2 x x y1
x= y1 + y2
y2 y1 y2 y1
de donde
y2 x x y1
f (x) f (y1 ) + f (y2 )
y2 y1 y2 y1
lo que implica que
f (x) M
esto significa que f es acotada sobre B (x0 ).
Para x 6= x0 , definamos

u = (sign (x x0 ))1 ,

entonces x tiene dos posibilidades :


(a) x esta en (x0 , x0 + u), o

(b) x esta en (x0 u, x0 ).


Consideremos la posibilidad (a); es decir x0 < x < x0 + u. De aqu se obtiene
que
x x0
0< <1
u
escribiendo
x x0 |x x0 |
t= =
u
se tiene que
x x0 = tu (1.1)
10 Funciones Convexas y Desigualdades

operando adecuadamente en (1.1) se obtiene

x = t (x0 + u) + (1 t) x0

x t
x0 = + (x0 u)
1+t 1+t
puesto que f es convexa

f (x) tf (x0 + u) + (1 t) f (x0 ) (1.2)

1 t
f (x0 ) f (x) + f (x0 u) (1.3)
1+t 1+t
teniendo en cuenta que f es acotada sobre B (x0 ) y operando adecuadamente
en (1.2) y (1.3) se tiene que

t [M f (x0 )] f (x) f (x0 ) t [M f (x0 )]

|f (x) f (x0 )| t [M f (x0 )]


|xx0 |
puesto que t =
resulta

[M f (x0 )]
|f (x) f (x0 )| |x x0 | . (1.4)

Al considerar la posibilidad (b) tambien se obtiene (1.4), concluyendose que
f es continua en (a, b).

Teorema 1.6 (Desigualdad de Jensen). Sea una medida positiva sobre


una -algebra A en un conjunto , tal que () = 1. Sea f una funcion
integrable sobre , es decir,
Z
f d <

si a < f (x) < b para todo x y es una funcion convexa en (a, b),
entonces Z  Z
f d f d.

La igualdad se da si f (x) = c para todo x donde c es un n


umero real.

Nota: Los casos a = y b = no estan excluidos.


1.1 Funciones Convexas 11

Demostracion. Si f (x) = c para todo x con c R, entonces


Z  Z Z
f d = (c) = (c) d = (f ) d.

Sea B la -algebra de Borel y A B, entonces 1 (A) B ya que es


continua en (a, b). Dado que f es A-medible se tiene que

( f )1 (A) = f 1 (1 (A)) A,

R hemos demostrado que f es A-medible.


de esta manera
Sea t0 = f d, entonces en virtud de que a < f (x) < b para todo x
y () = 1, tendremos que
a < t0 < b
por el corolario 1.3 tenemos

(t0 ) (s) (u) (t0 )


(1.5)
t0 s u t0
para todo a < s < t0 < u < b.
Sea  
(t0 ) (s)
= sup (1.6)
a<s<t0 t0 s
afirmamos que
(y) (t0 ) + (y t0 )
para todo y (a, b). En efecto

(I) Si y = t0 , no hay nada que probar.

(II) Si a < y < t0 , por (1.6) resulta

(t0 ) (y)
,
t0 y

luego (y) (t0 ) + (y t0 ).

(III) Si to < y < b, entonces por (1.5) y (1.6) se tiene que

(y) (t0 )

y t0

de donde (y) (t0 ) + (y t0 ).


12 Funciones Convexas y Desigualdades

De (I), (II) y (III) se concluye que

(y) (t0 ) + (y t0 )

para todo y de (a, b). Ahora tenemos y = f (x) entonces

(f (x)) (t0 ) + (f (x) t0 )

integrando sobre resulta


Z Z 
(f (x)) d (t0 ) + f (x) d t0 .

De aqu se deduce que


Z  Z
f d f d.

Observaci on 1.7. Note que en el teorema 1.6 no se exigio la integrabilidad


de la funcion f dado que si la integral de f es infinita la desigualdad
de Jensen se satisface trivialmente.
El siguiente ejemplo muestra que la condicion de que f sea integrable en
un conjunto con () = 1 es suficiente para que la desigualdad de Jensen
sea valida.
Ejemplo 1.8. Sea f (x) = x3/4 definida en (0, 1) y sea (x) = x2 definida
en R, notese que f es integrable en = [0, 1] y
Z 1  Z 1 2
3/4
f (x) dx = x dx = 16.
0 0

Por otra parte note que


Z 1 Z 1 2
( f ) dx = x3/4 dx =
0 0

Observaci on 1.9. El siguiente ejemplo nos dice que la condicion () = 1


en el teorema 1.6 es necesaria.
Ejemplo 1.10. Sea = [1, 16] y f (x) = x3/4 definida en [1, 16], asi
Rcomo (x) = x2 definida en . Notese que es convexa en , ademas
16 3/4
1
x dx = 4, luego
Z 16  Z 16 2
3/4 3/4
x dx = x dx = 16.
1 1
1.1 Funciones Convexas 13

Por otra parte Z 16 Z 16 2 3


f dx = x3/4 dx = .
1 1 2
As resulta Z 16  Z 16
3/4
x3/4 dx.

x dx >
1 1

Corolario 1.11. Bajo las hipotesis del Teorema 1.6 se cumple que
R  R
f gd (f ) gd
R R

gd
gd

donde g es una funcion positiva e integrable sobre .

Demostracion. En la demostracion del Teorema 1.6 se demostro que si es


convexa, existe tal que

(y) (t0 ) + (y t0 ) (1.7)

para todo y de (a, b). Ahora bien escribiendo y = f (x) y multiplicando (1.7)
por una funcion g positiva e integrando obtenemos
Z Z Z Z Z
(f ) gd (t0 ) gd + (f ) gd + f gd t0 gd. (1.8)

Definamos R
f gd
t0 = R

gd
sustituyendo en (1.8) resulta
R Z R Z
f gd f gd
Z Z

(f ) gd R gd + f gd R gd

gd
gd

de donde R  R
f gd (f ) gd
R R .

gd
gd
Observaci
on 1.12. El corolario 1.11 generaliza la desigualdad de Jensen.

Ejercicios

1. Sea : [0, +) [0, +) una funcion tal que

i. es convexa.
14 Funciones Convexas y Desigualdades

ii. (x) = 0 x = 0.

Demostrar que:

a) es estrictamente creciente en [0, +).


(x) (y)
b) Si 0 < x < y, entonces x
y
.
c) Si 0 x 1, entonces (x) x(1).

2. Una funcion f se dice superaditiva si f (x) + f (y) f (x + y). Si f es


una funcion convexa tal que f (0) = 0 entonces f es superaditiva.

3. Demostrar que el supremo de cualquier coleccion de funciones convexas


en (a, b) es convexo y que los lmites puntuales de sucesiones de fun-
ciones convexas tambien lo son. Que se puede afirmar de los lmites
superior e inferior de sucesiones de funciones convexas?

4. Supongamos que es convexa en (a, b) y que es convexa no decre-


ciente en el recorrido de . Demostrar que es convexa en (a, b).
Para > 0, demostrar que la convexidad de log implica la de , pero
no a la inversa.

5. Demostrar que la composicion de funciones convexas puede no ser con-


vexa.

6. Sea f : (a, b) R derivable por la derecha en todo punto. Si dicha


derivada es creciente, demostrar que f es convexa.

7. Si f es convexa en (a, b). Demostrar que

n
! n
X X
f j xj j f (xj ),
j=1 j=1

para todo x1 , x2 , . . . , xn (a, b) y escalares 1 , 2 , . . . , n tales que j


0, j = 0, 1, 2, . . . n y 1 + 2 + + n = 1.
R
8. Supongase que () = 1 y h : [0, +) medible. Si A =
h d.
Demuestrese que
Z
2
1+A 1 + h2 d 1 + A.

1.1 Funciones Convexas 15

9. Supongase que es una funcion real tal que


Z 1  Z 1
f (x) dx (f (x)) dx
0 0

para toda f acotada y medible. Demuestre que es entonces convexa.


10. Suponiendo que es estrictamente convexa y () = 1. Demostrar que
la desigualdad de Jensen en
Z  Z
f d f d

es una igualdad si y solo si f es constante c.t.p.


11. Supongamos que () = 1 y que f, g son medibles no negativas en
con f g 1. Demostrar que
Z Z
f d g d 1.

12. Sea unaR medida positiva en X y supongamos que f : X (0, +)


satisface X f d = 1. Demostrar que las desigualdades
a) Z
1
log(f ) d (E) log
E (E)
b) Z
f p d ((E))1p (0 < p < 1)
E

son validas para todo conjunto medible E con 0 < (E) < .
13. Sea f una funcion medible positiva en [0, 1]. Cual de las dos cantidades
Z 1 Z 1 Z 1
f (x) log f (x) dx; f (s) ds log f (t) dt
0 0 0

es mayor?
umeros no negativos tal que
P
14. Sea {n }nN una sucesion de n n=1 n = 1
y {n }nN una sucesion de numeros positivos. Demuestre que

Y
X
nn n n .
n=1 n=1
16 Funciones Convexas y Desigualdades

15. Use el ejercicio anterior para demostrar que la media geometrica siem-
pre es menor o igual que la media aritmetica, es decir
p
n 1 + 2 + + n
1 2 n .
n

16. Demuestre que la media armonica es siempre menor o igual que la


media geometrica, es decir
n
1 1 1 n
x1 x2 xn .
x1
+ x2
+ + xn

17. Sea (, A, ) un espacio de medida tal que () = 1 y f L1 () con


a < f (x) < b para todo x . Demuestre que si es concava en (a, b)
entonces Z  Z
f d f d.

18. Sea g una funcion no negativa y medible en [0, 1]. Demuestre que
Z 1 Z 1
log g(t) dt log g(t) dt.
0 0

1.2. Desigualdad de Young


En esta seccion damos una demostracion analtica de la desigualdad de
Young. Usualmente esta, es motivada va geometrica. Sin menos cabo del
uso de la intuicion geometrica, consideramos que esta demostracion por su
sencillez, resulta muy sugestiva y formativa al mismo tiempo, debo aclarar
que la demostracion referida no es original del autor.

Teorema 1.13 (Desigualdad de Young). Sea y = f (x) una funcion creciente


en [0, ) con f (0) = 0. Supongamos que f es de clase C 1 en [a, b] con a y b
n
umeros reales no negativos, entonces
Z a Z b
ab f (x) dx + f 1 (y) dy,
0 0

donde f 1 (y) es la funcion inversa de f . La igualdad se cumple si b = f (a).


1.2 Desigualdad de Young 17

Demostracion. Si f es de clase C 1 en [a, b], integrando por partes obtenemos:


Z b Z b
f (x) dx = bf (b) af (a) xf 0 (x) dx. (1.9)
a a

Sea y = f (x), entonces dy = f 0 (x)dx, ademas x = f 1 (y). Reemplazando en


(1.9) resulta
Z b Z f (b)
f (x) dx = bf (b) af (a) f 1 (y) dy. (1.10)
a f (a)

Ahora, si r x a, entonces f (r) f (x), as


Z a
(a r)f (r) f (x) dx
r
Z a
af (r) rf (r) f (x) dx. (1.11)
r

Notese que
Z a Z 0 Z a
f (x) dx = f (x) dx + f (x) dx
r r 0
Z a Z r Z a
f (x) dx = f (x) dx + f (x) dx.
r 0 0

En virtud de (1.11), tenemos


Z r Z a
af (r) rf (r) f (x) dx + f (x) dx. (1.12)
0 0

Por (1.10),
Z r Z f (r)
f (x) dx = rf (r) f 1 (y) dy, (1.13)
0 0
reemplazando (1.13) en (1.12) resulta
Z f (r) Z a
1
af (r) rf (r) rf (r) + f (y) dy + f (x) dx
0 0
Z a Z f (r)
af (r) f (x) dx + f 1 (y) dy.
0 0

Si 0 < b < f (a), podemos escoger r = f 1 (b). As,


Z a Z f (f 1 (b))
1
af (f (b)) f (x) dx + f 1 (y) dy
0 0
Z a Z b
ab f (x) dx + f 1 (y) dy.
0 0
18 Funciones Convexas y Desigualdades

Finalmente, en virtud de (1.10) para b = f (a) se tiene que


Z a Z f (a)
f (x) dx = af (a) f 1 (y) dy
0 0

luego,
Z a Z b
f (x) dx = ab f 1 (y) dy
0 0
Z a Z b
ab = f (x) dx + f 1 (y) dy.
0 0

Corolario 1.14 (Young). Sea 1 < p < q < tal que p1 + 1q = 1, entonces
p q
ab ap + bq para a y b reales positivos. La igualdad se cumple si ap = bq .

Demostracion. En primer lugar si ap = bq , entonces


a = bq/p
luego,
a = bq1 ,
por lo tanto,
ab = bq .
As,  
q 1 1
ab = b + ,
p q
es decir,
ap b q
ab = + .
p q
Ahora, consideremos f (x) = x con > 0 y f 1 (y) = y 1/ , notese que f
satisface las hipotesis del Teorema 1.13, entonces
Z a Z b

ab x dx + y 1/ dy
0 0
+1 1/+1
a b
= + .
+ 1 1/ + 1
+1 1 1
Si p = + 1 y q =
. Note que p
+ q
= 1. As,
ap b q
ab + .
p q
1.2 Desigualdad de Young 19

Observaci on 1.15. Es importante se nalar que esta u


ltima desigualdad es
susceptible de ser demostrada de varias maneras conceptualmente diferentes.
Corolario 1.16 (Young). Para a > 0, b > 0, se tiene que
ab a log+ a + eb1 ,
donde 
+ log x si x > 1;
log x =
0 si 0 x 1.

Demostracion. Sean y funciones definidas por:



log x + 1 si x > 1;
(x) =
1 si 0 x 1.
y 
ey1 si y > 1;
(y) =
0 si 0 y 1.
Ahora, definamos
(x) = (x + 1) 1 y (y) = (y + 1) 1.
Notese que es de clase C 1 en [0, ) y (0) = 0, ademas, note que es
la funcion inversa de , entonces por el Teorema 1.13 para a > 1 y b > 1
tenemos
Z a1 Z b1

(a 1)(b 1) (x) dx + (y) dy
0 0
Z a1 Z b1
= (x + 1) dx (a 1) + (y + 1) dy (b 1)
0 0
Z a Z b
= (u) du + (t) dt (a + b) + 2.
1 1

Entonces,
Z a Z b
ab (a + b) + 1 (u) du + (t) dt (a + b) + 2
1 1
Z a Z b
ab (u) du +
(t) dt + 1
1 1
Z a Z b
ab (log u + 1) du + et1 dt + 1
1 1
+ 1 b
= a log a + e [e e] + 1
= a log+ a + eb1 1 + 1,
20 Funciones Convexas y Desigualdades

finalmente,
ab a log+ a + eb1 .

Ejercicios

umeros reales con 0 < p < 1 y < q < 0 tal que


1. Sean p y q dos n
1 1
p
+ q = 1. Demostrar que

ap b q
ab +
p q
para a y b reales positivos.

2. Use las funciones


1

p log a si 0 x

u(x) = p
1
q log b si x 1

p

(a, b > 0) y f (x) = ex para demostrar que

ap b q 1 1
ab + con + = 1.
p q p q

3. Dados a, b > 0 y > 0. Demostrar que

ab ap + c()bq

(p)q/p
donde c() = .
q
Captulo 2

Los espacios Lp

Las clases Lp de Lebesgue ocupan la posicion central de mu-


chos trabajos modernos, en teora de funciones reales o complejas,
en la teora de las series de Fourier, o en la teora general de desa-
rrollos ortogonales. Este trabajo exige un considerable dominio de
la tecnica de desigualdades; en todo momento se requieren las de-
sigualdades de Holder y de Minkowski, y otras desigualdades mas
modernas y sofisticadas del mismo caracter general.

G.H. Hardy, J.E. Littlewood & G. Polia.

2.1. Funciones en Lp y sus normas


Recordemos que un espacio de medida es una tripleta (X, A, ) la cual
consiste de un conjunto X no vaco, de una -algebra A de subconjuntos de
X y de una medida en A.
Definici on 2.1. Sea (X, A, ) un espacio de medida y p un numero real
positivo. Una funcion f A-medible definida de X en R (f : X R) se dice
que pertenece al espacio Lp () (pre-Lebesgue) si
Z
|f |p d < .
X

Es decir,
 Z 
Lp () = f : f : X R medible y p
|f | d < .
X

Ejemplo 2.2. Sea X = [0, 16] y f : X R definida por f (x) = x1/4 , note
que f L1 (m), pero f
/ L4 (m) donde m denota la medida de Lebesgue.

21
22 Los espacios Lp

Observacion 2.3. El ejemplo 2.2 nos dice que en general los espacios Lp
no son comparables.

Ejemplo 2.4. Sea X = [0, 1/2] y f : X R definida por


  1
1
2
f (x) = x log ,
x

entonces f L1 (m)

Ejemplo 2.5. Sea X = (0, ) y f : X R definida por f (x) = (1 + x)1/2 ,


entonces f Lp () para 2 < p < .

Observaci on 2.6. No es difcil verificar que Lp con 1 p < es un


espacio vectorial, en efecto, observe que si f, g Lp (), entonces en virtud
que

|f + g|p (|f | + |g|)p


(2 max(|f |, |g|))p
= 2p max(|f |p , |g|p )
2p (|f |p + |g|p ),

se tiene que f + g Lp ().

Ademas, si f Lp () y R, entonces f Lp ().

Por otra parte, las desigualdades

0 f + |f |

0 f |f |
implican que f + , f y |f | estan en Lp ().

2.2. Supremo esencial


Definicion 2.7. Sea (X, A, ) un espacio de medida y f una funcion A-
medible. Para cada M > 0 definamos EM = {x X : |f (x)| > M }. Notese
que EM A en virtud de que f es A-medible. Sea

A = {M > 0 : (EM ) = 0}
= {M > 0 : |f (x)| M c.t.p}.
2.2 Supremo esencial 23

El supremo esencial de f denotado por ess sup f o kf k es definido por



si A = ;
kf k = ess sup f =
nf A si A 6= .

Observaci on 2.8. Note que si A 6= , entonces 0 es una cota inferior de A,


luego nf A R. Sea = kf k < , afirmamos que A, en efecto, note
que

[
E = {x X : |f (x)| > } = {x X : |f (x)| > + 1/n},
n=1

ademas, para cada n {x X : |f (x)| > + 1/n} A. Como = nf A,


entonces para cada n N existe n A tal que n < + 1/n, de aqu se
tiene que

{x X : |f (x)| > + 1/n} {x X : |f (x)| > n }

luego,

({x X : |f (x)| > + 1/n}) ({x X : |f (x)| > n }) = 0,

as, (E ) = ({x X : |f (x)| > }) = 0, lo que demuestra que A,


luego si = kf k < , entonces

|f (x)| kf k c.t.p.

Ahora bien, definamos



f (x) si x
/ E ;
f (x) =
0 si x E ,

como f (x) = f (x) c.t.p, entonces

kf k = kf k = sup |f (x)| = sup |f (x)|.


xX xX\E

Definici
on 2.9.

L () = {f : f : X R medible y kf k < } .

Los miembros de L () se llaman funciones esencialmente acotadas.

Ejemplo 2.10. Toda funcion acotada en X esta en L ().


24 Los espacios Lp

Ejemplo 2.11. Sea



1 si x I [0, 1];
f (x) =
si x Q [0, 1].
Note que,
kf k = nf{M > 0 : ({x X : |f (x)| > M }) = 0} = 1,
por lo tanto f L ().
Ejemplo 2.12. Sean X = R, A = L y = m. Sea {r1 , r2 , . . . , rn , . . .} una
enumeracion de n
umeros racionales en R, definamos

n si x = rn Q;
f (x) =
1 si x R \ Q.
Queremos demostrar que
A = {M > 0 : m({x X : |f (x)| > M }) = 0} = [1, ).
En efecto, sea M [1, ), entonces
{x X : |f (x)| > M } Q,
luego,
m({x X : |f (x)| > M }) m(Q) = 0,
as, M A, es decir
[1, ) A. (2.1)
Por otra parte, supongamos que y
/ [1, ), entonces y < 1, luego
R \ Q {x X : |f (x)| > y},
por lo cual,
m({x X : |f (x)| > y}) 6= 0
lo que significa que y
/ A, entonces
A [1, ), (2.2)
de (2.1) y (2.2) tenemos
A = {M > 0 : m({x X : |f (x)| > M }) = 0} = [1, ).
Ahora, observe que
nf{M > 0 : m({x X : |f (x)| > M }) = 0} = 1,
por lo tanto f L ().
2.2 Supremo esencial 25

Ejemplo 2.13. Sean X = N, A = P(N ) y la medida de contar, sea


f (x) = x (x N). Afirmamos que

A = {M > 0 : m({x X : |f (x)| > M }) = 0} = .

En efecto, sea M > 0 arbitrario, escojamos k > M , k N, luego

({x X : |f (x)| > M }) ({k}) = 1,

lo que implica que M / A y como M es arbitrario, podemos concluir que


A = , por lo tanto kf k = .
Ejemplo 2.14. Sea (X, A, ) un espacio de medida tal que (X) < . En-
tonces L () Lp () para cualquier 1 p < .

En efecto, sea f L (), entonces |f | kf k c.t.p, luego


Z Z
p p
|f | kf k d = (X)kf kp < .
X X

As f Lp ().
Ejemplo 2.15. Sean 1 p < q < , entonces Lq () Lp () con (X) <
.
En efecto, sea f Lq (), si A = {x X : |f (x)| 1}, entonces

X = A + X\A

y
|f (x)|p < |f (x)|q para x X \ A y |f (x)| 1,
para x A, luego
Z
kf kpp = |f |p d
ZX Z
p
= A |f | d + X\A |f |p d
ZX Z X

A d + X\A |f |p d
X Z X

(A) + X\A |f |q d
X
(X) + kf kqq < ,

as, f Lp ().
26 Los espacios Lp

Observaci on 2.16. Los ejemplos 2.14 y 2.15 nos dicen bajo que condiciones
se pueden comparar los espacios Lp .
Observacion 2.17. a) Las inclusiones en los ejemplos precedentes 2.14 y
2.15 son estrictas. Para ver esto, consideremos el siguiente ejemplo.

Sea X = [0, 1] y 1 p < < q , donde = p+q


2
, luego si p < < q
tendremos que p/ < 1 y q/ > 1. Escojamos = 1/ y definamos
 1
x
si x 6= 0;
f (x) =
0 si x = 0.
Luego consideremos
Z 1 Z 1
p dx
|f (x)| dx =
0 xp
Z0 1
dx
= < ,
0 xp/
puesto que p/ < 1, entonces f Lp (m), por otra parte,
Z 1 Z 1
q dx
|f (x)| dx =
0 xq
Z0 1
dx
= q/
,
0 x

/ Lq (m). Por lo tanto,


ya que q/ > 1, as f

Lq (m) Lp (m).

b) Los ejemplos precedentes 2.14 y 2.15 no son ciertos si (X) = .

En efecto, consideremos la funcion constante f (x) = c con c 6= 0 en


(0, ), es facil ver que f L (), pero f
/ Lp () para 0 < p < .
Por otra parte, sea X = [1, ) y definamos f : X R por f (x) = x1 ,
entonces Z
1
= 1,
1 x2
as, f L2 (m), pero
Z
1
= ln x|
1 ,
1 x
/ L1 (m).
as, f
2.3 Lp no es un espacio normado 27

2.3. Lp no es un espacio normado


Sea (X, A, ) un espacio de medida, definamos

k kp : Lp () R+

por
Z 1/p
p
kf kp = |f | d
X
con 1 p < .
Ejemplo 2.18. Sea X = [0, 1], consideremos la funcion

1 si x Q [0, 1];
f (x) =
0 si x I [0, 1].

Entonces, para p = 1 se tiene que


Z
kf k1 = f (x) dm
[0,1]
Z Z
= 1 dm + 0 dm
Q[0,1] I[0,1]

= m(Q [0, 1])


= 0.

Por otra parte, si p = , entonces

kf k = nf{M > 0 : m({x [0, 1] : |f (x)| > M }) = 0} = 0,

sin embargo, note que f no es identicamente la funcion nula. Esto nos dice
que tanto k k1 como k k no definen una norma en L1 y L respectiva-
mente.
Ahora bien, para corregir esta debilidad del espacio Lp , tomemos dos
funciones f y g en Lp de modo que f este relacionada con g si y solo si f = g
c.t.p, en smbolos f g f = g c.t.p. Es solo un asunto de rutina verificar
que define una relacion de equivalencia.

Una vez verificado esto, denotemos la clase generada por f como

[f ] = {g Lp () : g f } y k[f ]kp = kgkp

para g [f ]. Ahora bien, sean g1 y g2 en [f ], entonces


28 Los espacios Lp

g1 f si y solo si g1 = f c.t.p
y
g2 f si y solo si g2 = f c.t.p,
por lo tanto g1 = g2 c.t.p, entonces

kg1 kp = kg2 kp .

Esto nos dice que k[f ]kp = kgkp esta bien definida por ser independiente del
representante de la clase [f ].

Ahora, definamos Lp () = Lp (X, A, ) = Lp  , en lo sucesivo, si no hay


confusion usaremos f en lugar de [f ]. Con algo de paciencia, podemos ver que
Lp es un espacio vectorial sobre R. Por otra parte, note que k k : Lp R+
ahora satisface kf kp = 0 si y solo si f = 0 c.t.p.

El siguiente resultado nos provee de la desigualdad triangular para la funcion


k kp .
Teorema 2.19 (Desigualdad de Minkowski). Sean 1 p y f, g
Lp (), entonces f + g Lp () y

kf + gkp kf kp + kgkp .

La igualdad se cumple si A|f | = B|g| c.t.p para A y B del mismo signo y no


simultaneamente nulos.

Demostracion. Verifiquemos la igualdad. Sean A y B n umeros del mismo


signo y no simultaneamente nulos tales que A|f | = B|g| c.t.p, entonces
Akf kp = Bkgkp , luego kf kp = B
A
kgkp . Por otra parte,

B
kf + gkp = A g + g

p
B+A
= kgkp
A
B
= kgkp + kgkp
A
= kf kp + kgkp .

Los casos p = y p = 1 son inmediatos, al igual que kf kp = kgkp = 0. Su-


pongamos que 1 < p < y kf kp = 6= 0 y kgkp = 6= 0, entonces existen
2.3 Lp no es un espacio normado 29

funciones f0 y g0 tales que |f | = f0 y |g| = g0 con kf0 kp = kg0 kp = 1.


Ahora, consideremos = +
y 1 = +
, note que 0 < < 1, lue-
go
|f (x) + g(x)|p (|f (x)| + |g(x)|)p
= (f0 (x) + g0 (x))p
= [( + )f0 (x) + ( + )(1 )g0 (x)]p
= ( + )p (f0 (x) + (1 )g0 (x))p
( + )p [(f0 (x))p + (1 )(g0 (x))p ]. (2.3)
Dado que (t) = tp es convexa en [0, ), integrando en (2.3) tenemos
Z
|f (x) + g(x)|p d ( + )p [kf0 kpp + (1 )kg0 kpp ]
X
= ( + )p < ,
es decir, f + g Lp (). Finalmente,
kf + gkpp (kf kp + kgkp )p
as,
kf + gkp kf kp + kgkp .

Observaci
on 2.20. Consideremos la funcion
|x|1/2 si |x| < 1;

f (x) =
0 si |x| 1.
Observe que Z Z
dx
f (x) dx = p = 4,
R [1,1] |x|
as, f L1 (m), pero
Z Z
2 dx
f (x) dx = ,
R [1,1] |x|
as, f 2
/ L1 (m).
Ahora, queremos estudiar bajo que condiciones el producto de dos fun-
ciones de L1 () se queda en L1 (). El siguiente resultado nos dice que si
f Lp () y g Lq () para p y q n umeros conjugados es decir, p1 + 1q = 1, se
tendra que f g L1 (). Previo a la demostracion de este poderoso resultado,
necesitaremos el siguiente Lema.
30 Los espacios Lp

Lema 2.21. Sea 1 p < . Entonces, para n umeros a, b y t no negativos


se tiene que
(a + tb)p ap + ptbap1 .

Demostracion. Definamos

(t) = (a + tb)p ap ptbap1 .

Note que (0) = 0 y 0 (t) = pb[(a + tb)p1 ap1 ] 0 para p 1 y a, b, t


positivos, luego es creciente en [0, ) y as no negativa para t 0. As,

(t) (0)
(a + tb)p ap + ptbap1 .

Teorema 2.22 (Desigualdad de Holder). Si p y q son n umeros extendidos no


1 1
negativos tales que p + q = 1 y si f Lp (), g Lq (), entonces f g L1 ()
y Z
|f g| d kf kp kgkq .
X

La igualdad se cumple si existen constantes A y B no simultaneamente nulos


tales que A|f |p = B|g|q c.t.p.

Demostracion. En primer lugar consideremos p = 1 y q = , entonces es


claro que
|g| kgk c.t.p.
Como |f | 0, se tiene que |f g| |f |kgk c.t.p. Por lo cual,
Z Z 
|f g| d |f | d kgk ,
X X

as, Z
|f g| d kf k1 kgk .
X

Ahora, supongamos que 1 < p < y 1 < q < y f 0, g 0. Definamos


h(x) = [g(x)]q/p , entonces

g(x) = [h(x)]p/q = [h(x)]p1 .


2.3 Lp no es un espacio normado 31

Luego, en virtud del Lema 2.21 se tiene


ptf (x)g(x) = ptf (x)[h(x)]p1
(h(x) + tf (x))p [h(x)]p .
As,
Z Z Z
p
pt f (x)g(x) d (h(x) + tf (x)) d [h(x)]p d
X X X
= kh + tf kpp khkpp .
En vista de la desigualdad de Minkowski se tiene
Z
pt f (x)g(x) d (khkp + tkf kp )p khkpp
ZX
(khkp + tkf kp )p khkpp
p f (x)g(x) d .
X t
Sea F (t) = (khkp + tkf kp )p , luego F (0) = khkpp . Entonces,
F (t) F (0)
Z
p f g d lm = F 0 (0)
X t0 t
= p(khkp )p1 kf kp .
Note que
Z  p1
p
Z 1 p1
[h(x)]p d = [g(x)]q d
X X
Z  1q
q
= [g(x)] d ,
X

as, khkp1
p = kgkq . Por lo tanto,
Z
f g d kf kp kgkq .
X

Finalmente, escogiendo A = kgkqq y B = kf kpp tal que A|f |p = B|g|q , entonces

|g|q/p
|f | = kf kp q/p
,
kgkq
integrando, resulta Z
|f g| d = kf kp kgkq .
X
32 Los espacios Lp

Observaci on 2.23. Tradicionalmente, la desigualdad de Minkowski, se ob-


tiene de la desigualdad de Holder, aca obtenemos la desigualdad de Holder
a partir de la desigualdad de Minkowski, esto nos hace pensar que ambas
desigualdades dependen una de la otra. Sin embargo, si analizamos con dete-
nimiento la situacion planteada, podemos observar que la desigualdad Young
(Corolario 1.14) nos provee de una herramienta la cual nos permite demos-
trar la desigualdad de Holder sin utilizar la desigualdad de Minkowski. Tam-
bien, a traves de la desigualdad generalizada de Jensen (Corolario 1.11) po-
demos obtener la desigualdad de Holder (ver problema 34 seccion 2.3).
Los dos resultados a continuacion nos dan la generalizacion de la desigual-
dad de Holder.
1
Corolario 2.24. Sean p, q y r n umeros reales tales que p
+ 1q + 1r = 1. Sean
f Lp (), g Lq () y h Lr (). Entonces,
Z
|f gh| d kf kp kgkq khkr .
X

Demostracion. Sea p1 + 1q = 1s , entonces ps + qs = 1, luego 1s + 1r = 1. Queremos


demostrar que f g Ls (). En efecto, por el Teorema 2.22
Z Z s/p Z s/q
s sp/s sq/s
|f g| d |f | d |g| d
X X X
Z 1/s
s
|f g| d kf kp kgkq ,
X

as, f g Ls (). Finalmente, invocamos una vez mas el Teorema 2.22, es


decir
Z Z 1/s Z 1/r
s r
|f gh| d |f g| d |h| d
X X X
kf kp kgkq khkr .

n
X 1
Corolario 2.25. Sea pk > 1, k = 1, 2, 3, ..., n tal que = 1, para fk
k=1
pk
Lpk () tenemos que f1 , f2 , . . . , fn L1 () y
Z Y n

Yn
fk d kf kpk .

X k=1

k=1
2.3 Lp no es un espacio normado 33

El siguiente resultado nos provee de otra caracterizacion de la norma k kp .


Teorema 2.26. Sea f Lp () con 1 p < . Entonces,
 
1 1 1
kf kp = sup kf gk1 kgkq : g 6= 0, + = 1 .
gLq () p q

Demostracion. En virtud de la desigualdad de Holder se tiene


Z
kf gk1 = |f g| d kf kp kgkq ,

entonces,
kf gk1 kgk1
q kf kp
para g 6= 0, luego
 
1 1 1
sup kf gk1 kgkq : g 6= 0, + = 1 kf kp . (2.4)
gLq () p q
Por otra parte, supongamos f 6= 0 y g = c|f |p1 (c constante), entonces
|f g| = c|f |p ,
as,
kf gk1 = ckf kpp .
Si escogemos c = kf k1p
p , obtenemos

kf gk1 = kf k1p p
p kf kp = kf kp . (2.5)
Ahora bien,
|g|q = cq |f |q(p1)
e integrando ambos lados tenemos que
Z 1/q
p
kgkq = c |f | d

= kf kp1p kf kp/q
p
= kf kp1p kf kpp1
= 1.
Como f 6= 0, entonces kgk1
q = 1.

As, podemos escribir (2.5) como


 
1 1 1 1
kf kp = kf gk1 kgkq sup kf gk1 kgkq : g 6= 0, + = 1 . (2.6)
gLq () p q
Combinando (2.4) y (2.6) obtenemos el resultado.
34 Los espacios Lp

Teorema 2.27. Sea f L1 () L (), entonces

a) f Lp () para 1 < p < .

b) lm kf kp = kf k .
p

Demostracion. a) Sea f L1 () L (), entonces |f | kf k c.t.p, enton-


ces |f |p1 kf kp1 , |f |p kf kp1
, as
p p1
|f |, luego kf kp kf k kf k1 , de
donde
1 1 1
kf kp kf k p kf k1p . (2.7)
De esta manera hemos demostrado que f Lp ().

b) En virtud de (2.7) tenemos

lm sup kf kp kf k . (2.8)
p

Por otra parte, sea 0 <  < 21 kf k y

A = {x X : |f (x)| > kf k },

note que (A) > 0, entonces


Z Z
p
|f | d |f |p d (kf k )p (A),
X A

luego
1
lm inf kf kp (kf k ) lm inf [(A)] p ,
p p

de la arbitrariedad de  resulta

lm inf kf k , (2.9)
p

combinando (2.8) y (2.9) resulta

kf k lm inf kf kp lm sup kf kp kf k .
p p

As que
lm kf kp = kf k .
p
2.3 Lp no es un espacio normado 35

Lema 2.28 (Desigualdad de Markov). Sea f Lp () y g una funcion cre-


ciente en [0, ). Entonces,
R
g |f | d
({x X : |f (x)| > }) X ,
g()
donde g(x) 6= 0 para todo x [0, ).

Demostracion. Sea A = {x X : |f (x)| > } donde > 0, luego < |f (x)|


para todo x A , entonces
g()A (x) g(|f (x)|)A (x)
, integrando ambos lados resulta
R
X
g |f | d
({x X : |f (x)| > }) .
g()

Teorema 2.29. Si 1 p . Entonces (Lp (), k kp ) es un espacio com-


pleto.

Demostracion. Caso 1. 1 p < . Sea {fn }nN una sucesion de Cauchy


en Lp (). Entonces, para todo  > 0 existe n0 N tal que
kfn fm kpp < p
siempre que n, m n0 . En virtud de la desigualdad de Markov con
g() = p , obtenemos
p ({x : |fn (x) fm (x)| }) kfn fm kpp
siempre que n, m n0 . Esto ultimo nos dice que {fn }nN es una suce-
sion de Cauchy en medida, por lo tanto existe una subsucesion {fnk }kN
de {fn }nN que converge c.t.p a una funcion f medible. Por el Lema de
Fatou se tiene
Z Z
kf kp = |f | d lm inf |fnk |p d < .
p p
k

As f Lp (). Invocando una vez mas el Lema de Fatou podemos ver


que
Z Z
kfn f kp = |fn f | d lm inf |fn fnk |p d < p
p p
k

siempre que n n0 . Es decir fn converge a f en Lp ().


36 Los espacios Lp

Caso 2. p = . Sea {fn }nN una sucesion de Cauchy en L (). Para cada
n N definamos
Ak = {x : |fk (x)| > kfk k }
y para cada n, m N, sea

Bn,m = {x : |fn (x) fm (x)| > kfn fm k }.

Note que cada Ak y cada Bn,m tienen medida cero. Sea



! !
[ [
E= Ak Bn,m ,
k=1 n,m

entonces (E) = 0. Note que cada fn (x) es una funcion real y ademas

|fn (x) fm (x)| kfn fm k , x X \ E.

ltimo nos dice que {fn }nN es una sucesion uniforme de Cauchy
Esto u
en X \ E. Ahora, definamos
(
lm fn (x) si x X \ E;
f (x) = n
0 si x E.

Entonces f es medible, dado que f = lm fn E c , es decir, fn f


n
uniformemente en E c . Finalmente, queremos demostrar que

lm kfn f k = 0.
n

En efecto, dado  > 0, existe n1 N tal que |fn (x) f (x)| < /4
x X \ E siempre que n n1 . As,

{x : |fn (x) f (x)| /2} E para n n1 .

Como (E) = 0 concluimos que

kfn f k /2 <  siempre que n n1 ,

es decir, lm kfn f k = 0, en particular kfn1 f k < , as fn1 f


n
L (). Ahora, como fn1 L () y L () es un espacio vectorial,
entonces f = fn1 (fn1 f ) L ().

Observaci on 2.30. Como una consecuencia del Teorema 2.29 podemos ver
que para 1 p , Lp () es un espacio de Banach, es decir, un espacio
normado el cual es completo con respecto a la metrica inducida por la norma.
2.3 Lp no es un espacio normado 37

Teorema 2.31. Sea {fn }nN una sucesion de funciones en Lp () con 1


p < , la cual converge c.t.p a una funcion f Lp (). Entonces,

lm kfn f kp = 0 si y solo si lm kfn kp = kf kp .


n n

Demostracion. Dado que (t) = tp (con 1 p < ) es convexa en [0, ).


Entonces,
a b p
 p
|a| + |b| 1
(|a|p + |b|p ),
2 2 2
as,
|a b|p 2p1 (|a|p + |b|p ). (2.10)
Como fn f c.t.p, entonces |fn f |p 0 c.t.p. En vista de (2.10), obtene-
mos
0 2p1 (|fn |p + |f |p ) |fn f |p ,
luego,
lm 2p1 (|fn |p + |f |p ) |fn f |p = 2p |f |.
 
n

Por el Lema de Fatou se tiene que


Z Z
p p
lm inf 2p1 (|fn |p + |f |p ) |fn f |p d
 
2 |f | d =
X X Z
 p1
2 (|fn |p + |f |p ) |fn f |p d

lm inf
X
Z  Z 
p p p
=2 |f | d + lm inf |fn f | d ,
X X

es decir,
Z Z Z
p p p p
2 |f | d 2 |f | lm sup |fn f |p d,
X X X

por lo tanto Z
lm sup |fn f |p d 0.
X
Como
Z Z
p
0 lm inf |fn f | d lm sup |fn f |p d 0,
X X

entonces,
lm kfn f kp = 0
n
38 Los espacios Lp

si lm kfn kp = kf kp .
n

Ahora, si lm kfn f k = 0, entonces de la desigualdad


n

|kfn kp kf kp | kfn f kp .
se sigue el resultado.
Observaci on 2.32. Para p = , el Teorema 2.31 es falso. En efecto, sea
{fn }nN L ([0, 1]) definida por fn = (1/n,1] , note que fn 1 c.t.p en
[0, 1]. Ademas,
  
kfn k = nf M : x [0, 1] : (1/n,1] (x) > M = 0 = 1
y k1k = 1, luego lm kfn k = k1k . Pero
n

kfn 1k = sup (1/n,1] (x) 1 = 1.
x(0,1]

2.4. Los espacios lp


2.4.1. Desigualdad de H
older y Minkowski versi
on dis-
creta
En esta seccion estudiamos la desigualdad de Holder y Minkowski para
sumas finitas
Lema 2.33 (Desigualdad de Holder). Sean p y q n umeros reales con 1 <
p < y 1 < q < , de modo que p1 + 1q = 1, entonces

n n
!1/p n
!1/q
X X X
|xk yk | |xk |p |yk |q
k=1 k=1 k=1
n
para todo x, y R .
|xk |p |yk |q
Demostracion. = P
n y = n
P .
|xk |p |yk |q
k=1 k=1
En virtud del corolario 1.14, resulta

|xk ||yk | 1 |xk |p 1 |yk |q


1/p  n 1/q n + n .
pP qP
 n
P P |xk |p |yk |q
|xk |p |yk |q
k=1 k=1
k=1 k=1
2.4 Los espacios lp 39

Sumando miembro a miembro se tiene que


n
P
|xk ||yk |
k=1 1 1
1/p 1/q +
p q
 n
 n
P P
|xk |p |yk |q
k=1 k=1

y de aqu
n n
!1/p n
!1/q
X X X
|xk yk | |xk |p |yk |q ,
k=1 k=1 k=1

que era lo que se quera demostrar.

Lema 2.34 (Desigualdad de Minkowski). Sea p 1, entonces

n
!1/p n
!1/p n
!1/p
X X X
|xk + yk |p |xk |p + |yk |p
k=1 k=1 k=1

para todo x, y Rn .

Demostracion. Consideremos
n
X n
X
p
|xk + yk | = |xk + yk |p1 |xk + yk |
k=1 k=1
Xn n
X
p1
|xk ||xk + yk | + |yk ||xk + yk |p1
k=1 k=1

En virtud del lema 2.33 se tiene


!1/p !1/p n !1/q
n
X Xn n
X X
|xk + yk |p |xk |p + |yk |p |xk + yk |(p1)q ,
k=1 k=1 k=1 k=1

1 1
Como+ = 1, entonces p = (p 1)q, de donde
p q
!1/p !1/p n !1/q
n
X n
X n
X X
p p p p
|xk + yk | |xk | + |yk | |xk + yk | ,
k=1 k=1 k=1 k=1

as
40 Los espacios Lp

n
!1 1q n
!1/p n
!1/p
X X X
|xk + yk |p |xk |p + |yk |p .
k=1 k=1 k=1

Finalmente
n
!1/p n
!1/p n
!1/p
X X X
p p p
|xk + yk | |xk | + |yk | ,
k=1 k=1 k=1

que era lo que se quera demostrar.

2.4.2. El espacio lp y su norma


lp con 1 p < representara al conjunto de todas las sucesiones de
n
umeros reales
x = (1 , 2 , ...)
tales que

X
|k |p < .
k=1

lp sera de utilidad a la hora de introducir los espacios de Sobolev y las series


de Fourier en el toro (torus).
Mostremos que lp es un subespacio del espacio R . Sea x e y elementos
de lp y , n umeros reales, entonces en virtud del lema 2.34 se tiene que

n
!1/p n
!1/p n
!1/p
X X X
k |p
|k + e || |k |p + || k |p
|e
k=1 k=1 k=1

n
!1/p
!1/p
!1/p
X X X
k |p
|k + e || |k |p + || k |p
|e .
k=1 k=1 k=1

Por lo tanto

!1/p
!1/p
!1/p
X X X
p p p
|k + e
k | || |k | + || |e
k | , (2.11)
k=1 k=1 k=1

esto u
ltimo muestra que x + y es un elemento de lp y por ende lp es un
subespacio de R .
2.4 Los espacios lp 41

Observaci
on 2.35. El resultado (2.11) generaliza la desigualdad de Min-
kowski.
Sea x lp , tomemos como norma de x la funcion

!1/p
X
kxkp = |k |p ,
k=1

dejamos como ejercicio demostrar que en efecto esta funcion define una nor-
ma sobre lp . En consecuencia lp resulta un espacio normado.

El siguiente resultado nos permite obtener lp por medio de Lp (X, )


Teorema 2.36. Sea X un conjunto contable y la medida de contar sobre
X, entonces
Lp (X, ) = lp .
Demostracion. Sea la medida de contar sobre X es decir

numero de elementos de E ,si E es un conjunto finito
(E) =
,si E es un conjunto infinito

Sin perdida de generalidad supongamos que X = Z+ , ya que X, dotado de la



medida de contar, es isomorfo a Z+ , entonces podemos escribir Z+ =
S
{k},
k=1
sea f Lp (Z+ , ) y
n
X
n = |f (k)|p {k}
k=1

una sucesion de funciones simples tal que

lm n (k) = |f (k)|p para cada k,


n

ahora
Z n
X
|f (k)|p Z+ {k}

n d =
k=1
Z+
n
X
|f (k)|p {k}

=
k=1
n
X
= |f (k)|p ,
k=1
42 Los espacios Lp


ya que {k} = 1.

Claramente 1 2 3 . . . , en virtud del teorema de la convergencia


monotona tenemos
Z Z
p
|f (k)| d = lm n (k) d
n
Z+ Z+
X
= |f (k)|p ,
k=1

Como Z
X
p
|f (k)| = |f (k)|p d.
k=1
Z+
p
ltimo muestra que |f | es integrable si y solo si
Esto u

X
|f (k)|p < .
k=1

En otras palabras, decir que f pertenece a Lp (X, ) dotado de la medida de


contar es equivalente a decir que la sucesion {f (k)}kN es un miembro de lp ,
por lo tanto
Lp (X, ) = lp .

Teorema 2.37. lp es un espacio de Banach, (1 p < ).


Demostracion. Sea {xn }nN una sucesion de Cauchy en lp , donde xn =
(n) (n)
(1 , 2 , . . .), entonces para cualquier  > 0 existe un n0 N tal que si
n, m n0 , entonces kxn xm kp < , es decir


!1/p
X (n) (m)
|j j |p < , siempre que n, m n0 , (2.12)
j=1

de aqu, obtenemos que para todo j = 1, 2, 3, . . .


(n) (m)
|j j | < , siempre que n, m n0 , (2.13)
(1) (2)
escojamos j fijo de (2.13) se ve que (j , j , . . .) es una sucesion de Cauchy
en R, por lo cual existe j R tal que
(m)
lm j = j .
m
2.4 Los espacios lp 43

Definamos x = (1 , 2 , . . .) y mostremos que x esta en lp y lm xn = x.


n

De (2.12) tenemos que para todo n, m n0


k
X (m) (n)
|j j |p < p , k = 1, 2, 3, . . . ,
j=1

de aqu
k
X k
X
(n) (m) (n)
|j j |p = | lm j j |p < p .
m
j=1 j=1

Siempre que n n0 , esto muestra que


(n)
x xn = (j j ) esta en lp ,

deduciendose ademas que lm xm = x, finalmente, en virtud de la de-


m
sigualdad de Minkowski tenemos


!1/p
!1/p
X X
|x|p = |xn + x xn |p
n=1 n=1


!1/p
!1/p
!1/p
X X X
|x|p |xn |p + |x xn |p ,
n=1 n=1 n=1

lo que muestra que x esta en lp y con esto finaliza la demostracion


Observacion 2.38. Recordemos que un espacio metrico y por ende uno nor-
mado es separable si contiene un subconjunto denso numerable.
Teorema 2.39. lp es separable (1 p < ).
Demostracion. Sea M el conjunto de todas las sucesiones de la forma y =
(n1 , n2 , . . . , nn , 0, 0, . . .) donde n N y los nk Q, observe que M es contable.
A demostrar que M es denso en lp . Sea x = (k )kN en lp arbitrario, entonces
para  > 0 existe n que depende de  de modo que

X
|k |p < p /2.
k=n+1

Ahora, como Q = R, tenemos que para cada k existe un racional nk tal


que

|k nk | <
p
,
2n
44 Los espacios Lp

luego
p
|k nk |p < ,
2n
as
n
X
|k nk |p < p /2.
k=1

entonces
n
X
X
p p
kx yk = |k nk | + |k |p < p ,
k=1 k=n+1

por lo cual
kx yk < .
Mostrandose as que M es denso en lp , luego resulta lp separable.

El siguiente resultado nos muestra una profunda diferencia entre Lp y


lp . El ejemplo 2.15 demuestra que Lq () ( Lp () con 1 p q <
y (X) < , sin embargo tenemos que lp lq con 0 < p < q < . Es
importante aclarar que este u ltimo resultado no contradice el ejemplo 2.15
+
dado que Z con la medida de contar no tiene medida finita.

Teorema 2.40. Si 0 < p < q < , entonces lp lq .



||p < , en virtud de esto existe
P
Demostracion. Sea x lp , entonces
n=1
n0 N tal que si n n0 , entonces |n | < 1.

Ahora, como 0 < p < q, entonces 0 < q p, as |n |qp < 1 si n > n0 , por
lo cual
|n |q < |n |p si n > n0 .
Sea M = max{|1 |qp , |2 |qp , . . . , |n0 |qp , 1}, luego

X
X
q
|n | = |n |p |n |qp
n=1 n=1

X
<M |n |p < +,
n=1

por lo tanto x lq .
2.4 Los espacios lp 45

Observaci on 2.41. La inclusion lp lq es propia, en efecto, sea xn = n1/p


q
para todo n N con 1 p < q , luego puesto que p < q, entonces > 1,
p
ahora consideremos

X
q
X 1
|xn | = q/p
< .
n=1 n=1
n

La u
ltima serie es convergente en virtud del criterio de las series p,
as xn lq , por otra parte

X
p
X 1
|xn | = , (Serie armonica)
n=1 n=1
n

por lo tanto xn
/ lp .

2.4.3. Desigualdad de Hardy en lp


Teorema 2.42 (Desigualdad de Hardy). Sea {an }nN una sucesion de n
P ume-
p
ros reales positivos tal que n=1 an < , entonces

n
!p 
p X
X 1X p
ak apn .
n=1
n k=1 p1 n=1

An
Demostracion. Sea n = n
donde An = a1 + a2 + + an , luego

An = nn ,

as
a1 + a2 + + an = nn , (2.14)
de donde
an = nn (a1 + a2 + + an1 )
por (2.14)
an = nn (n 1)n1 .
Consideremos ahora
p p
np np1 an = np [nn (n 1)n1 ] np1
p1 p1
pn p(n 1)
= np n np1 + n1 np1 .
p1 p1
46 Los espacios Lp

En virtud del corolario 1.14 resulta


p q(p1)
p(n 1) p(n 1) n1 p(n 1) n
n1 np1 +
p1 p1 p p1 q
 
n1 p p(n 1) 1
= + 1 np
p 1 n1 p1 p
n1 p
= + (n 1)np ,
p 1 n1
luego
p pn p n 1 p
np np an np + + (n 1)np
p1 p 1 n p 1 n1
p
pnp np pnnp (n 1)n1 + (p 1)(n 1)np
= +
p1 p1
p
p n pnn + (n 1)n1 + (pn p n + 1)np
p p p
= n
p1
1  p
nnp ,

= (n 1)n1
p1
de donde
N N N
X p X p 1 X p
np nnp

n an (n 1)n1
n=1
p 1 n=1 p 1 n=1
1
= [1p + 1p 22p + N N
p
]
p1
p
N N
= 0.
p1
As
N N
X p X p1
np an .
n=1
p 1 n=1 n
Por la desigualdad de Holder se tiene que

! p1
! 1q
X p X X
np apn nq(p1)
n=1
p1 n=1 n=1

! p1
! 1q
p X X
= apn np ,
p1 n=1 n=1
2.4 Los espacios lp 47

luego


!1 1q
! p1
X p X
np apn
n=1
p 1 n=1
 p  p X
X a1 + a2 + + an p
apn
n=1
n p 1 n=1

!p 
X 1X p
p X
ak apn .
n=1
n k=1
p 1 n=1

2.4.4. Desigualdad de Hilbert en lp


Recordemos algunos resultados basicos de analisis complejo.
 
1 X 1 1
= + (1)n + . (2.15)
sen(z) z n=1 z+n zn

Consideremos la funcion
1
f (z) = (p > 1)
p
z(z + 1)

definida en la region D1 = {z C : 0 < |z| < 1}. Queremos obtener su


desarrollo en terminos de la serie de Laurent. En efecto, si |z| < 1, entonces

1 1 X
= = (z)n
1+z 1 (z) n=0

X
= (1)n z n ,
n=0

luego

X 1
f (z) = (1)n z n p . (2.16)
n=0

En este mismo orden de ideas, consideremos

1
g(z) = 1+ p1 1

z 1+ z
48 Los espacios Lp


definida en la region D2 = {z C : |z| > 1}, dado que z1 < 1, entonces
 n
1 1 X 1
= =
1 + z1 1 ( z1 ) n=0 z

X
= (1)n z n .
n=0

Por lo tanto
X 1
g(z) = (1)n z n1 p . (2.17)
n=0

Proposici on 2.43. Para cada n


umero positivo m y para cada n
umero real
p > 1 se tiene que
1
X mp
1  .
n=1 n (m + n)
p
sen p

Demostracion. Note que


1 1
Z
X mp mp
1 1 dx
n=1 n p (m + n) 0 x p (m + x)
Z
dz
= 1
0 z p (1 + z)
Z 1 Z
dz dz
= 1 + 1+ 1 .
0 z p (1 + z) 1 z p 1 + z1

En virtud de (2.16) y (2.17) se deduce que


1
Z 1 X ! Z X
!
X mp 1 1
1 (1)n z n p dz + (1)n z n1 p dz
n=1 n (m + n)
p 0 1
n=0 n=0
Z 1 Z
X 1 X 1
= (1)n z n p dz + (1)n z n1 p dz
n=0 0 n=0 1

X (1)n (1)n X
= +
n=0
n p1 + 1 n=0 p1 + n

X (1)n X (1)n
= 1 +p+ 1
n=1 p
n n=1 p
+n

!
X 1 1
=p+ (1)n 1 + 1
n=1 p
n p
+n
2.4 Los espacios lp 49


=  .

sen p

ltimo se obtuvo de (2.15) con z = p1 .


Esto u

Teorema 2.44 (Desigualdad de Hilbert). Sean p, q > 1 tales que p1 + 1q = 1


umeros no negativos tales que p
P
{an }nN , {bn }nN sucesiones de n
yP m=1 am
y q
n=1 bn son convergentes. Entonces


! p1
! 1q
X am b n X X
  apm bqn
m,n=1
m + n sen p m=1 n=1

Demostracion. En virtud de la desigualdad de Holder y de la proposicion


anterior resulta

X am b n
m,n=1
m+n
1 1
X m pq am n pq bn
= 1 1 1 1
m,n=1 n (m + n) m (m + n) q
pq p pq

1
! ! p1 1
! ! 1q
X mq X n p
1 apm 1 bqn
m,n=1 n (m + n)
q
m,n=1 m (m + n)
p

1 ! ! 1q
1 1
! ! p
X X mq X X n p
= 1 apm 1 bqn
n=1 n (m + n) m=1 m (m + n)
q p
m=1 n=1
1 1

! p
! q
X p X q
a b
m=1
sen q m n=1
sen p n

! p1 ! 1q
X p X
q
am bn
m=1
sen p n=1
sen p
! p1 ! 1q
! p1 ! 1q
X
p
X
= a m bqn
sen p sen p m=1 n=1

! p1 ! 1q
X
p
X
= a bqn .
sen p m=1 m n=1
50 Los espacios Lp

As hemos demostrado que


! p1
! 1q
X am b n X X
  apm bqn .
m,n=1
m+n sen p m=1 n=1

2.4.5. El espacio l
l representara al conjunto de todas las sucesiones reales {n }nN aco-
tadas, es claro que l es un espacio vectorial. Como norma de x l ,
x = (1 , 2 , . . . , n , . . .) tomaremos

kxk = sup |n |,
nN

resultando as l un espacio normado.


Teorema 2.45. l es un espacio de Banach.
Demostracion. Sea {xn }nN una sucesion de Cauchy en l , donde xn =
(n) (n)
(1 , 2 , . . .), entonces para cualquier > 0 existe n0 > 0 tal que si

m, n n0 , entonces kxm xn k < ,

luego para j fijo tenemos que si m, n n0 , entonces


(m) (n)
|j j | < (2.18)
(1) (2)
as, resulta que para todo j fijo la sucesion (j , j , . . .) es una sucesion de
(m)
Cauchy en R, por lo cual existe j R tal que lmm j = j .
Definamos x = (1 , 2 , . . .), ahora queremos demostrar que x l y
lmn xn = x.
De (2.18) tenemos que si n n0 , entonces

(n) (m) (n)
j j = lm j j < , (2.19)

n
 
(n) (n)
puesto que xn = j l , existe un n
umero real Mn tal que j
jN
Mn para todo j.
Por la desigualdad triangular tenemos

(n) (n)
|j | j j + j < + Mn

2.4 Los espacios lp 51

siempre que n n0 , esta desigualdad se mantiene para cualquier j, ademas,


observe que el lado derecho no depende de j, por lo tanto (j )jN es una
umeros reales acotada, esto implica que x = (j )jN l .
sucesion de n
De (2.19) tambien obtenemos

(n)
kxn xk = sup j j < .

jN

siempre que n n0 . De esto u


ltimo concluimos que

lm xn = x
n

y as l es completo.
El siguiente resultado nos muestra una manera naturalde introducir la
norma en el espacio l .
Teorema 2.46. lmp kxkp = kxk , donde

n
! p1
X
kxkp = |xk |p .
k=1

Pn 1
Demostracion. Observe que |xk | ( k=1 |xk |p ) p , as

|xk | kxkp

para k = 1, 2, 3, . . . , n, de donde

sup |xk | kxkp ,


1kn

luego
kxk lm inf kxkp . (2.20)
p

Por otra parte, note que


n
! p1 n  p ! p1
X X 1
|xk |p sup |xk | = n p kxk ,
1kn
k=1 k=1

luego
1
kxkp n p kxk ,
as  
1
lm sup |xkp lm sup n kxk = kxk ,
p
p p
52 Los espacios Lp

es decir
lm sup |xkp kxk . (2.21)
p

Combinando (2.20) y (2.21) resulta

kxk lm inf kxkp lm sup kxkp kxk ,


p p

de esto u
ltimo se concluye que

lm kxkp = kxk .
p

Ejercicios

1. Si f L (X, A, ) y (X) < . Demostrar que lmp kf kp = kf k .


2. Demuestre que las siguientes funciones:
a) f (x) = ex para todo x [0, 1].
1
b) f (x) = x2
para todo x [1, +).
2
c) f (x) = ex para todo x R ( > 0).
d) f (x) = xn para todo x [0, a], n N, a R+ .
e) f (x) = 1 ex para todo x [0, 1].
Son esencialmente acotadas. Hallar su norma esencial.
3. En todos los casos anteriores demuestre que

lm kf kp = kf k .
p

4. Dada f (x) = ln(1 x) para todo x [0, 1). Demuestre que f


/
L ([0, 1), L, m) pero f Lp ([0, 1], L, m) con 1 p < .
5. Demuestre que el resultado del problema 1 es falso si (X) = +.
6. Sea f L (X, A, ) tal que kf k > 0. Si 0 < (X) < , demostrar
que
n+1
lm = kf k
n n

donde Z
n = |f |n d, n = 1, 2, 3, . . .
X
2.4 Los espacios lp 53

7. Sea I = [0, 1], sea f L1 (I, L, m) y S = {x I : f (x) Z}. Demostrar


que Z
lm | cos f (x)|n dm = m(S).
n I

8. Sea f Lp (X, A, ) con (X) = 1 y p > 0. Demostrar que


R
ln(f ) d
lm kf kp = e X .
p0

9. Sea (X, A, ) un espacio de medida y f Lp () (1 p < ). Su-


pongamos que {En }nN es una sucesion de conjuntos medibles tal que
(En ) = n1 (n N). Demostrar que
Z
p1
lm n p |f | d = 0.
n En

10. Sea (X, A, ) un espacio de medida


R ntal que (X) < y f una funcion
positiva A-medible. Si lmn X f d < , demostrar que
Z
lm f n d = ({x X : f (x) = 1}) .
n X

11. Sea f Lp0 (X, A, ) para algun 0 < p0 < . Demostrar que
Z
lm |f |p d = ({x X : f (x) 6= 0}) .
p0 X

12. Sea u, v L4 (X, A, ) y w L2 (X, A, ). Demostrar que


Z Z 1/4 Z 1/4 Z 1/2
4 4 2
uvw d |u| d |v| d |w| d .

X X X X
R
13. Sea p > 1 y f Lp ([1, +), L, m). Defina g(x) = 1
f (t)etx dt.
Demostrar
a) g L1 ([1, +), L, m).
 1/q  
b) kgk1 1 p1 kf kp p1 + 1
q
=1 .

14. Para 1 p < y 0 < (X) < se define


 Z 1/p
1 p
Np (f ) = |f | d .
(X) X
Demostrar que
54 Los espacios Lp

a) Si p1 < p2 entonces Np1 (f ) Np2 (f ).


b) Np (f + g) Np (f ) + Np (g).
 
1
c) (X) X |f g| d Np (f )Nq (g) con p1 + 1
R
q
=1 .
d ) lmp Np (f ) = kf k .

15. Sea (X, A, ) un espacio de medida. Sean f y g funciones


R A-medibles
t
Ry positivas sobre X. Sean 0 < t < r < m < . Si X f g d < y
m
X
f g d < , demostrar que
Z mt Z mr Z rt
r t m
f g d f g d f g d .
X X X

Nota. Esta desigualdad se conoce como la desigualdad de Rogers.


R R
16. Si X f d < y X f g m d < para m > 1. Demostrar que
Z m Z m1 Z 
m
f g d f d f g d .
X X X

17. Usar el problema 13 para dar una demostracion alternativa del corolario
2.24.

18. Demostrar el corolario 2.25.

19. Sea h una funcion creciente en (0, +). Si 0 < 1 y 0, demos-


trar que
Z  Z
1 1
t h(t) dt tp1 [h(t)] dt.
0 0

20. Si f es una funcion no negativa y decreciente en (0, +) para p 1.


Demostrar que
Z Z p
p p
(f (x)) dx f (x) dx .
0 0

21. Sea f L1 (). Demostrar que


Z  
2 /2
({x X : |f (x)| > }) 2e cosh f d.
X 2

22. Sean 0 < < 1, b > 1 y m > 1. Demuestre que


2.4 Los espacios lp 55

a) |am bm | m|b a| max{am , bm }.


b) |x y|p |xp y p | para x, y R+ y 1 p < .
c) |ln b ln a| 1 |b a| max{a , b }.
d) ||b|p |a|p | p|b a| max{|b|p1 , |a|p1 } (1 p < ).
23. Para 1 p < . Demostrar que
n R p o
a) kf kp = nf 0 : X |f| dm 1 .
b) a) define una norma en Lp .
24. Sea E = {p (0, ) : kf kp < }. Demuestre que E es un intervalo.
25. Sea {fn }nN una sucesion de funciones reales en L4/3 ((0, 1), L, m) tal
que fn 0 en medida (n ) y (0,1) |fn (x)|4/3 dx 1. Demostrar
R

que Z
lm |fn (x)| dx = 0.
n (0,1)

26. Sea (X, A, ) un espacio de medida y {fn }nN una sucesioP


n de funciones
medibles tal que fn Lp (X, A, ) para todo n N. Si n=1 kfn kp <
. Demostrar que
X X
fn kfn kp .



n=1 p n=1

27. Demostrar que si kf + gkp = kf kp + kgkp , entonces


f g
= c.t.p.
kf kp kgkp

28. Sea fn f en Lp 1 p < y sea {gn }nN una sucesion de funciones


medibles tal que |gn | M n y gn g c.t.p. Demostrar que
gn fn gf en Lp .

29. Utilizar el corolario 1.14 para deducir la desigualdad de Holder.


30. Utilizar el problema 29 para deducir la desigualdad de Minkowski.
31. Sea (X, A, ) un espacio de medida tal que (X) = 1. Hallar todas las
funciones en (0, +) tal que
  Z
lm kf kp = (f ) d.
p0 X
56 Los espacios Lp

32. Sean p, q, r R+ tal que p1 + 1q = 1r . Si f Lp () y g Lq (), demostrar


que f g Lr () y
kf gkr kf kp kgkq .

33. Si 0 < p < q y (X) = 1, demostrar que


Z 1/p Z 1/q
p q
|f | d |f | d .
X X

34. Use el Corolario 1.11 para demostrar el Teorema 2.22 (Desigualdad de


Holder).

35. La funcion f : [a, b] R se dice que tiene p-variacion acotada si


n
X |f (xk ) f (xk1 )|
Vp (f, [a, b]) = sup < +,
k=1
|xk xk1 |p1

con 1 < p < , donde el supremo se toma sobre todas las particiones
de [a, b]. El conjunto de todas las funciones con p-variacion se denota
por BVp ([a, b]). Si f BVp ([a, b]) demostrar que f 0 Lp ([a, b], L, m) y
ademas
Vp (f, [a, b]) = kf 0 kp .
T
36. (a) Sea f 1p< Lp () y supongamos que hay una constante C tal
que kf kp C, 1 6 p < . Demuestre que f LT ().
(b) De un ejemplo de una funcion f tal que f 1p< Lp () pero
f/ L ().

Sean {an }nZ y {bnP


37. P }nZ sucesiones de n
umeros reales tales
P que k =
p
n= |an | < y m= |bm | < (p > 1). Sea Cn = m= anm bm .
Demostrar que

p 1/p
P  1 1
a) |Cn | k 1/q m= |anm ||bm | donde p
+ q
= 1.
p 1/p p 1/p
P  P 
b) n= |C n | k n= |b n | .

38. Si an > 0 para n = 1, 2, 3, . . . demostrar que



X X
n
a1 a2 an e an .
n=1 n=1
2.5 Aproximaciones en Lp () 57

39. Si a1 a2 ak an 0 y > 0. Demostrar que

n
!1/ n
!1/
X X
ak ak .
k=1 k=1

40. Demostrar que l no es separable.

2.5. Aproximaciones en Lp()


Sea (X, d) un espacio metrico, recordemos que un subconjunto D de X,
D X es denso en X si D = X, es decir dado x X, para todo  > 0 existe
d D tal que d(x, d) < .

En otro orden de ideas, sea (X, A, ) un espacio de medida. Que una funcion
simple s se anule fuera de un conjunto de medida finita significa que

({x X : s(x) 6= 0}) < .


n
X
Ahora, supongamos que s = k Ek donde k 6= 0 para 1 k n y Ek
k=1
A. Si s se anula fuera de un conjunto de medida finita, entonces (Ek ) <
para 1 k n, luego
Xn
p
kskp = |k |p (Ek ).
k=1

As, s Lp () si y solo si s se anula fuera de un conjunto de medida finita.

Lema 2.47. Para 1 p < , el conjunto de las funciones simples A-


medibles las cuales se anulan fuera de un conjunto de medida finita es denso
en Lp ().

Demostracion. Sea 1 p < y f Lp (), queremos demostrar que dado


 > 0 existe una funcion s simple, A-medible la cual se anula fuera de un
conjunto de medida finita tal que kf skp < . Para ello consideremos dos
casos:

Caso 1 f 0. Sabemos que existe una sucesion {sn }nN de funciones sim-
ples no negativas y A-medibles tal que sn f puntualmente en X,
dado que 0 sn f para todo n y f Lp () se tiene que sn Lp (),
58 Los espacios Lp

esto significa que cada sn se anula fuera de un conjunto de medida


finita, ahora, observe que

lm |sn f |p = 0 en X
n

y
|sn f |p (|sn | + |f |)p (2|f |)p = 2p |f |p ,
dado que f Lp (), entonces 2p f p L1 (). En virtud del Teorema de
la convergencia dominada tendremos que
Z
lm |sn f |p d = 0,
n X

es decir,
lm ksn f kpp = 0,
n

luego, dado  > 0 existe n0 N tal que

ksn f kpp < p ,

ahora, escogemos s = sn0 , entonces

ks f k < .

Caso 2 f A-medible, entonces f = f + f donde f + y f son funciones


no negativas A-medibles. Por el caso 1 existen funciones simples s1 y
s2 no negativas A-medibles las cuales se anulan fuera de un conjunto
de medida finita tal que

kf + s1 kp < /2 y kf s2 k < /2.

Sea s = s1 s2 , note que s es una funcion simple, A-medible, la cual


se anula fuera de un conjunto de medida finita. Finalmente, por la
desigualdad de Minkowski tenemos

kf skp = k(f + s1 ) (f s2 )kp


kf + s1 kp + kf s2 kp
< .

Lema 2.48. El conjunto de las funciones simples es denso en L ().


2.5 Aproximaciones en Lp () 59

Demostracion. Sea  > 0 y f L (), entonces |f | kf k c.t.p en X,


as existe E A con (E) = 0 y |f (x)| kf k para todo x X \ E.
Definamos 
f (x) si x X \ E;
f (x) =
0 si x E.
Entonces, |f (x)| kf k para todo x X, luego existe una sucesion {tn }nN
de funciones simples tal que tn f uniformemente en X, as, existe n0 N
tal que
|tn (x) f (x)| < /2 x X,
luego,
|tn0 (x) f (x)| < /2 x X \ E,
esto significa que con s = tn0
ks f k /2 < .

Sea (X, d) un espacio metrico y E X , definamos


d(x, E) = nf{d(x, e) : e E}.
De cursos anteriores sabemos que:
i) d(x, E) = 0 si solo si x E.
ii) d(, E) : X R+ es continua en X.
Lema 2.49 (Lema de Urysohn). Sea (X, d) un espacio metrico. Sea F un
conjunto cerrado en X y V un conjunto abierto en X tal que F V . En-
tonces, existe g : X [0, 1] tal que g es continua en X, g(x) = 1 para todo
x F y g(x) = 0 para todo x X \ V .

Demostracion. Para x X, definamos


d(x, X \ V )
g(x) = .
d(x, F ) + d(x, X \ V )
En virtud de II) es claro que g es continua en X.
d(x,X\V )
Si x F , entonces g(x) = 0+d(x,X\V )
= 1.
Si x X \ V , entonces d(x, X \ V ) = 0, as g(x) = 0 para todo x X \ V .

Ahora, dado que d(x, X \ V ) 0 y d(x, F ) 0 podemos ver que


0 g(x) 1.
Con esto finaliza la demostracion.
60 Los espacios Lp

Observaci on 2.50. El lema de Urysohn tambien es valido para espacios


topologicos.

Teorema 2.51. Sea f Lp (R, L, m) con 1 p < , para todo  > 0, existe
una funcion continua g Lp (R, L, m) tal que kf gkp < .

Demostracion. Caso 1 Sea f = E , entonces E L y m(E) < , por lo


tanto podemos hallar un conjunto Fcerrado y un conjunto V abierto
p
tal que F E V y m(V \ F ) < 2 . Consideremos ahora g definida
como en el Lema anterior, entonces g es continua en R, g = 1 en F y
g = 0 en X \ V . Por otra parte,

{x : g(x) 6= f (x)} V \ F,

en efecto, si x0 / V \ F , entonces x0 (V \ F )c , as x0 V c F ,
luego si x V c , entonces f (x0 ) = g(x0 ) = 0, lo que significa que
x0
/ {x : g(x) 6= f (x)}, de esta manera hemos exhibido que

{x : g(x) 6= f (x)} V \ F.

Entonces, Z Z
p
|f g| dm |f g|p dm.
R V \F

Pero |f g| 2, entonces
Z Z
p p
|f g| dm 2 dm
R V \F

= 2p m(V \ F )
p
< 2p p ,
2
es decir,
kf gkp < .
Demostrandose as el caso 1.

Caso 2 Sea f una funcion simple A-medible, la cual se anula fuera de un


conjunto de medida finita, sea
n
X
f= k Ek , donde k 6= 0 para 1 k n
k=1
2.5 Aproximaciones en Lp () 61

y m( Ek ) < , Ek A.
Por el caso 1, para todo k N existe una funcion continua gk tal que

kEk gk kp < .
n|k |
Note que g = nk=1 k gk es una funcion continua en R y ademas en
P
virtud de la desigualdad de Minkowski

Xn
kg f kp = k (gk Ek )


k=1 p
n
X
|k |kgk Ek kp
k=1
n
X 
< |k |
k=1
n|k |
= .

Demostrandose as el caso 2.

Caso 3 Sea f una funcion arbitraria. En virtud del Lema 1 existe una fun-
cion s simple la cual se anula fuera de un conjunto de medida finita tal
que
kf skp < /2,
por el caso 2 existe una funcion continua g tal que

ks gkp < /2.

Entonces,
kf gkp kf skp + ks gkp < .
ltimo significa que (f g) Lp .
Esto u

Con esto finaliza la demostracion.

Observaci on 2.52. El Teorema 2.51 no es cierto para p = . Para ver esto,


consideremos el siguiente ejemplo. Sea 0 <  < 1/2 y f = (a,b) con a < b,
a, b R. Supongamos que existe una funcion continua g tal que kf gk < ,
de aqu obtenemos que |(a,b) (x) g(x)| <  c.t.p. Ahora, para cada > 0
podemos hallar x0 (a, a + ) y x1 (a , a) tal que

|(a,b) (x0 ) g(x0 )| < 


62 Los espacios Lp

y
|(a,b) (x1 ) g(x1 )| < ,
es decir,
|1 g(x0 )| <  y |g(x1 )| < . (2.22)
Como g es continua en a, entonces

g(a+) = g(a) = g(a). (2.23)

De la definicion de g(a+) y g(a), existe > 0 tal que

|g(x) g(a+)| <  si a < x < a + y


|g(x) g(a)| < /2 si a < x < a. (2.24)

Para este > 0, en virtud de (2.22) tenemos

1  < g(x0 ) < 1 + /2

y
/2 < g(x1 ) < /2. (2.25)
Por (2.24),

|g(x0 ) g(a+)| < /2


y
|g(x1 ) g(a)| < /2.
Por (2.25),

g(a+) > g(x0 ) /2


> 1 /2 /2
=1
> 1/2

g(a) < g(x1 ) + /2


< /2 + /2
=
< 1/2,

esto significa que g(a+) 6= g(a), lo cual contradice (2.23), por lo cual tal g
no existe.
2.5 Aproximaciones en Lp () 63

Definici
on 2.53. Una funcion escalonada es una funcion de la forma
n
X
k Ik
k=1

donde k 6= 0, 1 k n y cada Ik es un intervalo acotado.

Observaci on 2.54. Observe que cada funcion escalonada se anula fuera de


un conjunto de medida finita. As, cada funcion escalonada es un miembro de
Lp (R, L, m). Ademas, el conjunto de todas las funciones escalonadas forman
un subespacio vectorial de Lp (R, A, m).

Teorema 2.55. El conjunto de todas las funciones escalonadas es denso en


Lp (R, L, m) para 1 p < .

Demostracion. Sea f Lp (R, L, m) y  > 0.

Caso 1 f = E , entonces E L, como f Lp (R, L, m), se tiene m(E) <


, de esto u ltimo, podemos concluir que existe una union finita de
intervalos abiertos y disjuntos, digamos I tal que m(E4I) < p , sea
[n Xn
I= Ik , escojamos = Ik = I , luego
k=1 k=1
Z Z
p
|f | dm = |E I |p dm
R ZR
= |E4I |p dm
R
= m(E4I)
< ,

por lo tanto,
kf kp < .
De esta manera queda demostrado el caso 1.
n
X
Caso 2 f = k Ek donde k 6= 0 para todo k y m(Ek ) < . Por el caso
k=1
1 existe una funcion escalonada k tal que

kEk k kp < .
n|k |
64 Los espacios Lp

Note que
n
X
= k k
k=1

es una funcion escalonada y


n
X
kf kp = k k (Ek k )kp
k=1
n
X
|k |kEk k kp < ,
k=1

demostrandose as el caso 2.

Caso 3 Sea f una funcion arbitraria, en virtud del Lema 2.48 existe una
funcion simple s que se anula fuera de un conjunto de medida finita tal
que
kf skp < /2,
por el caso 2 podemos hallar una funcion escalonada tal que

ks kp < /2.

Finalmente,
kf kp < .

Teorema 2.56. Lp (R, L, m) es separable para 1 p < .

Demostracion. Definamos
( n )
X
S= bk Jk : n N, bk Q ,
k=1

donde Jk es un intervalo finito con puntos extremos racionales 1 k n.


Sea  > 0 y f Lp (R, L, m), entonces existe una funcion escalonada
n
X
ak Ik
k=1

tal que n
X
ak Ik f < /2.



k=1 p
2.5 Aproximaciones en Lp () 65

Ahora, sea > 0, para todo 1 k n, escojamos bk Q tal que |bk ak | <
/2 y Jk un intervalo con extremos racionales de modo que Ik Jk con
m(Jk \ Ik ) < /n.
Entonces, en virtud de la desigualdad de Minkowski tenemos

X n X n
ak Ik bk Jk



k=1 k=1 p

X n
= (ak Ik bk Ik + bk Ik bk Jk )


k=1
p
X n X n
= (ak bk )Ik + bk (Ik Jk )


k=1 k=1 p

X n X n
(ak bk )Ik + bk (Ik Jk )


k=1 p k=1 p
n
X n
X
|ak bk | kIk kp + |bk | kIk Jk kp
k=1 k=1
Xn n
X
= |ak bk | (m(Ik ))1/p + |bk | (m(Jk \ Ik ))1/p
k=1 k=1
n  
X 1/p
< (m(Ik )) + + max |ak | 1/p
2 k=1
2 1kn
= 0 ,
dado que |bk | |ak bk | + |ak | 2 + max1kn |ak |. Ahora, note que 0 0
si 0, as, podemos escoger tal que 0 < /2. Entonces, invocando una
vez mas la desigualdad de Minkowski tendremos que
n

X
f bk Jk


k=1 p
n
n n

X X X
f ak Ik + ak Ik bk Jk


k=1 p k=1 k=1 p
< /2 + /2
=
Finalmente, note que S es un conjunto contable en virtud que
[
QQQ= Q Q {q}
qQ
66 Los espacios Lp

es contable. De esta manera hemos demostrado que Lp (R, L, m) con 1 p <


es denso.
Ejercicios
1. Sea = {0 , 1 , . . . , m } una particion finita del intervalo [a, b] y sea
f Lp ([a, b], L, m) (p 1). La funcion T definida por
Z k
1
T f (k ) = f (t) dt
k k1 k1
se llama la -aproximacion de f en media. Demostrar que
kT f kp kf kp .

2. Sea f Lp ([a, b], L, m) demostrar que kT f f kp 0 cuando la


longitud del mas grande subintervalo de tiende a cero.

3. Demostrar que T f f cuando 0.
4. Dada f Lp ([a, b], L, m) 1 p < . Demostrar que dado > 0 existe
una funcion medible fM tal que |fM | M y kf fM kp < .
5. Sea una medida positiva y supongamos que f, g Lp (). Demostrar:
a) Si 0 < p < 1, entonces
Z Z
p p
(|f | |g| ) d |f g|p d
X X

b) Si 1 p < y kf kp M , kgkp M , entonces


Z
||f |p |g|p | d 2pM p1 kf gkp .
X

6. Si 0 < p < q < r , demostrar que


Lp (X, A, ) Lr (X, A, ) Lq (X, A, )
1 1
donde q
= p
+ r
( > 0) y ademas
kf kq kf kp kf k1
r .

7. Si 0 < p < q < r . Demostrar que


Lq (X, A, ) Lp (X, A, ) + Lr (X, A, )
es decir cada f Lq (X, A, ) es la suma de una funcion en Lp (X, A, )
y de una funcion en Lr (X, A, ).
8. Sea 1 p < . Si g Lp (X, A, ) tal que |fn | g c.t.p. n. Si
fn f c.t.p. demostrar que fn f en Lp (X, A, ).
2.6 Funcionales Lineales y Acotados sobre el Espacio Lp () 67

2.6. Funcionales Lineales y Acotados sobre el


Espacio Lp()
Sea X un espacio normado , una funcion F : X R se llamara un
funcional lineal si satisface

F (f + g) = F (f ) + F (g)

y diremos que F es acotado si existe un n


umero real M > 0 tal que

kF (f )k M kf k

para toda f de X. La constante M mas peque na para la cual la desigualdad


anterior es verdad la llamaremos la norma de f . Esto es,

|F (f )|
kF k = sup .
f 6=0 kf k

Sea g una funcion fija en Lq (), mostraremos que F dado por


Z
F (f ) = f g d
X

define un funcional lineal en Lp (). En efecto, sean y n


umeros reales, f
y h elementos de X, entonces
Z
F (f + h) = (f + h)g d
XZ Z
= f g d + hg d
X X
= F (f ) + F (h),

ademas, f es acotada, en efecto


Z Z

|F (f )| = f g d |f g| d kf kp kgkq
X X

de aqu se deduce que


|F (f )|
kgkq
kf kp
lo que significa que
kF k kgkq ,
68 Los espacios Lp

ltimo muestra que F es acotado. Por otra parte, para 1 < p < ,
esto u
definamos
f = |g q1 | sign(g),
entonces
f g = |g|q1 sign(g)g = |g|q .
Por otra parte,
|f | = |g|q1 | sign(g)|,
de donde |f | = |g|q1 , entonces |f |p = |g|p(q1) , as |f |p = |g|q , ahora bien
Z Z
F (f ) = f g d = |g|q d = kgkqq ,
X X

luego
kgkpq
Z
q
|g|q d = kgkqq = kgkp(q1) = ,
X
q
kgkpq
de donde Z
kgkpq |g|q d = kgkpq
q ,
X
por lo tanto Z
kgkpq |f |p d = kgkpq
q ,
X
de aqu se obtiene
kgkqq = kf kp kgkq ,
por lo cual
F (f ) = kf kp kgkq ,
as
|F (f )|
kgkq ,
kf kp
luego existe f = |g|q1 sign(g) para el cual
|F (f )|
kgkq ,
kf kp
en consecuencia, kF k kgkq . Por lo tanto la norma alcanza el supremo y
kF k = kgkq .
Ahora, consideremos los casos p = 1 y p = . Sea g L1 () y f = sign(g),
entonces kf k = 1 y
Z Z Z
f g d = g sign(g) d = |g| d = kgk1 ,
X X X
2.6 Funcionales Lineales y Acotados sobre el Espacio Lp () 69

luego
F (f ) = kf k kgk1
as,
F (f )
= kgk1 ,
kf k
por lo tanto
kF k kgk1 .
La otra desigualdad se obtiene usando la desigualdad de Holder, por lo cual
resulta
kF k = kgk1 .
Ahora, si g L ()], entonces para  > 0 dado se tendra

|g| > (kgk ),

para f = sign(g) tendremos que

f g = g sign(g) = |g|,

luego Z Z Z
f g d = |g| d > (kgk ) d
X X X
esto es, Z
f g d > (kgk )
X
pero Z
kf k1 = | sign(g)| d = 1,
X
entonces Z
f g d > (kgk )kf k1 ,
X
por lo cual
F (f )
> kgk ,
kf k1
por la arbitrariedad de  podemos escribir
F (f )
> kgk
kf k1
pero
|F (f )| F (f )
,
kf k1 kf k1
70 Los espacios Lp

luego
kF k kgk . (2.26)
Por otra parte, |g| kgk c.t.p luego |f g| |f |kgk , entonces
Z Z 
|f g| d |f | d kgk
X X

pero Z Z

f g d |f g| d,

X X

por lo tanto Z 
|F (f )| |f | d kgk ,
X

as
|F (f )|
kgk ,
kf k1
de aqu obtenemos
kF k kgk , (2.27)
en vista de (2.26) y (2.27) resulta

kF k = kgk .

As, hemos demostrado el siguiente Teorema.

Teorema 2.57. Cada funcion g de Lq () define un funcional F lineal y


acotado en Lp [0, 1] dado por
Z
F (f ) = f g d
X

y ademas, kF k = kgkq .

Lema 2.58. Sea g una funcion integrable en [0, 1] y supongamos que existe
una constante M tal que
Z 1


f g d M kf kp
0

para toda funcion f medible y acotada, entonces g Lq [0, 1] y kgkq M .


2.6 Funcionales Lineales y Acotados sobre el Espacio Lp () 71

Demostracion. En primer lugar supongamos que 1 < p < y definamos la


sucesion de funciones medibles y acotada por

g(x) si |g(x)| n;
gn (x) =
0 si |g(x)| > n.

y definamos
fn = |gn |q1 sign(gn ),
de aqu
|fn | = |gn |q1 ,
luego
|fn |p = |gn |p(q1) ,
entonces
kfn kpp = kgn kqq
y ademas, |fn |p = |g|q , por otra parte,

fn gn = gn |gn |q1 sign(gn ) = |gn |q

pero fn gn = fn g, luego
Z 1
kgn kqq = fn g d M kfn kp = M kgn kq/p
q ,
0

de aqu
kgn kqq M kgn kq/p
q ,

entonces
q pq
kgn kq M
pero
qp q q(p 1) p
= = = 1,
p p p
entonces
kgn kq M
y
Z 1
|gn |q d M q
0
puesto que
lm |gn |q = |g|q
n
72 Los espacios Lp

casi en todo [0, 1], entonces por el Lema de Fatou


Z 1 Z 1
q
|g| d lm |gn |q d M q ,
0 n 0

esto significa que g Lq [0, 1] y ademas

kgkq M.

Para el caso p = 1, sea  > 0 y consideremos el conjunto

E = {x : |g(x)| M + }

y la funcion f = sign(g)E , entonces

kf k1 = (E),

de donde
1
Z

M (E) = M kf k1 f g d (M + )(E),
0

de aqu se obtiene que 0 (E) 0, por lo tanto (E) = 0, en consecuencia


kf k1 = 0 y kgk M que era lo que se quera demostrar.

El Lema anterior puede ser extendido a cualquier espacio de medida finita.


En efecto.

Lema 2.59. Sea (X, A, ) un espacio de medida finita. Sea g L1 () tal


que para alg
un M > 0 y para toda funcion simple s se cumple que
Z

sg d M kskq

X

(1 p < ). Entonces g Lq () y kgkq M , donde q es el ndice conjugado


de p.

Demostracion. Caso p = 1. Sea A = {x : g(x) > M } y B = {x : g(x) <


M }. Note que A y B estan en A.

Si escogemos s = A , entonces por hipotesis tenemos


Z

A g d M kA k1 ,

X
2.6 Funcionales Lineales y Acotados sobre el Espacio Lp () 73

es decir Z Z

g d M (A) = M d,

A A

de donde Z
(g M ) d 0.
A

Como g > M en A, concluimos que (A) = 0.

Similarmente, escojamos s = B , luego podemos demostrar que

(B) = 0.

As (A B) = 0, lo que significa que

|g(x)| M c.t.p [].

Entonces kgk M .

De esta forma queda demostrado el Lema para el caso p = 1.

Caso 1 < p < . Dado que |g|q > 0 podemos hallar {sn } una sucesion de
funciones simples no negativas tal que sn |g|q (puntualmente).
1/p
Definamos tn = sn (sign(g)) (n N), observe que cada tn es una fun-
cion simple y
Z 1/p
p
ktn k = |tn | d)
X
Z 1/p
= sn d .
X

Como

gtn = s1/p g sign(g)


= s1/p
n |g|

s1/p
n sn
1/q

= sn ,

entonces Z Z
0 sn d gtn d M ktn kp .
X X
74 Los espacios Lp

R R 1/p
X
sn d M X
sn d
R 1/q
X
sn d M
R 
X
s n d M q.

Por el teorema de la convergencia monotona concluimos que


Z
|g|q d M q ,
X

de donde g Lq () y kgkq M

Teorema 2.60 (Representacion de Riesz). Sea F un funcional lineal y aco-


tado en Lp [0, 1], 1 p < , entonces existe una funcion g Lq [0, 1] tal
que
Z 1
F (f ) = f g d
0

para cualquier f Lp [0, 1] y ademas,

kF k = kgkq .

Demostracion. Sea S la funcion caracterstica del intervalo [0, s]. Definamos

: [0, 1] R

tal que (s) = F (S ). vamos a demostrar que es absolutamente continua.


Sea {Sk , Sbk }nk=1 cualquier coleccion de subintervalos de [0, 1] disjuntos tales
que
Xn
|Sbk Sk | < ,
k=1

entonces si
 
k = sign (Sbk ) (Sk ))
2.6 Funcionales Lineales y Acotados sobre el Espacio Lp () 75

tenemos que

n
X n 
X   
|(Sbk ) (Sk )| = (Sbk ) (Sk ) sign (Sbk ) (Sk )
k=1 k=1
n 
X 
= F (Sbk ) F (Sk ) k
k=1
n
X
= F (k (Sbk Sk ))
k=1
n
!
X
=F k (Sbk Sk ) ,
k=1

luego
n
X
(Sk ) (Sk ) = F (f ),
b
k=1

de donde
n
X
f= k (Sbk Sk )
k=1

por otra parte consideremos


Z 1
kf kpp = |f |p d
0
Z 1 X  p

n 
= k Sbk Sk d


0 k=1
 p
Z 1 X n
!

|k Sbk Sk | d
0 k=1
n
!p
Z 1 X
= Sbk Sk d

0 k=1
n
!p
Z 1 X
= [Sk ,Sbk ] d
0 k=1
Z 1  p
= Sn d
k=1 [Sk ,Sk ]
b
0
76 Los espacios Lp

pues [Sk , Sbk ] son disjuntos, continuando


Z 1  p Z 1
Sn d = Sn d
k=1 [Sk ,Sk ] k=1 [Sk ,Sk ]
b b
0 0
n
!
[
= [Sk , Sbk ]
k=1
n
X
= ([Sk , Sbk ])
k=1
n
X
= Sk Sk
b
k=1
<

por lo cual
kf kpp < .
Ahora bien,
n
X
|(Sbk ) (Sk )| = F (f ) kF kkf kp < kF k 1/p ,
k=1

si
p
= ,
kf kp
entonces n
X
|(Sbk ) (Sk )| < 
k=1

siempre que kf kpp < , lo que muestra que es absolutamente continua


en [0, 1]. Como toda funcion absolutamente continua es integrable, entonces
existe g [0, 1] tal que Z s
(s) = g(t) d,
0

de aqu se tiene que Z 1


F (S ) = g(t)S (t) d.
0

Por otra parte, como toda funcion escalonada de [0, 1] se puede escribir
como n
X
= ck S ,
k=1
2.6 Funcionales Lineales y Acotados sobre el Espacio Lp () 77

entonces tenemos en particular


Z 1
F (Sk ) = gSk d
0
Z 1
ck F (Sk ) = ck gSk d
0
Z 1
F (ck Sk ) = gck Sk d
0
n
X n Z
X 1
F (ck Sk ) = gck Sk d
k=1 k=1 0
n
! n
X Z 1 X
F ck Sk = g ck Sk d
k=1 0 k=1
Z 1
F () = g d.
0

Ahora, consideremos una funcion f medible y acotada en [0, 1], entonces por
un teorema conocido de teora de la medida existe una sucesion {n }nN de
funciones escalonadas tal que f c.t.p, de donde resulta que la sucesion
{| f |p }nN es uniformemente acotada y tiende a cero en casi todo [0, 1],
entonces por el teorema de la convergencia acotada

lm kn f kp = 0
n

y como f es acotada, entonces

|F (f ) F (n )| = |F (f n )| kF kkf n kp ,

entonces
lm F (n ) = F (f ).
n

Por otra parte, existe M > 0 tal que

|n | M

por ser {n }nN convergente, de donde

gM gn gM,

entonces
|gn | gM M |g|,
78 Los espacios Lp

por lo cual
Z 1 Z 1
lm gn d = f g d
n 0 0
Z 1
lm F (n ) = f g d
n 0
Z 1
F (f ) = f g d
0

para cada funcion f medible acotada, puesto que


|F (f )| kF kkf kp
es decir,
1
Z


f g d kF kkf kp ,
0
entonces por el Lema 2.58 g Lq [0, 1] y
kgkq kF k.
Ahora tenemos solamente que mostrar que
Z 1
F (f ) = f g d
0

para cada f Lp [0, 1]. Sea f una funcion arbitraria de Lp [0, 1]. En virtud
del Teorema 2.55 para cada  > 0 existe una funcion escalonada tal que
kf kp < .
Puesto que es acotada, tenemos entonces que
Z 1
F () = g d,
0

luego
1 Z 1
Z

F (f ) f g d = F (f ) F () + F ()
f g d

0 0
Z 1

|F (f ) F ()| + F () f g d
0
Z 1

= |F (f )| + ( f )g d
0
kF kkf kp + kgkq kf kp
< (kF k + kgkq ) ,
2.6 Funcionales Lineales y Acotados sobre el Espacio Lp () 79

por la arbitrariedad de  resulta


Z 1
F (f ) = f g d.
0

La igualdad kF k = kgkq se sigue del Teorema 2.57.

El Teorema 2.60 puede ser extendido a un espacio de medida -finita. En


efecto
Teorema 2.61 (Representacion de Riesz). Sea (X, A, ) un espacio -finito
y T un funcional lineal en Lp () (1 p < ). Entonces existe un u nico
g Lq () tal que Z
T (f ) = f g d (2.28)
X
para toda f Lp () y ademas

kT k = kgkq . (2.29)

Como siempre q es el ndice conjugado de p.

Demostracion. En primer lugar demostremos la unicidad de g. Supongamos


que existen g1 y g2 en Lq () tal que satisfacen (2.28), dado que es -finita,
podemos hallar una sucesion de conjuntos {Xn }nN en A disjuntos tal que

[
(Xn ) < n y X = Xn ,
n=1

es decir Z Z
g1 d = g2 d
E E
para todo E A con (E) < . Luego
Z Z
g1 d = g2 d,
Xn A Xn A

as Z
(g1 g2 ) d = 0,
Xn A

pero g1 > g2 en Xn A, lo que significa que

(Xn A) = 0 n N.
80 Los espacios Lp

Entonces

X
(A) = (A Xn ) = 0.
n=1

Similarmente (B) = 0, por lo tanto

g1 = g2 c.t.p [].

Esto demuestra la unicidad.

A demostrar la existencia de g.

Caso 1. (X) < , luego para cada E A definamos (E) = T (E ).

Note que (X) < implica que (E) < . As

E Lp ().

A demostrar que es una medida con signo en A. Claramente es la


funcion cero en Lp (), entonces

() = T ( ) = 0.

Note que T es una funcion real, entonces tambien es una funcion


real. En este mismo orden de ideas, escojamos {En }nN una sucesion
de conjuntos en A disjuntos. Ahora, definamos

[ n
[
E= En y An = Ei ,
n=1 i=1

entonces

[
An = E,
n=1

observese que {An }nN es una sucesion creciente, por induccion es facil
demostrar que
Xn
An = Ek .
k=1

De la linealidad de T , se tiene que


n
X
T (An ) = T (Ek ),
k=1
2.6 Funcionales Lineales y Acotados sobre el Espacio Lp () 81

es decir
n
X
T (An ) = (Ek ).
k=1

A demostrar que An E en Lp (). Para ello consideremos


Z
p
kAn E kp = |An E |p d
ZX
= E\An d
X
= (E \ An )
= (E) (An ).

S que {An }nN es una sucesion de conjuntos crecientes tal que E =


Dado
n=1 An , entonces
(E) = lm (An ),
n

es decir
lm [(E) (An )] = 0,
n

as
lm kAn E kpp = 0,
n

luego
lm kAn E kp = 0.
n

En virtud de la continuidad de T en Lq (), se sigue que

lm T (An ) = T (E ).
n

Entonces

(E) = T (E )
= lm T (An )
n
n
X
= lm (Ek ),
n
k=1

esto u
ltimo nos dice que es una medida con signo. Ahora queremos
demostrar que
<< .
82 Los espacios Lp

Supongamos que E A con (E) = 0, entonces


Z 1/p
kE kp = E d
X
= [(E)]1/p
= 0.
Esto nos dice que E es la funcion cero en Lp (), as T (E ) = 0,
as T (E ) = 0, es decir (E) = 0 por lo tanto v << . En virtud del
Teorema de Radon-Nikodym para medidas (con signo) finitas, existe
una funcion medible g tal que
Z
(E) = g d
E

E A. Luego
Z
g d = (X)
X
= T (X )
= T (1)
< ,
as g L1 ().

Verifiquemos que g satisface las hipotesis del Lema 2.59, sea s Lp ()


una funcion simple A-medible con representacion canonica
n
X
s= k Ek ,
k=1

entonces
n
!
X
T (s) = T k Ek
k=1
n
X
= k (Ek )
k=1
n
X Z
= k g d
k=1 Ek
Z n
!
X
= g k Ek d
X k=1
2.6 Funcionales Lineales y Acotados sobre el Espacio Lp () 83

Z
= gs d.
X

por lo cual Z
T (s) = sg d
X
para toda funcion simple s Lp (), de aqu se sigue que
Z

sg d = |T (s)|

X
kT kkskp .

Si M = kT k entonces 0 < M < , esto nos dice que g satisface las


condiciones del Lema 2.59, por lo tanto podemos concluir que g Lq ()
y
kgkq M = kT k. (2.30)
En lo que sigue demostraremos que
Z
T (f ) = f g d
X

para toda f Lp ().

Sea f Lp () y  > 0 arbitrario, en virtud del Lema 2.22 existe


una funcion simple s Lp () tal que

kf skp < .
kgkq + kT k + 1
Entonces
Z Z

T (f ) f g d = T (f ) T (s) + T (s)
f g d

X X
Z Z

|T (f s)| + sg d
f g d
ZX X

|T (f s)| + |s f ||g| d
X
kT kkf skp + ks f kp kgkq
= (kT k + kgkq ) kf skp
(kT k + kgkq ) 
<
kT k + kgkq + 1
< .
84 Los espacios Lp

Dado que  es arbitrario, podemos concluir que


Z
T (f ) = f g d
X

para todo f Lp ().

Finalmente por la desigualdad de Holder (Teorema 2.22) tenemos

|T (f )| kgkq kf kp

de donde
kT k kgkq , (2.31)
de (2.30) y (2.31) resulta

kT k = kgkq ,

quedando as demostrado el caso 1.

Caso 2. (X) = .

[
En virtud de la -finitud de , existe {Xn }nN tal que X = Xn
n=1
con Xn Xn+1 y (Xn ) < n. Aplicaremos el caso 1 al espacio de
medida (Xn , A Xn , n ) donde n = |AXn .

Sea Tn = T |Lp (n ) , por el caso 1 para todo n N existe gn Lq (n ) tal


que Z
Tn (h) = hgn dn (2.32)
Xn

para todo h Lp () que se anula fuera de Xn , ademas

kgn kq = kTn k kT k. (2.33)

Definamos
gn (x) si x Xn ;
gn (x) =
0 si x
/ Xn .

Entonces podemos escribir (2.32) como


Z
T (h) = hgn d (2.34)
X
2.6 Funcionales Lineales y Acotados sobre el Espacio Lp () 85

para toda h Lp () el cual se anula fuera de Xn .

Ahora, dado que gn+1 restringido a Xn tiene las mismas propiedades


que gn en virtud de la unicidad se tiene que gn = gn+1 en Xn . Bien,
ahora definamos
g(x) = gn (x) si x Xn .
Como
|gn (x)| |gn+1 (x)| x X
y
lm gn (x) = g(x),
n

entonces por el Teorema de la convergencia monotona


Z Z
q
|g| d = lm |gn |q d
X n X
kT kq ,

entonces
g Lq () y kgkq kT k. (2.35)
Sea f Lp () y fn = f Xn , note que fn se anula fuera de Xn y fn f
(puntualmente) en X. Claramente

|fn f | |f |,

as
|fn f |p |f |p ,
por el Teorema de la convergencia dominada tenemos que
Z
lm |fn f |p d = 0.
n X

De la continuidad de T se sigue que

T (fn ) T (f ) (n ).

Por otra parte , note que

|fn g| |f g|; f g L1 () y lm fn g = f g,
n
86 Los espacios Lp

ahora invocamos el teorema de la convergencia dominada para obtener


Z Z
f g d = lm fn g d
X n X
Z
= lm f Xn g d
n X
Z
= lm (f Xn ) (gXn ) d
n X
Z
= lm fn gn d
n X
= lm T (fn )
n
= T (f ).

As, hemos demostrado (2.28), una vez mas por Holder se tiene que

|T (f )| kf kp kgkq ,

es decir
kT k kgkq ,
por (2.35)
kgkq kT k.
Por lo tanto kT k = kgkq , que era lo que queramos demostrar.

2.7. Espacios duales


En espacios vectoriales X de dimension finita es conocido el hecho de que
si {e1 , . . . , en } es una base de X, entonces el espacio dual X 0 , denominado
dual algebraico de X definido por

X 0 = {f : X R | f es lineal}

tiene dimension n y el conjunto {f1 , . . . , fn }, donde fi (ek ) = ik , es una


base de X 0 . Un resultado analogo puede demostrarse en dimension infinita,
utilizando bases de Schauder.
Este hecho es el punto de partida para definir el concepto de espacio dual
en espacios normados arbitrarios.
2.7 Espacios duales 87

on 2.62. Sea (X, +, , k k) un espacio normado, llamamos espacio


Definici
dual de X a
X = {f : X R | f es lineal y acotado.}
Se dim X < , este concepto coincide con el dual algebraico.

Observaci on 2.63. Los teoremas 2.60 y 2.61 nos demuestran que el espacio
dual de Lp ()(1 p < ) es Lq (). Es decir [Lp ()] = Lq ().
1 1
Teorema 2.64. El espacio dual de lp es lq si p
+ q
= 1, (1 < p < ).

Demostracion. Una P base de Schauder de lp es ek = (kj )j=1 . Si f (lp ) ,
entonces f (x) = kNk f (ek ) . Una vez mas, definamos T (f ) = (f (ek ))kN .
Queremos demostrar que la imagen de T esta en lq . Para ello, se define la
(n)
sucesion (n) = (k )k=1 para cada n con
(
|f (ek )|q
(n) f (ek )
si k n y f (ek ) 6= 0,
k =
0 si k > n o f (ek ) = 0.

Entonces
X n
X
n (n)
f (x ) = k f (ek ) = |f (ek )|q .
kN k=1

Como ademas

f (xn ) kf kkxn kp
n
! p1
X (n)
= kf k |k |p
k=1
n
! p1
X
= kf k |f (ek )|qpp
k=1
n
! p1
X
= kf k |f (ek )|q ,
k=1

resulta que

n
!1 p1 n
! 1q
X X
|f (ek )|q = |f (ek )|q
k=1 k=1
kf k.
88 Los espacios Lp

Haciendo n , obtenemos
n
! p1
X
|f (ek )|q kf k
k=1

donde (f (ek ))kN lq .


Ahora, afirmamos que: i) T es sobreyectiva, en efecto dado b = (k )kN
lPq , podemos asociarle un funcional lineal y acotado g lp , mediante g(X) =

k=1 k k con x = (k )kN lp (la acotaci on se deduce de la desigualdad
de Holder). Entonces g (lp )0 .
No es difcil ver que T es inyectiva. Por u ltimo veamos que la norma de
f es la norma en lq de T f

X
|f (x)| = k f (ek )


kN
! p1 ! 1q
X X
|k |p |f (ek )|q
kN kN
! 1q
X
= kxk |f (ek )|q .
kN

Tomando el supremo sobre los x de norma 1, se tiene que


! 1q
X
kf k |f (ek )|q .
kN

Como la otra desigualdad tambien es cierta, se deduce la igualdad


! 1q
X
kf k = |f (ek )|q ,
kN

con lo que se establece el isomorfismo f (f (ek ))kN deseado.


Teorema 2.65. El espacio dual de l1 es l .
P
Demostracion. Para todo x l1 , podemos escribir x = k=1 k ek , donde
ek = (kj )
j=1 forma una base de Schauder de l1 , dado que

n
X
x k ek = (0, . . . , 0, n+1 , . . .)
| {z }
k=1 n
2.8 Convergencia debil en Lp 89

y
X n X
x k ek = k ek 0


k=1 k=n+1

en virtud que la serie


P
k=1 k ek es convergente.

P Definamos la aplicacion T (f ) = (f (ek ))kN , f (l1 ) . Como f (x) =
kN k f (ek ), entonces |f (ek ) kf kkek k, pues kek k = 1. En consecuencia,
supkN |f (ek )| kf k, donde (f (ek ))kN l .
Afirmamos que:// i) T es sobreyectiva,
P en efecto b = (k )kN l ,
definamos q : l1 E como g(x) = kN k k si x = (k )kN l .
El funcional g es lineal y acotado dado que
X X
|g(x)| |k k | sup |k | |k | = kxk1 sup |k |,
kN kN
kN kN

entonces g (l1 ) . Ademas, como g(ek ) =


P
jN kj j ,

T (g) = (g(ek ))kN = (k )kN = b.

ii) T es inyectiva; si T f1 = T f2 , entonces


P P
f1 (ek ) = f2 (ek ), k. Como f1 (x) = kN xk f1 (ek ) y f2 (x) = kN xk f2 (ek ),
entonces f1 = f2 .
iii) T es una isometra (ver definicion 9.1) En efecto,

kf k = sup |f (ek )| kf k (2.36)


kN

y
X X
|f (x)| = k f (ek ) sup |f (ek )| |k | = kxk sup |f (ek )|

jN kN
kN kN

As,
kf k sup |f (ek )| = kT f k (2.37)
kN

Combinando (2.36) y 2.37 resulta kT f k = kf k. Demostrandose as que los


espacios (l1 ) y l son isometricos.

2.8. Convergencia d
ebil en Lp
Consideremos la funcion x 7 cos(xn) para n = 1, 2, . . .. Note que
Z 2
cos2 (nx)dx = , para todo n N.
0
90 Los espacios Lp

Por lo tanto la sucesion {cos nx}nN de funciones en L2 ([0, 2], L, m) no


converge a cero en la norma de L2 ([0, 2], L, m). Sin embargo tal sucesion
converge a cero en el sentido siguiente. Sea g = [a,b] donde [a, b] [0, 2].
Un calculo directo nos muestra que
Z 2
1
[a,b] cos(nx)dx = [sin(nb) sin(na)] 0
0 n
cuando n . Ahora consideremos {(aj , bj )}m j=1 una colecci
on finita de
subintervalos disjuntos de [0, 2] y la funcion simple de la forma
m
X
= j [aj ,bj ] .
j=1

Observe que Z 2
lm (x) cos(nx)dx = 0.
m 0
Los ejemplos anteriores motivan la siguiente definicion.
Definicion 2.66. Sea (X, A, ) un espacio de medida y 1 p, q tal que
1/p + 1/q = 1. Una sucesion de funciones {fn }nN en Lp () (1 p ) se
dice que converge debilmente a f Lp () si
Z Z
lm fn gd = f gd
n X X

para todo g Lq ().


Teorema 2.67. Sea (X, A, ) un espacio de medida finita y 1 < p < .
Sea {fn }nN una sucesion de funciones en Lp () tal que fn f c.t.p [].
Entonces
lm (fn ) = (f )
n

para todo Lp () si y solo si {kfn kp }nN es acotada.

Demostracion. (). Supongamos que lmn (fn ) = (f ) para todo


Lp (). Definamos
: Lp Lp ()
por
(f ) = (f ),
observe que
k(f )k = kf kp .
2.8 Convergencia debil en Lp 91

Por hipotesis
sup |(fn )| < .
nN

En virtud del principio de acotacion uniforme se tiene que

sup |(fn )()| <


nN

implica que
sup k(fn )k < ,
nN

as
sup kfn kp < .
nN

(). Ahora, si kfn kp M , entonces por el Lema de Fatou se sigue


Z Z
p
|f | d = lm inf |fn |p d
X X n Z
lm inf |fn |p d
n X
M.

Por otra parte, por el Teorema 2.60 (Representacion de Riesz) para


todo Lp () existe g Lq () tal que
Z
(h) = hg d
X

para todo h Lp (). Dado que g Lq (), entonces |g|q L1 (), luego
existe > 0 para todo E A tal que (E) < implica que
Z   q
q
|g| d < ,
E 4M

como (X) < podemos invocar el Teorema de Egoroff para garanti-


zar que existen n0 N y A A tal que (A) < y

|fn (x) f (x)| <
2(kgk1 + 1)

para todo x Ac , siempre que n n0 .


92 Los espacios Lp

Finalmente, en virtud de la desigualdad de Holder se tiene que


Z

|(fn ) (f )| = (fn f )g d
ZX Z

= (fn f )g d + (fn f )g d
A Ac
Z 1/q Z
q
kfn f kp |g| d + |fn f ||g| d
A Ac
 kgk1
2M +
4M 2(kgk1 + 1)
=

si n n0 .

Corolario 2.68. Sea {fn }nN una sucesion de funciones en Lp 1 < p <
la cual converge casi en todas las partes a una funcion f Lp y ademas su-
pongamos que existe una constante M tal que kfn kp M , n N. Entonces
para cada g Lq se tiene que
Z Z
lm fn gd = f gd.
n

Observacion 2.69. El Corolario 2.68 para p = 1 no es cierto, en efecto.


Sea X = [0, 1] y n N. Consideremos

n si x Q [0, 1];
g(x) =
0 si x I [0, 1].

Sea M 1, luego

kgk = nf{M : ({x [0, 1] : |g(x)| > M }) = 0} = 1.

As g L [0, 1]. Por otra parte sea fn = n(0,1/n) , note que fn 0 (n )


R1
puntualmente y kfn k1 = 0 fn dm = 1. Pero
Z 1 Z
fn gdm = fn ndm = n.
0 Q[0,1]

Por lo tanto Z 1
lm fn gdm = .
n 0
2.8 Convergencia debil en Lp 93

Sin embargo el Corolario 2.68 es cierto para el caso p = . Sea {fn }nN
una sucesion de funciones en L , entonces kfn k < , luego |fn | kfn k
c.t.p. Dado que fn f puntualmente, obtenemos |f | kf nk c.t.p. Sea
g L1 , entonces |f g| kfn k |g| L1 y |fn g| kfn k |g| L1 . En virtud
del teorema de la convergencia dominada se tiene
Z Z
lm fn gd = f gd.
n

1 1
Teorema 2.70. Sea 1 < p < y + = 1. Supongamos que {fn }nN
p q
Lp (R) con 1 = sup kfn kp < . Entonces existe f Lp (R) tal que kf kp M
n
y una subsucesion {fnk }kN {fn }nN tal que
Z Z
lm fnk g dm = f g dm,
n R R

con g Lq (R).

Demostracion. Sea g una funcion perteneciente a la familia {gn }nN Lq (R).


Queremos demostrar que Z
lm fnk g dm
k0 R
existe. Para ello definamos
Z
Ck,n = fk gn dm.
R

En virtud de la desigualdad de Holder, resulta


Z

|Ck,1 | = fk g1 dm
R
kfk kp kg1 kq ,

de lo anterior y ya que fk Lp (R) se sigue que

|Ck,1 | M kg1 kq . (2.38)

Dado que {Ck,1 }kN es una sucesion de numeros reales y ademas, por (2.38)
es acotada, entonces podemos invocar el Teorema de Bolzano-Weierstrass
para obtener una subsucesion {Ck1 ,1 } {Ck,1 } tal que

lm Ck1 ,1
k1
94 Los espacios Lp

existe. Podemos repetir este argumento con {Ck1 ,1 } para obtener una nueva
subsucesion {Ck2 ,h } con h = 1, 2 tal que

lm Ck2 ,h
k2

existe. Ahora, por el proceso de diagonalizacion de Cantor obtenemos una


subsucesion {Ckm ,h } de modo que

lm Ckm ,h
km

existe.

Por otra parte, podemos escoger de {gn }nN una familia densa en Lq (R),
denotemos esta por G y definamos
Z
T (g) = lm fkm g dm
m R

para todo g G. Note que

T (gn1 + gn2 ) = T (gn1 ) + T (gn2 )

para todo gn1 , gn2 G. Una vez mas por la desigualdad de Holder se tiene

|T (g)| kfkm kp kgkq


M kgkq ,

de esta manera, hemos demostrado que T es un funcional lineal y acotado,


es decir T es continuo. En lo que sigue, queremos extender T a todo Lq (R),
en efecto para cada g Lq (R) existe una sucesion {gn }nN en G tal que

lm kg gn kq = 0
t

y
lm kgn kq = kgkq .
n

Observe que
|T (gn ) T (gL )| M kgn gL kq 0
cuando n , esto nos dice que {T (gn )}nN es una sucesion de Cauchy en
R, por lo tanto converge en R, digamos que su limite es T (g), es decir

T (g) = lm T (gn ),
n
2.8 Convergencia debil en Lp 95

lo cual esta bien definido en Lq (R). Luego, note que

|T (g)| lm sup |T (g) T (gn )| + lm sup |T (gn )|


n n
lm sup |T (gn )|
n
M lm sup kgn kq
n
= M kgkq

para toda g Lq (R). En virtud del Teorema de Representacion de Riesz


existe f Lp (R) tal que
kT k = kf kp ,
pero
kT k M,
as
kf kp M
y Z
T (g) = f g dm,
R
por lo tanto Z Z
lm fkm g dm = f g dm.
n R R

Teorema 2.71. Sea (X, A, ) un espacio de medida y f una funcion A-


medible. Consideremos : [0, ) [0, ) una funcion de clase C 1 tal que
(0) = 0. Entonces,
Z Z
(|f |) d = 0 ()({x X : |f (x)| > }) d
X 0

para > 0.

Demostracion. Si (({x X : |f (x)| > })) = no hay nada que demos-


trar, en tal sentido supongamos que (({x X : |f (x)| > })) < .

Queremos demostrar que ({x X : |f (x)| > }) es medible sobre [0, ).


Para ello consideremos el conjunto

E = {(x, ) X [0, ) : 0 < < |f (x)|}.


96 Los espacios Lp

A demostrar que E es medible en X [0, ). En efecto, como |f | 0,


entonces, existe una sucesion {sn }nN de funciones simples tal que sn f ,
podemos escribir

X
sn = anj Anj ,
j=1
con Anj A y j = 1, 2, . . . , n N. Luego
En = {(x, ) X [0, ) : 0 < < sn (x)}
[
= Anj (0, anj )
j=1

es medible en X [0, ). Ahora, observe que


lm En (x) = E (x),
n

dado que En es una funcion medible en X [0, ) ya que En es medible en


X [0, ), por lo tanto E es medible por ser el lmite de una sucesion de
funciones medibles. En consecuencia,
E = {(x, ) X [0, ) : 0 < < |f (x)|}
es medible, de este hecho se desprende que
E = {x X : |f (x)| > }
es medible en [0, ).

Por otra parte, como es de clase C 1 , entonces


(x, ) 0 ()E (x)
es medible, ademas, en virtud que (0) = 0 resulta
Z
(t) = 0 () d.
0
Luego
Z Z Z |f |
(|f |) d = 0 () dd
X
ZX Z0
= [0,|f |] ()() dd
ZX Z
0

= {xX: |f (x)|>} (x)0 () dd


Z0 X

= 0 ()({x X : |f (x)| > }) d.


0
2.8 Convergencia debil en Lp 97

Corolario 2.72. Si f Lp () con 0 < p < . Entonces,


Z Z
p
|f | d = p p1 ({x X : |f (x)| > }) d.
X 0

Teorema 2.73. Sea f Lp (); 1 p < . Entonces

lm tp ({x X : |f (x)| > t}) = lm tp ({x X : |f (x)| > t})


t0 t

Demostracion. Sea f Lp (), 1 p < , entonces en virtud de la desigual-


dad de Markov (Lema 2.28) con g(t) = tp , resulta
Z
1
({x X : |f (x)| > t}) p |f |p d < ,
t X

luego
lm tp ({x X : |f (x)| > t}) = 0.
t0

Por otra parte, definamos



|f |, si |f | > t;

ft = (2.39)

0, si |f | t,

como f Lp (), entonces |f | < c.t.p[]. En efecto, note que



\
{x : |f (x)| = } = {x : |f (x)| > n}.
n=1

As, Z
1
({x : |f (x)| > n}) p |f |p d,
n X

de donde
lm ({x : |f (x)| > n}) = 0,
n

entonces

({x X : |f (x)| = }) lm ({x X : |f (x)| > n}) = 0.


n

Por lo tanto
|f | < c.t.p[].
98 Los espacios Lp

Ahora, regresemos a (d), observe que lmt ft = 0 c.t.p[] implica que


lmt ftp = 0 c.t.p[], por el teorema de la convergencia dominada se tiene
Z
lm ftp d = 0.
t X

Entonces, Z Z
lm p
|f | d = lm ftp d = 0,
t {x:|f (x)|>t} t X

finalmente
Z
p
lm t ({x X : |f (x)| > t}) lm |f |p d = 0.
t t {x:|f (x)|>t}

Si f es una funcion medible en (X, A, ) entonces podemos definir su


funcion distribucion
Df : [0, ) [0, )
por
Df () = ({x X : |f (x)| > }).
Supongamos que Df es una funcion decreciente y continua por la derecha,
en efecto, sean 0 1 2 arbitrarias, entonces
{x X : |f (x)| > 2 } {x X : |f (x)| > 1 },
por la monotona de la medida, se tiene que
Df (2 ) Df (1 ),
esto u
ltimo nos dice que Df es decreciente. Para demostrar que Df es continua
por la derecha, consideremos 0 0 y escojamos 1 2 3 , luego
definamos
Ef () = {x X : |f (x)| > }.
Observe que
Ef (1 ) Ef (2 )
y
    
1 1
lm Df 0 + = lm Ef 0 +
n n n n
 !
[ 1
= Ef 0 +
n=1
n
= (Ef (0 )) ,
2.8 Convergencia debil en Lp 99

estableciendose as la continuidad por la derecha.

De este hecho podemos definir una medida de Borel en (0, ) dada por

((a, b]) = Df (b) Df (a)

para todo a, b > 0. En tal sentido podemos considerar la integral de Lebesgue-


Stieltjes Z Z
dDf = d,
0 0

donde es una funcion no negativa y Borel medible en (0, ).

El siguiente resultado nos muestra que la integral sobre X de la funcion


|f | se puede expresar como una integral de Lebesgue-Stieltjes.

Teorema 2.74. Si Df () < para todo > 0 y es una funcion no


negativa, Borel medible en (0, ). Entonces
Z Z
|f | d = () dDf ().
X 0

Demostracion. Sea la medida de Borel en (0, ) dada por

((a, b]) = Df (b) Df (a) (2.40)

para todo a, b > 0. Afirmamos que

Df (a) Df (b) = ({x : a < |f (x)| b}).

En efecto, dado que b > a, entonces

{x X : |f (x)| > b} {x X : |f (x)| > a}.

Como Df (a) < se tiene

({x X : |f (x)| > a}) < ,

luego

{x X : a < |f (x)| b} = {x X : |f (x)| > a} {x X : |f (x)| b}


= {x X : |f (x)| > a} {x X : |f (x)| > b}c
= {x X : |f (x)| > a} \ {x X : |f (x)| > b}.
100 Los espacios Lp

As, resulta
({x X : a < |f (x)| b}) =
({x X : |f (x)| > a}) ({x X : |f (x)| > b}) =
Df (a) Df (b).
En virtud de (2.40) podemos escribir
((a, b]) = [Df (a) Df (b)]
= ({x X : a < |f (x)| b})
= (|f |1 (a, b]).
En vista del teorema de extension u
nica de la medida, se sigue que
(E) = (|f |1 (E))
para todo conjunto de Borel E [0, ).

Ahora, consideremos
a) = E y observemos que
|f |(x) = E |f |(x)

1, si |f (x)| E;

=

0, si |f (x)| / E,


1
1, si x |f | (E);

=

0, si x / |f |1 (E),

= |f |1 (E) (E).
As,
Z Z
|f | d = |f |1 (E) (x) d
X X
= (|f |1 (E))
= (E)
Z
= E d
0
Z
= d
0
Z
= dDf .
0
2.8 Convergencia debil en Lp 101

Pn
b) Si = k=1 ak Ek , entonces
Z n
X Z
|f | d = ak |f |1 (Ek ) (x) d
X k=1 X
n
X
= ak (|f |1 (Ek ))
k=1
n
X
= ak (Ek )
k=1
Xn Z
= ak Ek d
0
Zk=1
= d
0
Z
= dDf .
0

c) Si es cualquier funcion no negativa Borel medible, entonces existe una


sucesion {sn }nN de funciones simples tal que sn , entonces sn |f |
|f |, luego en virtud de b)
Z Z
sn |f | d = sn D f ,
X 0

por el teorema de la convergencia monotona se tiene


Z Z
lm sn |f | d = lm sn dDf ,
n X n X
Z Z
|f | d = dDf .
X 0

Ejercicios

1. Sea (X, A, ) un espacio de medida positiva. Sea f una funcion A-


medible. Demuestre que

kf k lm inf kf kp .
p
102 Los espacios Lp

2. Sea 1 p < q < . Supongamos que para g Lq (X, A, ) se cumple


que Z
f g d = 0 f Lp (X, A, ).
X

Demostrar que g = 0 c.t.p. en X.

3. Sea (X, A, ) un espacio de medida y f L1 (). Definamos


Z
Lf (h) = f h d
X

para h L (). Demostrar que Lf es un operador lineal y acotado en


L () y ademas
kLf k = kf k1 .

4. Sea (X, A, ) un espacio de medida -finito. Dada g L () y > 0,


demostrar que existe f L1 () tal que kf k1 = 1 y
Z
kgk f g d > kgk .
X

5. Sea (X, A, ) un espacio de medida con (X) = 1. Sea 1 p < .


Supongamos:

1) S es un subespacio cerrado de Lp ().


2) S L ().

Demostrar que S es de dimension finita.

6. Sea X = C[0, 1] el espacio de Banach de las funciones continuas en


[0, 1] dotado con la norma del supremo. Sea S un subespacio de X el
cual es cerrado como subespacio de L2 ([0, 1], L, m). Demostrar que

i) S es un subespacio cerrado de X.
ii) Existe una constante M tal que

kf k M kf k2 f S.

iii) Para todo y [0, 1] existe ky L2 ([0, 1], L, m) tal que


Z 1
f (y) = ky (x)f (x) dx f S.
0
2.9 Continuidad de la traslacion en Lp () para 1 p < . 103

7. Demuestre que f Lp ([0, ), L, m) (p > 1) si y solo si


  X
X 1 1
p+1
m |f | > + np1 m ({|f | > n}) < .
n=1
n n n=1

1/p
8. Sea kxkp = ( nk=1 |xk |p ) 1 p < y lpn = {x : kxkp < }.
P
Blpn (0, 1) = {x : kxkp 1} denota la bola unitaria en lpn . Denotemos
vn (p) = vol(Blpn ) = vol{x lpn : kxkp 1}.
Observese que Rn = lpn . Calcular vn (p). Ayuda: Calcule la integral
Z
p
Ip = ekxkp dx
Rn

9. Sea (X, A, ) un espacio de medida y f una funcion A-medible, demos-


trar que
Z Z
sen |f (x)| d(x) = cos ({x X : |f (x)| > }) d.
X 0

10. RPara g Lq (), sea F un funcional lineal en Lp () definido por F (f ) =


f gd, F Lp (). Sin usar el teorema de representacion de Riesz,
demuestre que kF k = kgkq .
11. Sea (X, A, ) un espacio de medida finita. Demostrar que el espacio
dual de L1 () es L ().
12. Sea (X, A, ) un espacio de medida y f : X R una funcion medible.
Si ({x X | |f (x)| > }) e para todo 0, entonces demostrar
que f Lp () para cada 1 p < .
13. Demostrar que l no es el espacio dual de l1 .

2.9. Continuidad de la traslaci


on en Lp() pa-
ra 1 p < .
Sea Rn un conjunto medible. Para f Lp () y h Rn , definamos
la traslacion de f como
(
f (x + h) si x + h
Th f (x) =
0 si x + h Rn r .
La siguiente proposicion nos dice que la operacion traslacion es continua en
la topologa generada por la norma Lp () para p [1, +).
104 Los espacios Lp

on 2.75. Sea Rn un conjunto medible y sea f Lp () para


Proposici
1 p < . Para cualquier > 0 existe = () tal que

sup kTh f f kp .
|h|

Demostracion. En primer lugar supongamos que es un subconjunto de


Rn acotado, es decir que esta contenido en una bola BR (0) centrada en el
origen y de radio R, para algun R > 0 suficientemente grande. Sin perdida
de generalidad podemos suponer que = BR (0) definiendo f = 0 en Rn r .
Para E y R definamos

E = {x Rn : x + E}.

Para > 0 (dado) existe > 0 tal que para todo E medible con
m(E) < entonces
p
Z
|f |p dm < p+1 .
E 2
En virtud de que la medida de Lebesgue es invariante por traslacion se tiene
que
m ((E ) ) < .
Por lo tanto
p
Z Z Z 
p p1 p p
|Th f f | dm 2 |f | dm + |f | dm .
E E (E) 2

Dado que f es medible, por el teorema de Lusin (ver apendice) f es cuasi-


continua. Por lo tanto, fijamos un n
umero positivo

= ,
2 + m()

entonces existe un conjunto cerrado tal que m( r ) y f es


uniformemente continua en . En particular, existe > 0 tal que
p
|Th f (x) f (x)| ,
2m( )

siempre que |h| < , y x, x + h pertenezcan a .


Para cualquier h, tenemos que
Z
1
|Th f f |p dm p .
( h) 2
2.9 Continuidad de la traslacion en Lp () para 1 p < . 105

Para cualquier Rn tal que || < , calculamos


m ( r ( )) = m (( + ) r )
m( r ) + m (( + ) r )
+ m().
De esto u
ltimo se tiene que
m ( r [ ( )])
m( r ) + m ( r ( ))
(2 + m()) = .
Si h Rn tal que |h| = mn{, }, podemos estimar
Z Z
p
|Th f f | dm |Th f f |p dm

Z ( h)
+ |Th f f |p dm
Zr{ ( h)}
1
= |Th f f |p dm + p p .
( h) 2
Sea tal que || < , entonces
m ( r ( )) = m (( + ) )
m( r ) + m (( + ) )
+ m().
De esto u
ltimo se tiene que
m ( r [ ( )])
m( r ) + m ( r ( ))
(2 + m()) = .
Ahora consideremos el caso cuando es un subconjunto de Rn no acotado.
Entonces dado > 0 existe R > 0 suficientemente grande tal que para todo
|h| < 1
Z Z
p p 1
|Th f f | dm 2 |f |p dm p p .
{|x|>2R} {|x|>R} 2
Para tal R, existe 0 = 0 () tal que
1
sup kTh f f kp,{|x|<2R} .
|h|<0 2
106 Los espacios Lp

Por lo tanto

sup kTh f f kp, kTh f f kp,{|x|<2R}


|h|<0

+ 2kf kp,{|x|>R}
< .

on 2.76. El teorema 2.75 es falso cuando p = .


Observaci
Captulo 3

Operadores Integrales

3.1. Desigualdades b
asicas y algunos opera-
dores importantes
Definicion 3.1 (Operadores Compactos). Supongamos que X e Y son es-
pacios de Banach y B la bola unitaria en X. Un operador lineal T : X Y
se dice que es compacto si la clausura de T (B) (T (B)) es compacta en Y .
De este hecho es claro que T es acotado.

Observaci on 3.2. Esta definicion es equivalente a decir que T es compacto


si y solo si toda sucesion {xn } acotada en X contiene una subsucesion {xnk }
tal que {T (xnk )} converge puntualmente en Y .

Teorema 3.3 (Arzela-Ascoli). Supongamos que (X, d) es un espacio


metrico compacto. Entonces un conjunto F C(X) (C(X) es el espacio
de las funciones continuas de X en si mismo) es compacto si y solo si F es
cerrado, acotado y equicontinuo.

Definici
on 3.4. Un espacio metrico (X, d) se dice que es -compacto si se
puede expresar como la union contable de subespacios compactos.

Teorema 3.5. Sea (X, d) un espacio metrico -compacto y una medida de


Borel en X. Si K : X X R es una funcion continua tal que
Z
|K(x, y)| d(x) C para casi todo y X
X

y Z
|K(x, y)| d(y) C para casi todo x X.
X

107
108 Operadores Integrales

Entonces el operador integral

T : Lp () Lp ()

dado por Z
T f (x) = K(x, y)f (y) d(y)
X
es compacto para 1 p .

Demostracion. Supongamos que 1 < p < . Sea q el n umero conjugado de


p, entonces aplicando la desigualdad de Holder al producto

|K(x, y)f (y)| = |K(x, y)|1/q |K(x, y)|1/p |f (y)|




se tiene
Z
|K(x, y)f (y)| d(y)
X
Z 1/q Z 1/p
p
|K(x, y)| d(y) |K(x, y)||f (y)| d(y)
X X
Z 1/p
1/q p
C |K(x, y)||f (y)| d(y)
X

para casi todo x X. Por el teorema de Tonelli resulta


Z Z p
|K(x, y)||f (y)| d(y) d(x)
X X
Z Z
p/q
C |K(x, y)||f (y)|p d(y)d(x)
X X Z
p/q+1
C |f (y)|p d(y).
X

Dado que f LP (), entonces la integral de arriba es finita, luego por el


teorema de Fubini, concluimos que K(x, ) L1 () para casi todo x X,
as T f esta bien definido c.t.p. Finalmente, resulta
Z
|T f (x)|p d(x) C p/q+1 kf kpp ,
X

es decir
kT f kp Ckf kp .
3.1 Desigualdades basicas y algunos operadores importantes 109

Por lo tanto T f es acotado.

Ahora, fijemos y0 X y sea  > 0. Por la continuidad de en X X


existe > 0 tal que si d(y, y0 ) < entonces, |K(x, y) K(x, y0 ) < | para
cada x X. Luego, si d(y, y0 ) < y f Lp () satisface kf kp 1, entonces
la desigualdad de Holder implica
Z

|T f (x) T f (x0 )| = [K(x, y) K(x, y0 )]f (y) d(y)

ZX
< |f (y)| d(y)
X
[(X)]1/q kf kp
[(X)]1/q .

As, hemos demostrado que {T f : kf kp 1} es un subconjunto de C(X) el


cual es k kp -acotado y equicontinuo, en virtud del teorema de Arzela-Ascoli
concluimos que T es un operador compacto.
La desigualdad de Minkowski (teorema 2.19) establece que la norma k kp
de la suma de dos funciones en Lp es al menos la suma de la norma k kp
de cada una de estas funciones. El siguiente resultado generaliza el teorema
2.19.
Teorema 3.6 (Desigualdad integral de Minkowski). Sean (X, A1 , ) y (X, A2 , )
espacios de medidas -finitas. Supongamos que f es una funcion A1 A2 me-
dible y f (, y) Lp () para todo y Y . Entonces para 1 p se tiene
que
Z Z p 1/p Z Z 1/p
p
f (x, y) d d
|f (x, y)| d d.
X Y Y X

Demostracion. Puesto que


Z Z

f (x, y) d |f (x, y)| d

Y Y

para p = 1, entonces
Z Z Z Z

f (x, y) d d |f (x, y)| dd,

X Y X Y

por Fubini Z Z Z Z

f (x, y) d d |f (x, y)| dd.

X Y Y X
110 Operadores Integrales

Ahora, si p = , entonces
Z Z

f (x, y) d |f (x, y)| d

Y Y
Z
kf (, y)k d,
Y

as, aplicando Fubini una vez mas se tiene


Z Z Z Z 

f (x, y) d d kf (, y)k d d

X Y X Y
Z Z 
kf (, y)k d d.
Y X

Ahora, supongamos que 1 < p < , entonces por Fubini y el Teorema 2.22
(Desigualdad de Holder) tenemos
Z Z p

f (x, y) d d

X Y
Z Z Z p1 !

= f (x, y) d f (x, y) d
d
X Y Y
Z Z Z p1

|f (x, y)| d f (x, y) d
d
X Y Y
Z Z Z p1 !

= |f (x, y)| f (x, y) d d d
Y X Y

Z Z 1/p Z Z q(p1) !1/q

|f (x, y)|p d f (x, y) d
d d
Y X X Y

Z " Z 1/p Z Z p 1/q #



= |f (x, y)|p d f (x, y) d d
d,
Y X X Y

luego
 Z Z p 11/q Z Z p 1/p

f (x, y) d d = f (x, y) d d

X Y X Y
Z Z 1/p
p
|f (x, y)| d d
Y X
3.1 Desigualdades basicas y algunos operadores importantes 111

Teorema 3.7. Sea K una funcion medible en (0, ) (0, ) tal que
K(x, y) = 1 K(x, y) para todo > 0 y ademas
Z
|K(x, 1)|x1/p dx = C < .
0

Para f Lp () definamos
Z
T f (y) = K(x, y)f (x) dx.
0
Entonces,
kT f kp Ckf kp .
Demostracion. Sea
Z 1/p
p
kT f kp = |T f (y)| dy
0
Z
Z
p 1/p

=
K(x, y)f (x) dx dy .
0 0

Escribiendo x = zy, entonces dx = ydz, luego por la desigualdad integral de


Minkowski
 Z Z p 1/p


K(zy, y)f (zy)y dz dy

0 0
 Z Z p 1/p
1

=
y K(z, 1)f (zy)y dz dy

0 0
Z
Z
p 1/p

= K(z, 1)f (zy) dz dy


0 0
Z Z 1/p
p
k(z, 1) |f (zy)| dy dz.
0 0
1
Ahora, si x = zy, entonces z dx = dy, luego
Z Z 1/p
p
K(z, 1) |f (zy)| dy dz =
0 0
Z Z 1/p
p 1
K(z, 1) |f (x)| z dx dz =
0 0
Z 
1/p
K(z, 1)z dz kf kp .
0

As, hemos demostrado que


kT f kp Ckf kp .
112 Operadores Integrales

Los siguientes resultados representan la version continua de las desigual-


dades de Hilbert y Hardy, respectivamente.

Teorema 3.8 (Desigualdad de Hilbert). Sean f Lp (m) y g Lq (m).


Entonces
Z Z
f (y)g(x)
dydx   kf kp kgkq .

0 0 x+y sen p

Demostracion. Sean f Lp (m) y g Lq (m). Note que


Z Z Z
|f (y)g(x)|
Z
f (y)g(x)
dydx dxdy.

0 0 x+y 0 0 x+y

Sea y = xz, entonces dy = xdz, luego por Fubini tenemos que


|f (y)g(x)| |f (xz)g(x)|
Z Z Z Z
dydx = xdzdx
0 0 x+y 0 0 x(1 + z)
Z Z
|f (xz)g(x)|
= dzdx
0 0 1+z
Z Z
1
= |f (xz)g(x)| dxdz.
0 1+z 0

Si u = xz, entonces du = zdx, ademas x = uz , aplicando la desigualdad de


Holder tenemos

Z
Z
1
|f (xz)g(x)| dxdz
0 1+z 0
Z Z 
1  u 
1
= f (u)g z du dz

0 1+z 0 z
Z Z 1/p Z   1/q
1 u q q
|f (u)| du g z du dz.
0 1+z 0 0 z
3.1 Desigualdades basicas y algunos operadores importantes 113

u
Ahora, para w = z
tenemos zw = u y zdw = du, por lo tanto
Z Z 1/p Z 1/q
1 q q+1
|f (u)| du |g(w)| z du dz
0 1+z 0 0
1
!

z q 1
Z
= dz kf kp kgkq .
0 1+z
1
!

z (1 q )
Z
= dz kf kp kgkq .
0 1+z
1
!

z p dz
Z
= kf kp kgkq .
0 1+z

=   kf kp kgkq ,

sin
p
ver Lema A.2. Finalmente,
Z Z
f (y)g(x)

dxdy   kf kp kgkq .

0 0 x+y
sen p

Definici
on 3.9 (Operador de Hardy). Sea f una funcion medible y positiva
en (0, ), el operador de Hardy se define como
1 x
Z
Hf (x) = f (y) dy.
x 0
Teorema 3.10 (Desigualdad de Hardy). Sea f Lp (0, ) positiva y 1 <
p < . Entonces
p
kHf kp kf kp .
p1

Demostracion. Observe que si y = zx, entonces dy = xdz, luego


1 x
Z
Hf (x) = f (y) dy
x 0
1 1
Z
= f (xz) xdz
x 0
Z 1
= f (xz)dz.
0
114 Operadores Integrales

Ahora, empleando la desigualdad integral de Minkowski, obtenemos


Z  Z x p 1/p Z Z 1 p 1/p
1
f (y) dy dx = f (zx) dz dx
0 x 0 0 0
Z 1 Z 1/p
p
(f (zx)) dx dz
0 0
Z 1 Z 1/p
1/p p
= z (f (u)) du dz
0 0
p
= kf kp .
p1

Observacion 3.11. Este ultimo resultado nos dice que H B(Lp (m)) donde
B(Lp (m)) denota al conjunto de todos los funcionales acotados en Lp (m).
Definicion 3.12. Sea T un operador lineal y acotado, un operador T se
dice que es el operador adjunto del operador T si satisface la identidad de
dualidad, es decir, si
hT x, yi = hx, T yi
para todo x X, y Y donde X e Y son espacios de Banach.
Teorema 3.13. Al menos de manera formal el adjunto del operador de Hardy
esta dado por Z
dx
H f (y) = f (x)
y x
para f 0.

Demostracion. Empleando la definicion 3.12 y el teorema de Fubini resulta


Z
hHf, gi = Hf (x)g(x) dx
Z0  Z x 
1
= f (y) dy g(x) dx
0 x 0
Z
1
 Z 
= (0,x) (y)f (y) dy g(x) dx
0 x 0
Z Z 
g(x)
= (y,) (x)f (y) dy dx
0 0 x
Z Z
dx
= f (y) g(x) dy
0 y x
= hf, H gi.
3.2 Lp es un espacio reflexivo para 1 < p < 115

Por lo tanto Z
dx
H g(y) = g(x) .
y x

3.2. Lp es un espacio reflexivo para 1 < p <


En lo sucesivo, necesitaremos el siguiente resultado, el cual es una conse-
cuencia del Teorema de Hahn-Banach (version norma).

Teorema 3.14. Sean (X, +, , k k) un espacio normado, Y un subespacio


de X y x0 X tal que
= nf kx0 yk > 0
yY

es decir, la distancia de x0 a Y es positiva.

Entonces, existe f un funcional lineal y acotado en X tal que

f (y) = 0

para todo y Y ,
f (x0 ) = 1
y
1
kf k = .

Observaci
on 3.15. Sean X e Y dos espacios vectoriales, un funcional lineal

T : X Y

se dice que es un isomorfismo, si T es uno-uno y sobreyectivo. Ademas, si X


e Y son espacios normados tal que kT (x)k = kxk para cada x X, entonces
se dice que T es un isomorfismo isometrico y X e Y son isometricamente
isomorfos.

Denotemos por X al espacio dual de X . Vamos a demostrar que X es


isometricamente isomorfo al subespacio X .

Supongamos que X es un espacio normado. Para cada x X sea (x) un


funcional lineal en X definido por

(x)(f ) = f (x)
116 Operadores Integrales

para cada f X . Dado que

|(x)(f )| kf kkxk,

el funcional (x) es acotado, de hecho

k(x)k kxk.

As (x) X

El siguiente resultado es la clave para entender la relacion entre X y X .

Proposicion 3.16. Sea (X, +, , k k) un espacio normado. Entonces para


cada x X
kxk = sup{|f (x)| : f X ; kf k = 1}.

Demostracion. Fijemos x X. Si f X con kf k = 1, entonces

|f (x)| kf kkxk kxk.

Por otra parte, si x 6= 0, entonces

= dist(x, {0}) = nf kx yk = kxk > 0,


y{0}

por el teorema 3.14 existe g X tal que

1
g(x) = 1 y kgk = .
kxk

Sea f = kxkg, note que

1
kf k = kxkkgk = kxk =1
kxk
y
f (x) = kxkg(x) = kxk,
luego
kxk sup{|f (x)| : f X , kf k = 1} kxk,
por lo tanto
kxk = sup{|f (x)| : f X , kf k = 1}.
3.2 Lp es un espacio reflexivo para 1 < p < 117

Observaci
on 3.17. En virtud del resultado anterior no es difcil demostrar
que
k(x)k = kxk,
lo que nos dice que es un isomorfismo isometrico de X en (X).

El funcional lo llamaremos la inmersion natural de X en X .

on 3.18. Un espacio normado (X, +, , k k) se dice que es reflexivo


Definici
si
(X) = X
en cuyo caso X es isometricamente isomorfo a X .

Teorema 3.19. Lp () con 1 < p < es reflexivo.

1 1
Demostracion. Si 1 < p < y p
+ q
= 1, consideremos

: Lq () (Lp ()) ,

definido por
(g) = (Fg )
R
para g Lq () donde Fg (f ) = X gf d.
Observe que es un funcional lineal y acotado, la acotacion la obtenemos
del hecho que

|(g)| = |(Fg )| kkkFg k


= kkkgkq ,

la u
ltima igualdad es consecuencia del Teorema
R 2.61. De nuevo por el Teorema
un f Lp () tal que (g) = X gf d para todo g Lq ().
2.61 existe alg
Por otra parte, para w (Lp ()) tenemos que
Z
w(Fg ) = Fg (f ) = f gd = (g) = (Fg )
X

para todo g Lq (). El Teorema 2.61 garantiza que w = . Es decir, la


inmersion natural
: Lp () (Lp ())
es sobreyectiva. As hemos demostrado que Lp () es reflexiva.
118 Operadores Integrales

3.3. El espacio L2
Definici on 3.20 (Producto interior). Sea (X, +, .) un espacio vectorial, una
funcion h, i : X X F, donde F = R o F = C, tal que satisface:

(a) hf + g, hi = hf, hi + hg, hi para todo f, g, h X,

(b) hcf, gi = chf, gi para todo f, g X y cualquier escalar c.

(c) hf, gi = hg, f i para todo f, g X.

(d) 0 hf, f i < + para todo f X.

(e) hf, f i = 0 si y solo si f = 0.

se llama producto interior.

Observaci on 3.21. hf, gi denota el conjugado de hf, gi el cual se cumple si


X es un espacio vectorial complejo, en caso de que X es un espacio vectorial
real (c) es nuevamente hf, gi = hg, f i. Un espacio vectorial dotado de un
producto interno, se llama espacio vectorial con un producto interior.

Todo producto interno genera una norma definida por


p
kf k = hf, f i.

Ademas se tiene que la desigualdad de Cauchy-Schwarz

|hf, gi| kf kkgk.

Definicion 3.22 (Espacio de Hilbert). Un espacio vectorial con un producto


interno se dice que es un espacio de Hilbert si es completo con respecto a la
norma generada por su producto interno.

Teorema 3.23. Una norma k k sobre un espacio vectorial esta inducida por
un producto interno si y solo si satisface la ley del paralelogramo, es decir, si
y solo si
kf + gk2 + kf gk2 = 2(kf k2 + kgk2 )
para cualesquiera vectores f, g.

Ahora, consideremos el caso p = 2.


3.3 El espacio L2 119

Definicion 3.24. Sea (X, A, ) un espacio de medida y p = 2. Una funcion


f definida de X en R o C (f : X R o C) se dice que pertenece a L2 ()
(pre-Lebesgue) si Z
|f |2 d < .
X
Es decir
 Z 
2
L2 () = f : X R o C | f es medible y |f | d < .
X

Por la construccion hecha en la seccion 2.3 podemos definir

L2 () = L2 (X, A, ) = L2 / .

En virtud del Teorema 2.29 para el caso p = 2, la funcion k k2 : X R o C


definida por
Z  21
2
kf k2 = |f | d
X

es una norma sobre L2 ().


Para el Teorema 2.29 (L2 (), k k2 ) es un espacio completo. Ahora consi-
deremos en L2 () el producto interno
Z
hf, gi = f gd,
X

f, g L2 ().
Observemos que este producto interno genera la norma k k2 y ademas
k k2 satisface la ley del paralelogramo (ver ejercicio 1). Por lo tanto se tiene
la siguiente
on 3.25. (L2 (), k k1 ) es un espacio de Hilbert.
Definici
Sea X un espacio con un producto interior. Si A es un subconjunto (no
vaco) de X, entonces el complemento ortogonal A de A es el conjunto de
todos los vectores que son ortogonales a cualquier vector de A, esto es,

A = {x X | x y para todo y A} ,
x y si y solo si hx, yi = 0.
De la linealidad y continuidad del producto interior es claro que A es
un subespacio cerrado de X tal que A = (A) y A A = {0}.
Teorema 3.26. Si M es un subespacio cerrado de un espacio de Hilbert H,
entonces H = M M .
120 Operadores Integrales

Observaci on 3.27. Dado que todo producto interno es continuo, se sigue


que todo vector y en un espacio X con producto interior define un funcional
lineal fy : X C va la formula

fy (x) = hx, yi, (3.1)

como veremos en el siguiente resultado. Si X es un espacio de Hilbert, en-


tonces todo funcional lineal y contnuo sera de la forma (3.1).

Teorema 3.28 (F. Riesz). Si H es un espacio de Hilbert y f : H C es un


nico vector y H tal que
funcional lineal y continuo, entonces existe un u

f (x) = hx, yi

para todo x H. Ademas kf k = kyk.

Demostracion. Sea F : H C un funcional lineal y continuo en H. Sea M


su n
ucleo, es decir

M = ker(f ) = f 1 (0) = {x H | f (x) = 0} ,

dado que f es un funcional lineal y continuo, se tiene que M es un subespacio


cerrado en H. Luego, si M = H, entonces y = 0 satisface f (x) = hx, yi = 0
para cada x H. Por lo tanto podemos suponer que M ( H (subespacio
propio de H). Entonces existe alg un x0 H con f (x0 ) = 1 y (por el Teorema

un vector w M tal que w 6= 0. Ahora, note que si x H, entonces
3.26) alg
x f (x)x0 M y hx f (x)x0 , wi = 0 o f (x)hx0 , wi > 0 y as hx0 , wi =6 0.
w
Observe que el vector y = hx0 ,wi satisface f (x) = hx, yi para todo x H.
Unicidad. Note que si hx, yi = hx, y1 i para cada x H, luego si x = yy1 ,
entonces hy y1 , y y1 i = 0, de aqu se desprende que y = y1 . Finalmente,
en virtud de la desigualdad de Cauchy-Schwarz |f (x)| |hx, yi| kxkkyk
y
se tiene que kf k kyk. Por otra parte, si y 6= 0, entonces x = kyk satisface
kxk = 1 y kf k |f (x)| = hy/kyk, yi| = kyk . As kf k = kyk.

Si H es un espacio de Hilbert, entonces el Teorema de F. Riesz nos muestra


que una funcion y 7 fy donde fy (x) = hx, yi puede definirse de H en H .
En vista de las propiedades:

a) fy + fz = fy+z

b) fy = fy

c) kfy k = kyk
3.3 El espacio L2 121

Se puede ver facilmente que fy es una aplicacion lineal conjudada la cual


es una isometra de H en H . Gracias a esta isometra, podemos demostrar
que todo espacio de Hilbert es reflexivo.
Corolario 3.29. Todo espacio de Hilbert es reflexivo.
Demostracion. Sea H un espacio de Hilbert y F : H C un funcional
lineal. Definamos : H C va la formula (y) = F (fy ), ahora note que:
i) (y + z) = F (fy+z ) = F (fy + fz ) = F (fy ) + F (fz ) = (y) + (z)

ii) (y) = F (fy ) = F (fy ) = F (fy ) = F (fy ) = (y)

iii) |(y)| = |F (fy )| = |F (fy )| kF kkfy k = kF kkyk


As en virtud de I), II) y III9 se tiene que H . Entonces por el Teorema
de F. Riesz, existe un u nico x X tal que hy, xi = (y) = F (fy ) para todo
y H. Esto implica que, para x H se tiene

x(fy ) = fy (x) = hx, yi = F (fy )

para cada y H, de esto u ltimo se tiene que x = F , esto nos dice que
la inmersion natural es sobreyectiva en H , por lo tanto H es un espacio
reflexivo.

3.3.1. Teorema de Radon-Nikodym


En esta subseccion presentamos una demostracion alternativa del clasi-
co Teorema de Radon-Nikodym la cual es independiente del Teorema de la
descomposicion de Hahn, sin embargo esta demostracion estara basada en el
Teorema de F. Riesz, esta demostracion alternativa se debe a Von Neumann,
la misma la dividiremos en varios Lemas.
Lema 3.30. Sean y medidas Rfinitas sobre un espacio medible (X, A) y
sea = + . Definamos F (f ) = X f d para f L2 (). Entonces F es un
funcional lineal y acotado en L2 ().
Demostracion. Note que L2 () L1 () L1 () ya que, en virtud de la
desigualdad de Holder con p = q = 2, resulta
Z Z  21
1
2
|f |d ((X)) 2 |f | d
X X

y Z Z
|f |d |f |d ( < ).
X X
122 Operadores Integrales

Ademas esto nos dice que F esta bien definida, no es difcil ver que F es un
funcional lineal.
Ademas
Z  12
1
2
|F (f )| ((X)) 2 |f | d
X
Z  12
1
2
((X)) 2 |f | d
X
1
((X)) kf kL2 ()
2

R 1
donde kf kL2 () = kf k2 = X |f |2 d 2 , de esta manera hemos demostrado
que F es un funcional lineal y acotado en L2 ().

Lema 3.31. Sea g L2 () tal que F (f ) = hf, gi. Entonces 0 g 1 c.t.p.


y
R
i) (E) = E gd.
R
ii) (E) = E (1 g)d con E X.

Demostracion. Por el lema 3.30 y por el Teorema de F. Riesz existe un u


nico
funcional g L2 () tal que F (f ) = hf, gi. Note que
 
[ 1
{g > 1} = g 1+ ,
n=1
n

Sea En = g 1 + n1 , entonces


Z Z
1
(En ) = F (En ) = hEn , gi = En gd = gd (1 + )(En ).
E En n

As
1 1
(En ) (1 + )(En ) (1 + )((En ) + (En ))
n n
(En ) 1
(En ) + + (1 + )(En )
n n
de donde tenemos que

(En )
0 + (En ),
n
3.3 El espacio L2 123

de esto u
ltimo deducimos que

(En ) = 0 = (En )

y as (En ) = 0, por lo tanto ({g > 1}) = 0.


Por otra parte, sea

[
{g < 0} = {g 1/n}
n=1

y An = {g 1/n}. Entonces
Z
1 1
(An ) = F (An ) = hAn , gi = gd (An ) [(An ) + (An )],
An n n
luego
1 1
0 (An ) + (An ) (An ) 0,
n n
por lo tanto (An ) = 0 = (An ).
As (An ) = 0, entonces ({g < 0}) = 0. Con esto, hemos demostrado
que
0 g 1 c.t.p.
Finalmente, para cualquier conjunto E X se tiene que
Z
(E) = F (E ) = hE , gi = gd,
E

es decir, Z
(E) = gd.
E
Por otra parte como
(E) = (E) (E),
entonces
Z Z Z Z Z
(E) = En d gd = d gd = (1 g)d.
X E E E E

Lema 3.32. Si  , entonces  y g = 0 solo en un conjunto de


medida cero con respecto a la medida , en este caso tenemos que
Z
(E) = g 1 d.
E
124 Operadores Integrales

Demostracion. Supongamos que  , si (E) = 0, entonces (E) = 0,


as (E) = 0, as  . Consideremos ahora E = {x : g(x) = 0}, entonces
Z
(E) = gd = 0,
E

de esto se deduce que g 6= 0 c.t.p. luego g 1 6= 0 c.t.p..


Ahora para cualquier conjunto A A, consideremos g 1 = A . Entonces

Z Z Z Z
1
g d = E A d = EA d = (E A) = gd
E X X Z EA Z
= A gd = g 1 gd.
E E

Lema 3.33. Si  , entonces (1 g)g 1 es integrable con respecto a y


ademas Z
(E) = (1 g)g 1 d.
E

Demostracion. Si  , entonces
Z Z Z
1 1
(1 g)g d = g d gg 1 d
E E E Z
= (E) d = (E) (E) = (E).
E

(1 g)g 1 d.
R
Por lo tanto, (E) = E

Teorema 3.34 (Radon-Nikodym). Sea (X, A, ) un espacio de medida -


finito y sea una medida definida en A la cual es absolutamente continua
con respecto a , esto es,  . Entonces existe una funcion F no negativa
y medible tal que para cada E A se tiene que
Z
(E) = f d. (3.2)
E

La funcion f es u
nica en el sentido que si g es cualquier funcion medible
que satisface (3.2), entonces f = g c.t.p.[].
El siguiente ejemplo nos muestra que en el Teorema de Radon-Nikodym
la hipotesis de que debe ser una medida -finita no se puede omitir.
3.3 El espacio L2 125

Ejemplo 3.35. Sea X = [0, 1] y A la clase de todos los subconjuntos de


[0, 1] medibles. Sea la medida de Lebesgue y la medida de contar en A.
Entonces es finita y absolutamente continua con respecto a , pero no existe
una funcion f tal que Z
(E) = f d
E
para todo E A.
En efecto, si (E) = 0, entonces E = ya que es la medida de contar,
as (E) = () = 0, -medida de Lebesgue por lo tanto  .
Dado que ([0, 1]) = 1 entonces es finita. Por otra parte X = [0, 1]
es no numerable y ({x}) = 1 x [0, 1] lo que nos dice que no es una
medida -finita.
Ahora supongamos
R que existe f : [0, 1] [0, ) definida por F = f (x){x}
tal que (E) = E f d para todo E A. Sea x [0, 1], entonces
Z Z
0 = ({x}) = f d = f (x){x} d = f (x)({x}) = f (x)
{x} X

para todo x [0, 1]. R


Pero ([0, 1]) = [0,1] f d = 0 lo cual es una contradiccion.

Ejercicios
R
1. Demostrar que hf, gi = X
f gd con f, g L2 () es un producto in-
terno.

2. Sean f, g L2 (), demostrar que

|hf, gi| kf k2 kgk2 .

Esta desigualdad es conocida como la desigualdad de Cauchy-Schwarz.

3. Demuestre que la igualdad en la desigualdad de Cauchy-Schwarz se da,


es decir
|hf, gi| = kf k2 kgk2 .
si y solo si f y g son linealmente dependientes.

4. Demuestre que k k2 : L2 () R o C definida por


Z  21
2
kf k2 = |f | d
X

es una norma sobre L2 ().


126 Operadores Integrales

5. Demuestre que la norma k k2 satisface la ley del paralelogramo.

6. Si f L2 (), demuestre que kf k2 = sup |hf, gi|.


kgk2 =1

7. Demuestre que las siguientes normas no pueden ser inducidas por un


producto interno:

a) kxk = max1kn {|xk |} en Rn


b) kf k = sup |f (x)| en C[a, b]
x[a,b]
R 1
c) kf kp = X
|f |p d p en Lp () con p 6= 2.

8. Sean fn , gn L2 () para n N. Si
Z Z
2
lm (fn f ) d = lm (gn g)2 d = 0.
n X n X

Demostrar que Z Z
lm fn gn d = f gd.
n X X

9. Sea I = [0, ] y f L2 ([0, ], L, m). Sera posible tener simultanea-


mente Z
(f (x) sen x)2 dx 4
I
y Z
1
(f (x) cos x)2 dx ?.
I 9

10. Sea I = [0, 1]. Sea f una funcion Lebesgue medible. Demostrar que
f L2 (I, L, m) si y solo si f L1 (I, L, m) tal que existe una funcion
g creciente de modo que para todo intervalo cerrado [a, b] [0, 1] se
cumple que
Z b 2

f (x) dx (g(b) g(a)) |b a|.

a
R1
11. Sea f L2 ([0, 1], L, m) tal que kf k2 = 1 y 0 f dm > 0. Tambien,
para R, sea E = {x [0, 1] : f (x) }. Si 0 < < , demostrar
que
m(E ) ( )2 .
Esta desigualdad se conoce en la literatura como la desigualdad de
Paley-Zygmund.
3.4 Espectro de un Operador 127

12. Consideremos elR espacio de medida (X, A, ) com (X) = 1 y sean


f, g L2 (). Si X f d = 0, demostrar que
Z 2 "Z Z 2 # Z
f g d g 2 d g d f 2 d.
X X X X

13. Sea f L1 () L2 (). Demostrar que:

a) f Lp () para cada 1 p 2.
b) lmp1+ kf kp = kf k1 .
R R
14. Si x2 |f (x)|2 dx < y |f 0 (x)|2 dx < , demostrar que si x
0, entonces
Z 1/2 Z 1/2
2 2 2 0 2
x|f (x)| 4 x |f (x)| dx |f (x)| dx .
x x

15. Sea f una funcion definida en R tal que f (x) y xf (x) pertenecen a
L2 (R). Demostrar que
Z 2 Z 1/2 Z 1/2
2 2 2
|f (x)| dx 8 |f (x)| dx |x| |f (x)| dx .

16. Usar el Teorema 3.10 para demostrar el Teorema 2.42. Ayuda: Escoja
una sucesion {anP
}nN de n P tal que an+1 an n N.
umeros positivos
Considere AN = N a
n=1 n y defina f = n=1 an (n1,n) .

3.4. Espectro de un Operador


Definicion 3.36. Sea (X, +, , k k) un espacio normado, el espectro (T )
de un operador T acotado, T : X X es el conjunto de todos los n umeros
complejos tal que el operador T I es no invertible, donde I representa
el operador identidad

(T ) = { C : T I es no invertible}

Un numero complejo se llama un valor propio de un operador T si existe


un vector x diferente de cero tal que

T (x) = x.
128 Operadores Integrales

Observese que los valores propios de T son exactamente los n umeros com-
plejos para los cuales T I no es uno a uno, ademas ellos pertenecen al
espectro de T . El conjunto de todos los valores propios de T se conoce como
el espectro puntual de T y se denota por

p (T ) = { (T ) : T I es no uno-uno}
= { (T ) : T (x) = x para x 6= 0}.

Teorema 3.37. Sea H el operador de Hardy. Entonces

a) H es positivo en C[0, 1].

b) H es uno-uno.

c) El espectro puntual de H es el conjunto


 
1 1
p (H) = C : < {1}.
2 2

d) H es no compacto.

Demostracion. a) Sea f C[0, 1], queremos demostrar que H(f ) es continua


en 0. Sea  > 0 y escojamos > 0 tal que x, x0 [0, 1], luego si |xx0 | < ,
entonces |f (x) f (x0 )| < . Si x [0, 1] satisface 0 < x < , tenemos que
|f (x) f (0)| < . En particular, para cada x [0, 1] resulta

1 x
Z

|Hf (x) Hf (0)| = [f (y) f (0)] dy
x 0
1 x
Z
|f (y) f (0)| dy
x 0
1
< x
x
=

Esto nos demuestra que H(f ) es continua en 0. De esto u


ltimo es claro
que H(f ) es un operador positivo en C[0, 1].

un f C[0, 1] se tiene H(f ) = 0, entonces


b) Supongamos que para alg
Z x
f (y) dy = 0
0
3.4 Espectro de un Operador 129

para cada x [0, 1], en virtud del teorema fundamental del calculo obte-
nemos Z x 
d
f (x) = f (y) dy = 0
dx 0

para cada x [0, 1]. As H es uno-uno, de este hecho se concluye que H


es estrictamente positivo.

c) Observe que para 1 C[0, 1] se tiene que

1 x
Z
H(1)(x) = 1(y) dy = 1,
x 0

luego H(1) = 1, as = 1 es un valor propio de H, en vista de esto,


podemos suponer 6= 1.

Para hallar los valores propios de H, debemos hallar todos los n umeros
complejos = + i para los cuales existe una funcion continua distinta
de cero (f : [0, 1] C) tal que

1 x
Z
Hf (x) = f (y) dy = f (x) (3.3)
x 0

para x [0, 1]. Dado que H es uno-uno podemos ver que = 0 no es un


valor propio de H, luego 6= 0. Por otra parte, podemos escribir (3.3)
como Z x
f (y) dy = xf (x)
0
Diferenciando a ambos lados resulta

f (x) = f (x) + xf 0 (x),

as
xf 0 (x) + ( 1)f (x) = 0,
buscamos valores propios diferentes de 0 y 1, en este caso

xf 0 (x) = (1 )f (x)

f 0 (x)
 
1 1
= 1 ,
f (x) x
integrando  
1
ln f (x) = 1 ln x

130 Operadores Integrales

1
ln f (x) = ln x 1 ,
luego
1
f (x) = x 1 = e( 1) ln x
1

para cada 0 < x 1. Si x 0+ en (3.3), gracias a la regla de LHopital


tenemos
f (0) = f (0),
1
ltimo nos dice que la funcion f (x) = x 1 es un
es decir, f(0)=0, esto u
valor propio de H si y solo si
1
lm+ x 1 = 0.
x0

Luego,
1 1
x 1 = x +i 1
 
i
1 ln x
=e 2 + 2

2 2 i
ln x ln x
=e 2 + 2 e 2 + 2

As, vemos que


1
lm+ x 1 = 0
x0
2 2
si y solo si > 0, entonces
2 2 > 0 2 + 2 < 0
1 1
2 + + 2 <
4 4
 2  2
1 1
+ 2 <
2 2
| 1/2| < 1/2.
Por lo tanto, el espectro puntual de H es
 
1 1
p (H) = C : < {1}.
2 2

d) Dado que el espectro de H no es contable, entonces H no puede ser


compacto. A continuacion damos una demostracion directa de este hecho.
Sea {fn }nN en C[0, 1] definida por

1
nx, si 0 x n ;

fn (x) =

1, si n1 < x 1.

3.4 Espectro de un Operador 131

Claramente, kfn k = 1 para cada n N y


1 x
Z
Hfn (x) = fn (y) dy
x 0

nx
2,
si 0 x n1 ;
=
1
1 nx , si n1 < x 1.

Ademas,
0, si x = 0;

lm Hfn (x) =
n
1, si 0 < x 1,

pero g
/ C[0, 1] donde

0, si x = 0;

g(x) =

1, si 0 < x 1.

As, observamos que ninguna subsucesion de {Hfn }nN puede converger


uniformemente, por lo tanto H es no compacto.

Ejercicios

1. La funcion Gamma se define como


Z
() = t1 et dt (0, )
0

siempre que esta integral sea convergente.

a) Si , (0, ) demostrar que


Z 1
()()
= t1 (1 t)1 dt.
( + ) 0

b) Sea f continua en [0, ) para (0, ) y x 0 definamos


Z x
1
I f (x) = (x t)1 f (t) dt.
() 0

Demostrar que I (I f )(x) = I+ f (x).


132 Operadores Integrales

c) Definamos J f (x) = x I f (x). Demostrar que para 1 < p <

(1 1/p)
kJ f kp kf kp .
( + 1 1/p)

2. Sea 1 p < , r > 0 y h una funcion no negativa medible en (0, ).


Demostrar que:
R R x p p R pr1
a) 0 xr1 0 h(y) dy dx pr 0
x [h(x)]p dx
R R p p R p+r1
b) 0 xr1 x h(y) dy dx pr 0
x [h(x)]p dx

3. Sea k una funcion medible no negativa en (0, ) tal que


Z
k(x)xs1 dx = (s),
0

para 0 < s < 1, si 1 < p < y p1 + 1q = 1, ademas si f, g son funciones


medibles no negativas en (0, ). Demostrar que
Z Z
k(xy)f (x)g(y) dx dy
0 0
Z 1/p Z 1/q
1 p2 p q
(p ) x [f (x)] dx [g(x)] dx
0 0

R f (y)
4. Sea F (x) = 0 x+y
dy; 0 < x < . Si 1 < p < . Demostrar que


kF kp   kf kp .

sen p

5. Demostrar que

Z Z
f (x)g(y)
dx dy kf k2 kgk2

0 0 x+y

para f, g L2 ((0, ), L, m).

6. Sea K : [0, 1] [0, 1] R definida por


(
0 si 0 t s 1
K(s, t) =
1 si 0 s t 1
3.4 Espectro de un Operador 133

y V : Lp [0, 1] Lp [0, 1] (1 p ) el operador definido por


Z 1 Z t
V x(t) = K(s, t)x(s) ds = x(s) ds
0 0

para x Lp [0, 1]. Este operador se conoce como el operador de Volterra.


Demostrar que el adjunto del operador de Volterra esta dado por
Z 1

V y(s) = y(t) dt.
s
R
7. Sea k una funcion medible no negativa en (0, ) tal que 0 k(x)xs1 dx =
(s) para 0 < s < 1. Sea f una funcion medible no negativa en (0, ).
Definamos Z
T f (x) = k(xy)f (y) dy.
0
Demostrar que  
1
kT f k2 kf k2 .
2
Que puede decirse de T f y (s) si k(x) = ex ?
8. Sea (X, A, ) un espacio de medida y f Lp (X, A, ). Si
Z
1
({x X : |F (x)| > }) |f | d.
{xX:|F (x)|>}
Demostrar que
p
kF kp kf kp .
p1
9. Sea f Lp ((0, ), L, m). Para cada t > 0, definamos
Z  s ds
Sf (t) = mn 1, f (s) ,
0 t s
demostrar que
p2
kSf kp kf kp .
p1
10. Sea T : Lp () Lp () un operador continuo donde 1 < p < y
0 r p. Demostrar que:
a) Si f Lp (), entonces |f |pr |T f |r L1 () y
Z
|f |pr |T f |r d kT kr (kf kp )r .
134 Operadores Integrales

un f Lp () con kf kp 1 tenemos que


b) Si para alg
Z
|f |pr |T f |r d = kT kr ,

entonces
|T f | = kT k|f |.
11. Si f L1 ((0, ), L, m), demostrar que:

a) Z Rx
Z
1
ln f (t) dt
e x 0 e f (x) dx
0 0

b) Z Z
 1 Rx
ln f (t)dt
 e
e x 0 xp dx f (x)xp dx
0 1p 0
para 0 < p < 1.
c) Sea f una funcion no negativa y medible en (0, b), 0 < b tal
Rb
que 0 < 0 [f (x)]p dx < . Demostrar
(c1 ) para p 1,
Z b Z x p Z b 
1 dx t dt
f (t)dt 1 [f (t)]p .
0 x 0 x 0 b t
(c2 ) para p 1,
Z b  Z b p
p1 +1 1
x f (t)dt dx
0 x 0
Z b  x  p1 
p p1 p
b p 1 [f (x)]p dx.
p1 0 b
d)
1 1
12. Sea 1 < q < y p tal que +
= 1. Definamos
p q
Z
T (f )(x) = k(x, t)f (t) dt

Demostrar que para toda f Lp (R) T es un operador lineal y acotado


de Lp (R) en Lq (R) y ademas
Z Z 1/q
q
kT k |k(x, t)| dt dx .
3.4 Espectro de un Operador 135

Rx
13. Dado 1 < p < y T f (x) = x1/p 0 f (t) dt con p1 + 1q = 1. Demostrar
que T es un operador lineal y acotado de Lq (0, ) en C0 ((0, )).
14. Sean s < r 1 y r > 1. Sea f definida en (0, ) tal que
Z
|f (x)|r xs dx < .
0
Rx
Sea F (x) = 0 f (t) dt. Demostrar que
1/r
F (x) r s
Z Z 1/r
r r s
x x dx
|f (x)| x dx .
0 rs1 0

15. Sean s < r 1 y r > 1. Sea f una funcion diferenciable en casi todo
(0, ) tal que Z
|f 0 (x)|r xs dx < ,
0
ademas f satisface las siguientes propiedades:
a) f (0) = 0.
b) f () = lmt f (t) = 0.
Demostrar que
Z 1/r Z 1/r
r sr r 0 r s
|f (x)| x dx |f (x)| x dx .
0 rs1 0

16. Sea > 0, si la ecuacion diferencial


d 0 q/p0 0
(y (x)) + g(x) [y(x)]q/p = 0
dx
tiene solucion y tal que
i) y(0) = y() = 0.
ii) y(x) > 0.
iii) y 0 (x) > 0.
0 < x < . Demostrar que
Z 1/q Z 1/p
q 1/q 0 p
|u(x)| g(x) dx |u (x)| dx
0 0

para toda funcion u(x) tal que


u(x) AC[0, )
u(0) = lm u(t) = 0.
t
136 Operadores Integrales

17. Supongamos que f y g son funciones no negativas y medibles en (0, )


y que
R
a) 0 f (t)t1/2 dt < .
R
b) 0 [g(t)]2 dt < .
Demostrar que Z Z x
g(x)
f (t) dt dx < .
0 0 x
18. Sea (X, A, ) un espacio de medida y u, v funciones no negativas A-
medibles tal que
Z
t ({x X : u(x) t}) v d.
{u(x)t}

Si u, v Lp (X, A, ) demostrar que


p
kukp kvkp .
p1
Rx
19. Sea f 0 y F (x) = 0
f (t) dt. Demostrar que
Z 1 1/p Z 1 1/p
p p p
[F (x)] dx [f (x)] dx
0 p1 0

para 1 < p < .


20. Sea T : C(X) R con (X) < (C(X) denota R el espacio de todas
las funciones continuas en X) tal que T (f ) = X f d. Demostrar que
T es un operador lineal y hallar kT k.
21. Sea una funci
dt
on Lebesgue medible, definida en (0, 1) tal que t(t)
Lp (0, 1), L, t . Demostrar que
Z 1

(1 + | log t|)1
p
dt p 1 kt(t)kLp ( dtt ) .
(s) ds
t Lp ( t )

22. Sea g una funcion medible y positiva en (0, ). Sea una funcion
convexa en (0, ). Demostrar que
Z  Z x  Z
1 dx dx
g(t) dt (g(x)) .
0 x 0 x 0 x
Captulo 4

Operador Maximal

4.1. Funciones localmente integrables


Definici on 4.1. Una funcion f : Rn C se dice que es localmente inte-
grable si Z
|f | d <
K
para todo conjunto compacto K Rn . El espacio de las funciones localmente
integrables se denota por L1,loc (Rn )
Note que L1 (Rn ) ( L1,loc (Rn ). En efecto, si f L1 (Rn ) y K Rn es un
conjunto compacto, entonces
K |f | |f |,
luego Z Z
K |f | d |f | d < ,
Rn R
es decir Z
|f | d < ,
K
por lo tanto f L1,loc (Rn ). Por otra parte observese que
1
f (x) = n1
/ L1 (Rn ),

|x|
pero f restringida a cualquier bola cerrada de centro 0 y radio r > 0, es
localmente integrable, es decir
1
f (x) = B(0,1) (x) L1,loc (Rn ).
|x|n1

137
138 Operador Maximal

4.2. El teorema de cubrimiento de Vitali


Teorema 4.2. Sea E Rn un conjunto acotado. Sea F la coleccion de bolas
abiertas concentradas en puntos de E tal que todo punto de E es el centro
de alguna bola en F. Entonces existe una sucesion B1 , B2 , . . . de bolas de F
tal que

1. Las bolas B1 , B2 , . . . son disjuntas


S
2. E 3B
1

Nota: E no esta cubierto por bolas disjuntas sin embargo esta cubierto
por bolas concentricas de radio tres veces su radio original.

4.3. Funci
on Maximal de Hardy-Littlewood
Definicion 4.3. Sea L1,loc(Rn ) . La funcion maximal de Hardy-Littlewood de
f se define como:
Z
1
M f (x) = sup  |f (y)| dy.
0<r< m B(x, r)
B(x,r)


donde B(x, r) = y Rn : |y x| < r es una bola abierta en Rn .

De la definicion de M f , podemos ver que las siguientes propiedades se


satisfacen:

(i) 0 M f (x)

(ii) M (f + g)(x) M f (x) + M g(x)

(iii) M (f )(x) = ||M f (x).

Se demuestra facilmente que la funcion f (t) = |t| con > 0 tiene


M f (x) = para cada x Rn . Nuestro proximo objetivo, es calcular M f
cuando f Lp (Rn ). Para f L (Rn ) vemos que:

M f (x) kf k

para cada x Rn , es decir M f L (Rn ).


4.3 Funcion Maximal de Hardy-Littlewood 139

Sin embargo, para f L1 (Rn ), no necesariamente M f L1 (Rn ). Por


ejemplo, f = [0,1] L1 (Rn ) despues de algunos calculos rutinarios obtene-
mos que:
1


si x 1



2x


M f (x) = 1 si 0 < x < 1



1


si x 0.


2(1 x)

Pero M f L1 (Rn ).
Proposici
on 4.4. M f es semicontinua inferiormente y por lo tanto medible.
Demostracion. Para demostrar que M f es semicontinua
 inferiormente, de-

bemos verificar que para cada > 0, el conjunto x Rn : M f (x) >
es abierto, para ello demostremos que el conjunto x Rn : M f (x)

es cerrado. Fijemos > 0 y supongamos que x x Rn : M f (x) ,
entonces
existe una sucesion {xk }kN en el conjunto x Rn : M f (x)
tal que xk x en Rn cuando k . Primero observemos que, co-
mo xk x, tendremos lm B(xk , r)B(x, r) = para todo r > 0. Sea
k
Ak = B(xk , r)B(x, r) y fk = f Ak , luego, se tiene que
|fk | |f (y)| y lm fk = 0, c.t.p.
k

Por el teorema de la convergencia dominada, tenemos:


Z
lm |fk | dy = 0. (4.1)
k
Rn

Pero
B(x, r) B(xk , r) 4 B(x, r) B(xk , r)
y  
m B(xk , r) = m B(x, r) ,
entonces
Z
1
 |f (y)| dy
m B(x, r)
B(x,r)
Z Z
1 1
 |f (y)| dy +  |f (y)| dy.
m B(xk , r) m B(xk , r)
B(xk ,r)B(x,r) B(xk ,r)
140 Operador Maximal

As, por (4.1) se tiene que


M f (x) ,

por lo tanto x x Rn : M f (x) con esto completamos la demostra-
cion.

La siguiente propiedad, se introduce a titulo de precaucion. La funcion


M f , la podemos ver definitivamente mas grande que |f |, sin embargo, no es
cierto que si f L1 (Rn ), entonces M f L1 (Rn ). En efecto, si M f L1 (Rn ),
se tiene que f = 0. Verifiquemos esto, si a > 0 arbitraria y |x| > a, resulta

Z
1
M f (x)  |f (y)| dy
m B(x, 2|x|)
B(x,2|x|)
Z
1
 |f (y)| dy
m B(0, 2|x|)
B(0,a)
Z
const
= |f (y)| dy,
|x|n
B(0,a)

dado que |x|n no es integrable para |x| > a, se tiene que:


Z
|f (y)| dy = 0.
B(0,a)

De la arbitrariedad de a, se concluye que f = 0. Otro ejemplo, para el caso


n = 1, sea
1
f (x) =  (x),
x log2 x 0,1/2
usemos r = x, note que f L1 (R) y

Z2x
1
M f (x) |f (y)| dy
2x
0
Zx
1 dy

2x y log2 y
0
1
= .
2x log x
4.3 Funcion Maximal de Hardy-Littlewood 141

Como
1
2x log x
no es integrable en la cercanas de x = 0, se tiene que M f
/ L1 (R). Ahora
bien, en descargo de esta situacion negativa, tendremos que si f L1 (Rn ),
entonces M f pertenece a L1 -debil.
on 4.5. Sea (X, A, ) un espacio de medida, una funcion f A
Definici
medible, se dice que pertenece al espacio de las funciones Lp -debil denotado
por WeakLp = Lp, (0 < p < ) si
  C p 

f
L(p,)
= nf C > 0 : Df () > 0

es finito. Bajo el convenio de que nf = . Como es usual dos funciones
en L(p,) se consideran iguales si ellas son iguales -c.t.p.

Ejercicios
|x|1/2
1. Dada f (x) = 1+|x|1/2
para x R, demostrar que f
/ L (m) pero
f WeakL2 .

Teorema 4.6. Si f L1 (Rn ), entonces


Z
n C 
m {x R : M f (x) > } |f (y)| dm.

Rn

As M f L(1,) .

Demostracion. Para cada R+ , definimos A = x : M f (x) > , de la
definicion 4.3, se desprende que para cada x A , existe 0 < r < (el cual
depende de x) tal que
Z
1
 |f (y)| dm > .
m B(x, r)
B(x,r)

Notese que esta u


ltima expresion la podemos escribir como
Z
 1
m B(x, r) < |f (y)| dm. (4.2)

B(x,r)

supongamos que A 6= , de lo contrario el resultado se tiene trivialmente.


Notemos que para hacer uso del teorema 4.2 debemos tener que A debe
142 Operador Maximal

ser acotado, a priori esto no se ve claro, sin embargo podemos considerar el


conjunto A B(0, k) (k fijo) en lugar del conjunto A . Ahora, sea F una
coleccion de bolas abiertas B con centro en A B(0, k) tal que satisfacen
(4.2). Observe que bajo esta situacion la hipotesis del teorema 4.2 se satis-
facen, as, si A B(0, k) 6= , entonces existe una sucesion B1 , B2 , . . . de
bolas de F tal que
(1) Las bolas B1 , B2 , . . . son disjuntas,
S
(2) A B(0, k) 3B .
1
 
Dado que m 3B = 3n m B , entonces por la desigualdad (4.2) tenemos
 X 
m A B(0, k) m 3B
1
X
3n m B

=
1
X Z
n 1
< 3 |f (y)| dm
1 B
Z
= 3n 1 |f (y)| dm
S
1
Z
n 1
3 |f (y)| dm
Rn

si k , se tiene que
3n
Z
m(A ) |f (y)| dm,

Rn

es decir
3n
Z
n

m {x R : M f (x) > } |f (y)| dm.

Rn

ltimo se tiene que M f L(1,) .


De esto u
Teorema 4.7 (Diferenciacion de Lebesgue). Sea f L1 (Rn ), entonces
Z
1
lm  |f (y)| dm = f (x) [m] c.t.p.
r0 m B(x, r)
B(x,r)
4.3 Funcion Maximal de Hardy-Littlewood 143

Demostracion. Sea f L1 (Rn ), entonces podemos hallar una funcion g con-


tinua tal que g L1 (Rn ) y para  > 0 (dado)
Z
|f (x) g(x)| dm < .
Rn

Ahora, observe que



Z
1
|g(y)| dm g(x)


m B(x, r)
B(x,r)
Z
1
 |g(y) g(x)| dm < ,
m B(x, r)
B(x,r)

es decir Z
1
lm  |g(y)| dm = g(x).
r0 m B(x, r)
B(x,r)

Por otra parte, observese que



Z
1
lm sup |f (y)| dm f (x) =

r0 m B(x, r)
B(x,r)

Z
1
lm sup f (y) g(y) dm+

r0 m B(x, r)
B(x,r)

Z
 1 
g(y) dm g(x) + (g(x) f (x))

m B(x, r)
B(x,r)
M (f g)(x) + 0 + |g(x) f (x)|.

Ahora, consideremos los siguientes conjuntos


Z
1
E = x : lm sup |f (y)| dm f (x) >


r0 m B(x, r)

B(x,r)

F = {x : M (f g)(x) > }
144 Operador Maximal

H = {x : |f (x) g(x)| > }.


Note que
E F/2 H/2
y Z

m F/2 |g(x) f (x)| dm < ,
2
Rn

luego
 2
m F/2 < .

Ademas, en vista del teorema 4.6 tenemos
Z
 2c 2c
m H/2 |f (x) g(x)| dm <

Rn

de donde
  
m E m F/2 + m H/2

2(1 + c) 0 si  0.

Por lo tanto

m E = 0,
as hemos demostrado que

Z
1
lm sup |f (y)| dm f (x) = 0

r0 m B(x, r)
B(x,r)

[m]-c.t.p. Como

Z
1
0 lm inf |f (y)| dm f (x)

r0 m B(x, r)
B(x,r)

Z
1
lm sup |f (y)| dm f (x) = 0 [m] c.t.p,

r0 m B(x, r)
B(x,r)
4.3 Funcion Maximal de Hardy-Littlewood 145

entonces tenemos que



Z
1
lm inf |f (y)| dm f (x)

r0 m B(x, r)
B(x,r)

Z
1
= lm sup |f (y)| dm f (x) = 0 [m] c.t.p,

r0 m B(x, r)
B(x,r)

por lo tanto, podemos escribir



Z
1
lm |f (y)| dm f (x) = 0 [m] c.t.p

r0 m B(x, r)
B(x,r)

de aqu obtenemos
Z
1
lm  |f (y)| dm f (x) = 0 [m] c.t.p
r0 m B(x, r)
B(x,r)

que era lo que queramos demostrar.


Teorema 4.8. Sea f Lp (Rn ), 1 < p , entonces M f Lp (Rn ). Ademas
existe una constante C = C(p) tal que

kM f kLp C(p)kf kp .

Demostracion. Para p = , podemos observar que M f L (Rn ) y

kM f kL kf k .

Supongamos que 1 < p < , luego para cada Rn definamos



f (x) si |f (x)| /2
f (x) =
0 si |f (x)| < /2

entonces para todo x Rn

|f (x)| |f (x)| + /2,

as
|M f (x)| M f (x)| + /2,
146 Operador Maximal

luego  
x : M f (x) > x : M f (x) > /2 .
De esto u
ltimo obtenemos
   
m x : M f (x) > m x : M f (x) > /2 ,

En virtud del teorema 4.6


Z
  C
m x : M f (x) > |f (x)| dm

R n
Z
C
= |f (x)| dm (4.3)

{x:|f (x)|/2}

Luego por el corolario 2.72 y (4.3), resulta


Z Z 
p p1

|M f (x)| dm = p m x : M f (x) > d
Rn 0

Z Z
p2

pC
|f (x)| dm
d,
0

x:f (x)/2
2|f (x)|
Z Z
= pC |f (x)| p2 d dm,
Rn 0
p1 Z
2 pC
= |f (x)||f (x)|p1 dm,
p1
Rn
p1 Z
2 pC
= |f (x)|p dm,
p1
Rn

es decir, para p > 1, tenemos


kM f kLp C(p)kf kLp ,
1/p
2p1 pC

donde C(p) = .
p1
Las propiedades de la funcion maximal de Hardy-Littlewood, motivan la
siguiente definicion.
4.3 Funcion Maximal de Hardy-Littlewood 147

Definicion 4.9. Un operador T definido del espacio de las funciones medibles


en si mismo, se dice que es sublineal si:
1. |T (f + g)(x)| |T f (x)| + |T g(x)|,
2. |T (f )(x)| |||T f (x)|
para toda funcion medible f y g y todo escalar .
Las desigualdades en la definicion 4.9, se cumplen casi en todas partes.
Definicion 4.10. Un operador sublineal T definido en L1 (Rn ), se dice que
es de tipo debil (p, q) con 1 p y 1 q < , si existe una constante
C, tal que para cada f L1 (Rn ) y cada > 0 se cumple
 q
  C
m x : |T f (x)| > kf kLp

y que es de tipo fuerte (p, q) si
kT f kLq Ckf kLp
Nota:
a) La definicion anterior se puede extender a cualquier espacio de medida
(X, A, ).
b) C no depende de f .
c) Claramente todo operador lineal es sublineal.
Proposici on 4.11. Sea T un operador de tipo fuerte (p, q). Entonces T es
de tipo debil (p, q).
Demostracion. Por la desigualdad de Markov, con g() = q y por el teorema
4.6 tenemos
   
q q
m x : |T f (x)| > = m x : |T f (x)| >
Z
C
q |T f (x)|q dm,

Rn

as
  C
m x : |T f (x)| > q kT f kqLq

 q
C
kf kLp .

148 Operador Maximal

Teorema 4.12 (Desigualdad de Kolmogorov). Sea E un subconjunto de Rn


de medida finita, si T es de tipo debil (p, q) y 0 < r < q, entonces
Z h i1 rq
r
|T f (x)| dm Cq,r m(E) kf krLp .
E

Demostracion. Para cada R+ podemos escribir


Z Z  
|T f (x)|r dm = r r1 m x E : |T f (x)| > d
E 0
 1/q
m(E) kf kLp
Z  
=r r1 m x E : |T f (x)| > d
0
Z 
r1

+r m x E : |T f (x)| > d
 1/q
m(E) kf kLp

 1/q
m(E) kf kLp
Z
r r1 m(E) d
0
Z
Cq
 
+r r1
kf kqLp d
q
 1/q
m(E) kf kLp

 1 rq
h ir/q m(E)
= m(E) m(E) kf krLp + rC q kf krLp
qr
 rC q  1 rq
= 1+ m(E) kf krLp .
qr
Finalmente
Z
rC q 
 1 rq
r
|T f (x)| dm 1 + m(E) kf krLp ,
qr
E

as Z  1 rq
|T f (x)|r dm Cr,q m(E) kf krLp ,
E
4.3 Funcion Maximal de Hardy-Littlewood 149

rC q
donde Cr,q = 1 + .
qr

El recproco del teorema 4.12 tambien es cierto.

Corolario 4.13. Sea E un subconjunto de Rn de medida finita, tal que


Z  1 rq
r
|T f (x)| dm Cr,q m(E) kf krLp ,
E

con 0 < r < q, entonces existe C > 0 tal que


 q
  C
m x E : |T f (x)| > kf kLp .


Demostracion. Sea E = x E : |T f (x)| > para R+ , entonces

i1 rq
Z h
r
|T f (x)| dm Cr,q m {x E : |T f (x)| > kf krLp .
{x:|T f (x)|>}

Por otra parte es claro que


 Z
r

m x : |T f (x)| > = r dm
{x:|T f (x)|>}
Z
|T f (x)|r dm,
{x:|T f (x)|>}

luego

r
  h i1 rq
m x : |T f (x)| > Cr,q m {x E : |T f (x)| > kf krLp

r
 h i rq 1
m x : |T f (x)| > m {x E : |T f (x)| > Cr,q kf krLp
!q
1/r
h  ir/q Cr,q
m x : |T f (x)| > kf kLp .

150 Operador Maximal

Teorema 4.14. Sea T un operador sublineal definido en L1 (Rn ) + L (Rn ),


tal que T es de tipo debil (1, 1) y ademas satisface

kT f kL Akf kL ,

donde A es una constante positiva, entonces para f L1 (Rn ) + L (Rn ) y


> 0, se tiene que
Z
  C  
m x : |T f (x)| > m x : |f (x)| > t dt

[/A,)

Demostracion. Sea f L1 (Rn ), dado > 0 podemos escribir

f (x) = f (x) + f (x),

donde
f (x) = f (x){s:|f (s)|/A} (x)

y
f (x) = f (x){s:|f (s)|>/A} (x).

De la sublinealidad de T , vemos que

x : |T f (x)| > x : |T f/2 (x)| > x : |T f /2 (x)| > /2


  

as
   
m {x : |T f (x)| > } m {x : |T f/2 (x)| > /2}
 
+ m {x : |T f /2 (x)| > /2} .


Por otra parte, notese que kf/2 kL , entonces por hipotesis
2A

kT f (x)kL /2,

por lo tanto
 
m {x : |T f (x)| > /2} = 0.
4.3 Funcion Maximal de Hardy-Littlewood 151

luego
   
/2
m {x : |T f (x)| > } m {x : |T f (x)| > /2}
Z
C
|f /2 (x)| dm

Rn
Z
C
= |f (x)| dm


{x:|f (x)|> 2A }
Z
C  
= m x : |f (x)| > t dt

[/A,)


Observaci on 4.15. Si reemplazamos 2A  por cero, obtenemos que T es de

tipo debil (1, 1); ademas notese que m x : |f (x)| > t = 0 siempre y

cuando t kf kL , as, la integral de arriba se anula, entonces m x :

|T f (x)| > = 0,si escogemos t = A , resulta kT f kL Akf kL . De esta
manera hemos demostrado que el recproco del teorema 4.14 se cumple.

Teorema 4.16 (Interpolacion de Marcinkiewick). Sea 1 p0 p1 < .


Supongamos que T es un operador sublineal definido en Lp0 (Rn )+Lp1 (Rn ), el
cual es simultaneamente de tipo debil (p0 , p0 ) y de tipo debil (p1 , p1 ). Entonces
T es de tipo fuerte (p, p) con p0 < p < p1 .

Demostracion. Sea f Lp (Rn ); para cada > 0 podemos escribir

f (x) = f (x) + f (x),

donde
f (x) = f (x){x:|f (x)|} (x)

y
f (x) = f (x){x:|f (x)|>} (x).

Consideremos solo el caso p1 < . Ahora, queremos demostrar que f


152 Operador Maximal

Lp0 (Rn ) y f Lp1 (Rn ), para ello consideremos


Z Z
p0 p0 f (x) p0
|f (x)| dm = | | dm

Rn {x:|f (x)|>}
Z
p0 f (x) p
< | | dm

{x:|f (x)|>}
Z
p0 p
< |f (x)|p dm
Rn
< p0 p
kf kpLp ,

as f Lp0 (Rn ). Por un argumento similar al dado en las lneas de arriba


podemos demostrar que f Lp1 (Rn ). En virtud de la sublinealidad de T se
sigue que

x : |T f (x)| > x : |T f/2 (x)| > /2 x : |T f /2 (x)| > /2 .


  

Por otra parte, gracias al corolario 6.6.1 resulta


Z
p
kT f kLp = |T f (x)|p dm
Rn
Z  
=p p1 m x : |T f (x)| > d
0
Z 
p1

p m x : |T f/2 (x)| > /2 d
0
Z  
+p p1 m x : |T f /2 (x)| > /2 d
0


Z p0 Z
C
p p1 |f (x)|p0 dm d
p 0
0 Rn
Z

p1 Z
C
+p p1 |f (x)|p1 dm d
p 1
0 n
R
Z Z
p pp0 1 C p0 |f (x)|p0 dm d

0 {x:|f (x)|>/2}
4.3 Funcion Maximal de Hardy-Littlewood 153


Z
C p1
Z
+p p1 |f (x)|p1 dm d,

p1

0 {x:|f (x)|/2}

Por el teorema de Fubini

2|f
Z (x)|

Z
kT f kpLp pC p0 |f (x)|p0 pp0 1 d dm
Rn 0

Z Z
+ pC p1 p
|f (x)| 1 pp1 1 d dm

Rn 2|f (x)|
pp0 p0 Z
2 pC
= |f (x)|p0 |f (x)|pp0 dm
p p0
Rn
pp1 p1 Z
2 pC
+ |f (x)|p1 |f (x)|pp1 dm,
p1 p
Rn

esto es
2pp0 C p0 2pp1 C p1
 Z
kT f kpLp p + |f (x)|p dm
p p0 p1 p
Rn

as
kT f kpLp C(p0 , p, p1 )kf kLp ,

donde
1/p
2pp0 C p0 2pp1 C p1

C(p0 , p, p1 ) = p +
p p0 p1 p

Teorema 4.17. Si T es un operador de tipo debil (1, 1) y B Rn tal que


m(B) < , entonces
Z Z
|T f (x)| dm m(B) + C |f (x)| log+ |f (x)| dm
B Rn

donde log+ t = max(log t, 0) y C es una constante independiente de f .


154 Operador Maximal

Demostracion. En virtud del corolario 6.6.1 con p = 1 resulta


Z Z 
 
|T f (x)| dm = m x B : |T f (x)| > d
B 0
Z1  
= m x B : |T f (x)| > d
0
Z 
 
+ m x B : |T f (x)| > d
1

Z Z
C
m(B) + |f (x)| dm d



1 {xB:|f (x)|>}
|f
Z(x)|

Z
d
m(B) + C |f (x)| dm

Rn 1
Z
= m(B) + C |f (x)| log+ |f (x)| dm.
Rn

Teorema 4.18 (Cotlar). Supongase que S y T son operadores sublineales


 y
n
que T esta mayorado
por S en el siguiente sentido: Si C(x, r) = y R :
r |x y| 2r para cada x Rn y f L1 (Rn ) le corresponde 0 < re < ,
tal que T f (x) nf |Sf (y)|. Entonces si S es de tipo debil (p, p) para
yC(x,e
r)
alg
un p > 0, T tambien es de tipo debil (p, p).
Demostracion. Sea 0 < q < p, entonces
|T f (x)|q nf |Sf (y)|q ,
yC(x,e
r)

luego
Z
q 1
|T f (x)|  |Sf (y)|q dm
m C(x, re)
C(x,e
r)

m B(x, 4er)
Z
  |Sf (y)|q dm (4.4)
m C(x, re) m C(x, 4e
r)
B(x,4e
r)
4.3 Funcion Maximal de Hardy-Littlewood 155

Donde B(x, re) denota la bola de centro en x y radio re. Entonces por (4.4)

|T f (x)|q CM |Sf |q (x)




Con C independiente de x y f .
Note que en virtud del teorema 4.6 M es de tipo debil (1, 1) y por el
q n
teorema 4.8 M |Sf | L (R ) y ademas

kM |Sf |q kL Ck|Sf |q kL .


Ahora, invocando al teorema 4.14 para obtener

Z

q q
 C 
1/q

m x : M (|Sf | (x)) > /2 q m x : |Sf (x)| > t dt

[Cq ,)
Z
C
q kf kpLp tp/q dt

[Cq ,)

= Cp kf kpLp .

As   kf k p
 Lp
m x : |T f (x)| > C .

Ejercicios

1. Sea (X, A, ) un espacio de medida finita. Demuestre que el dual de


L1 () es L ().

2. Si f 0 es una funcion no decreciente en (0, ) y 0 < p q ,


R. Demostrar que
Z 1/q Z 1/p
q dt p dt
(t f (t)) C (t f (t))
0 t 0 t

donde C = C(p, q, ).
Rb
3. Sea f unaRfuncion decreciente en [a, b] (a 6= 0) tal que 0 < 0
f (x) dx <
a
y 0 < 0 f (x) dx < . Demostrar que
Rb !  
0
f (x) dx b
ln R a ln
0
f (x) dx a
156 Operador Maximal

lo que es equivalente a
Z b Z a
a f (x) dx b f (x) dx.
0 0

4. Sea B(x, r) Rn una bola abierta de centro x y radio r. Definamos el


conjunto de Lebesgue de f como
 Z 
1
Lf = x : lm |f (y) f (x)| dy = 0 .
r0 m (B(x, r)) B(x,r)

Demostrar que si f L1 (Rn , L, m) entonces

|f (x)| M f (x) para cada x Lf

donde M f (x) es la funcion maximal de Hardy-Littlewood.

5. Sea f L1 (Rn , L, m) tal que


Z

f dm m(E)

E

donde E es un conjunto Lebesgue-medible. Demuestre que |f | 1 c.t.p.

6. Sea f : R [0, ) definida por


(
1
x ln2 x
si x (0, 1/e)
f (x) =
0 si x
/ (0, 1/e).

Demuestre que
R
a) (0,x) f (t) dt = 1/ ln x para x (0, 1/e).
Rr
b) 0 M f (x) dx = .
Captulo 5

Convoluci
on

5.1. Resultados b
asicos

Definici
on 5.1.
Sea E R definamos (E) R como (E) = (x, y)
2
R :xy E
Proposicion 5.2. Sea T : R2 R definida por T (x, y) = x + y. Entonces
T es continua en 0.
Demostracion. En primer lugar, queremos demostrar que T (V V ) = V + V
para V R, donde
V + V = x + y : x V, y V . En efecto, sea z T (V V ) existe
(x, y) V V tal que z = T (x, y) z = x + y con x V , y V
z V + V . As T (V V ) = V + V .
En segundo lugar, queremos demostrar que T es continuo en 0. Para ello
sea U una vecindad de 0 en R y V una vecindad de 0 tal que V + V U ,
note que V V es una vecindad de 0 en R2 pero
T (V V ) = V + V U,
demostrandose as la continuidad de T en 0.
Observaci on 5.3. Una demostracion alternativa de la proposicion 5.2, con-
siste en observar que R2 como espacio normado esta dotado de la norma
k(x, y)kR2 = kxkR + kykR ,
luego, para T (x, y) = x + y, se tiene
kT (x, y) T (x0 , y0 )kR = kx x0 + y y0 kR
kx x0 kR + ky y0 kR
= k(x x0 , y y0 )kR2
p
= (x x0 )2 + (y y0 )2 < = .

157
158 Convolucion

Esto u
ltimo, nos dice que la continuidad de T en (x0 , y0 ) es uniforme.

on 5.4. Sea h : R2 R definida por h(x, y) = x y. Entonces


Proposici

i. h es continua en 0.

ii. h 1(E) = (E) E R.

iii. Si E es abierto en R, entonces (E) es abierto en R2 .

iv. Si E es cerrado en R, entonces (E) es cerrado en R2 .



 S 
S
v. En = (En )
n=1 n=1


 T 
T
vi. En = (En ).
n=1 n=1

Demostracion. (i) es una consecuencia inmediata de la proposicion 5.1.

(ii)

Sea E R y (x, y) h1 (E) h(x, y) E


xy E
(x, y) (E).

Por lo tanto h1 (E) = (E).

(iii) y (iv) se obtienen de (ii).



 S 
S
(v) (x, y) En x y En x y En para alg
un n
n=1 n=1

S
(x, y) (En ) (x, y) (En ).
n=1
De esta manera hemos demostrado que

[ 
[
En = (En ).
n=1 n=1

(vi) Lo dejamos como ejercicio.

Lema 5.5. Si E R es un conjunto Lebesgue medible, entonces (E) es un


conjunto medible en el espacio producto.
5.1 Resultados basicos 159

Demostracion. Supongamos que E es un conjunto acotado, entonces


m(E) < . El lema es cierto.
Si E es un conjunto G o F . En efecto, por un resultado conocido de
teora de la medida podemos hallar un conjunto K el cual es F y un conjunto
H el cual es G tal que

KEH y m(K) = m(E) = m(H). (5.1)

Entonces m(H r K) = 0.
Es claro que (K) (E) (H).
Notese que para todo A R tenemos que

(A) (x, y) = A (x y).

Luego para cada x R tenemos


Z Z
(K) (x, y) dy = K (x y) dy
R R
Z
= K (y) dy
R
Z
= K (y) dy
R
= m(K) (5.2)

Sea C R un conjunto arbitrario, acotado en R, en virtud del teorema


de Tonelli, tenemos
Z Z Z
(K) dm m = (K) dy dx
CR C R
Z
= m(K) dx
C
= m(K)m(C). (5.3)

Similarmente, podemos demostrar que


Z
(H) dm m = m(H)m(C). (5.4)
CR

As
160 Convolucion

Z
(H) (K) dm m = 0. (5.5)
CR

Como (K) (H), entonces


(H) (K) = (H)(K)
= (H)(K) ,
luego
Z Z
(H)(K) dm m = (H)(K) dm m
CR CR

es decir
Z  
(H)(K) dm m = m m [(H) (K)] C R .
CR

Por (5.5) resulta


 
m m [(H) (K)] C R = 0.

En particular
 
m m [(H) (K)] [n, n] R = 0. n N
pero

[ 
[(H) (K)] [n, n] R = (H) (K),
n=1

de aqu, concluimos que


 
m m (H) (K) = 0.
Por otra parte, sabemos que
(E) (K) (H) (K)
en virtud de que m m es una medida completa se tiene (E) (K)
es un conjunto medible, ademas (K) es un conjunto F , por lo tanto
 
(E) = (K) (E) (K)

es un conjunto medible, con esto queda demostrado el lema 5.5


5.1 Resultados basicos 161

Corolario 5.6. Sea f una funcion medible. Definamos F : R2 R por


F (x, y) = f (x y). Entonces F es medible en R2 .

Demostracion. Sea h : R2 R definida por

h(x, y) = x y.

Note que
F (x, y) = f (h(x, y)) = f h(x, y)
Sea R, luego

(x, y) R2 : F (x, y) < = F 1 (, )




= (f h)1 (, )
= h1 f 1 (, ) .


Como f es medible, entonces f 1 (, ) es medible.


Sea F (x, y) = f (x y) y G(x, y) = g(y).
En virtud del corolario 5.6 F es medible en R2 , Ahora, observese que

(x, y) R2 : G(x, y) < = R g 1 (, ),




entonces G es medible en R2 . As, resulta medible en R2 .


Por otra parte, en virtud del teorema de Tonelli, resulta


Z Z Z
|(x, y)| dm m = |f (x y)| dx |g(y)| dy
RR R R

Z Z
= |f (x)| dx |g(y)| dy
R R
Z
= kf k1 |g(y)| dy
R
= kf k1 kgk1 < ,

si E = f 1 (, ), entonces E es medible, as

(x, y) R2 : F (x, y) < = h1 (E),




es decir (x, y) R2 : F (x, y) < = (E).
Por el lema 5.5 (E) es medible, por lo tanto F es medible.
162 Convolucion


Teorema 5.7. Sean f, g L1 R, L, m . Para cada x R definamos
Z
C(x) = f (x y)g(y) dy
R

Entonces C L1 (m) y ademas kCk1 kf k1 kgk1 .

Demostracion. Primero debemos demostrar que si (x, y) = f (x, y)g(y), en-


tonces es medible en R2 . Por lo tanto LR2 . Luego, por el teorema de
Fubini, tenemos


Z Z Z
|C(x)| dx |f (x y)||g(y)| dy dx
R R
R
Z Z
= |f (x y)| dx |g(y)| dy
R
R
Z Z
= |f (x)| dx |g(y)| dy
R R
= kf k1 kgk1 .

As

kCk1 kf k1 kgk1 .

Definici
on 5.8. La funcion C definida en el teorema 5.7 recibe el nombre
de convolucion de f y g y se denota por f g, es decir
Z
(f g)(x) = f (x y)g(y) dy.
R

Note que si f, g L1 (m), entonces f g L1 (m) y

kf gk1 kf k1 kgk1 .

Teorema 5.9. Sean f, g L1 (m) y , C, entonces tenemos:

a) f g = g f (Conmutatividad)
5.1 Resultados basicos 163

b) (f g) h = f (g h) (Asociatividad)
c) f (g + h) = (f g) + (f h) (Distributividad)
Demostracion. Ejercicio.
Teorema 5.10. Sea g L1 (m) y f Lp (m) con 1 p . Entonces
kf gkp kgk1 kf kp
Demostracion. Si p = y f L , entonces
Z
|(f g)(x)| |f (x y)||g(y)| dy
R
Z
kf k |g(y)| dy
R
= kgk1 kf k .
De donde
k(f g)(x)k kgk1 kf k .
Sea 1 < p < y 1 < q < el ndice conjugado de p, es decir p1 + 1q = 1.
Ahora, note que por la desigualdad de Holder, el teorema de Tonelli y la
invariancia por traslacion de la integral, tenemos
p
Z Z
1/p 1/q
kf gkpp = |f (x y)| g(y)
 
(g(y) dy dx
RR
1/p p p/q
Z Z Z
|f (x y)|p |g(y)| dy |g(y)| dy dx

R R R

Z Z
p/q
= |f (x y)|p |g(y)| dy kgk1 dx
R R

Z Z
p/q
= kgk1 |f (x y)|p |g(y)| dy dx
R R

Z Z
p/q
= kgk1 |f (x y)|p dx |g(y)| dy
R R
p/q
= kgk1 kf kpp kgk1
p
+1
= kgk1q kf kpp .
164 Convolucion

De donde se sigue que


p
+1
kf gkpp kgk1 q
kf kpp ,

finalmente
1
+1
kf gkp kgk1q p kf kp
kgk1 kf kp .

Observacion 5.11. El teorema 5.10 juega un papel importante en la teora


de semigrupos. Por ejemplo, definamos en Lp (m) el siguiente operador
Z
1 |xy|2
Tt (f )(x) = f (y)e 4t dy
4t
R

Con f Lp (m), entonces podemos escribir


1 |x|2
Tt (f )(x) = (Gt f )(x), donde Gt (x) = e 4t
4t
Por el teorema 5.10 tenemos

kTt (f )kp = kGt f kp kGt k1 kf kp ,

pero
Z Z
1
|x|2 1 y 2
kGt k1 = e 4t dx = e dy = = 1.
4t
R R

As, finalmente kTt (f )k 1, tomando T (0) = I es el operador identidad,


se demuestra facilmente que Tt Ts (f ) = Tt+s (f ), este semigrupo se llama
semigrupo de Gauss-Weierstrass.

Observaci on 5.12. La siguiente aplicacion del teorema 5.9, es sencillamente


sorprendente, ya que podemos transformar un operador que no esta definido
por una convolucion y as poder aplicar el teorema 5.9. En efecto, sea H el
operador dado en la definicion 3.9(Operador de Hardy), es decir
Zx
1
Hf (x) = f (y) dy para 0 < x < .
x
0
5.1 Resultados basicos 165

hagamos el siguiente cambio de variable:


x = es y = et
Observe que
Zs
Hf (es ) = es f (et )et dt.

Por otra parte, note que
Z Z
p p
kf kp = |f (x)| dx = |f (es )|p es ds
0

= kes/p f (es )kpLp


Ahora, bien la ecuacion
Zs
s s
Hf (e ) = e f (et )et dt

nos conduce a
Zs
s/p s s/q
e Hf (e ) = e et/p f (et )et/q dt

Zs
st
= et/p f (et )e q dt

Zs
= et/p f (et )g(s t) dt,

donde
y/q
e si 0 < y <
g(y) =
0 si < y < 0.

Como podemos ver hemos transformado (via un cambio de variable) el ope-


rador de Hardy en un operador definido por una convolucion, en tal sentido
podemos aplicar el teorema 5.9 y as obtener
kHf kp = kes/p Hf (es )kp
kgk1 ket/p f (et )kp
= kgk1 kf kp .
166 Convolucion

Como
Z Z
kgk1 = |g(y)| dy = |g(y)| dy
0
Z
= ey/q dy
0
=q
p
= ,
p1

Finalmente
p
kHf kp kf kp .
p1

on 5.13. Si fijamos g L1 (m) y definimos


Observaci

T (f ) = f g.

Entonces podemos interpretar el teorema 5.10 de la siguiente manera, para


1 p el operador T : Lp (m) Lp (m) es un operador lineal acotado.

Teorema 5.14 (W.H. Young). Sean p, q y r n umeros reales tales que p > 1,
q > 1 y p1 + 1q 1 = 1r > 0. Sean f Lp (m) y g Lq (m). Entonces
f g Lr (m) y
kf gkr kf kp kgkq .

1 1
Demostracion. Sean a, b y c n
umeros reales tales que p
= a
+ 1b , 1
q
= 1
a
+ 1
c
y a = r. Note que

1 1 1 1 1 1 1 1
+ + = + + +
a b c a b a c a
1 1 1
= + = 1.
p q r

Ahora, escribamos

|f (x y)g(y)| = |f (x y)||g(y)|
p/a q/a
 p( p1 a1 ) q( 1q a1 )

= |f (x y)| |g(y)| |f (x y)| |g(y)| .
5.1 Resultados basicos 167

En virtud del corolario 2.24, obtenemos que


Z
|f (x y)g(y)| dy
R
1/a 1/b
Z Z
a 1 1
|f (x y)|p/a |g(y)|q/a dy |f (x y)|pb( p a ) dy
 

R R
1/c
Z
1 1
. |g(y)|qc( q a ) dy ,
R

1 1 1 1 1 1
pero = y = , as
p a b q a c
Z
|f (x y)g(y)| dy
R
1/a 1/b 1/c
Z Z Z
|f (x y)|p |g(y)|q dy |f (x y)|p dy |g(y)|q dy
R R R
1/r
Z Z
|f (x y)g(y)| dy |f (x y)|p |g(y)|q dy kf kp/b q/c
p kgkq .
R R
Definamos Z
h(x) = |f (x y)g(y)| dy,
R
entonces
Z
rp rq
|h(x)|r |f (x y)|p |g(y)|q dy kf kp b kgkq c ,
R
luego

Z Z Z
r pb r qc
|h(x)|r dx kf kp kgkq |f (x y)|p |g(y)|q dy dx
R R R
r pb r qc
kf kp kgkq kf kpp kgkqq
Z
p( rb +1) q( rc +1)
|h(x)|r dx kf kp kgkq
R
168 Convolucion

1/r
Z
p( 1b + r1 ) q( 1c + r1 )
|h(x)|r dx kf kp kgkq .
R

Note que
1 1 1 1
p( + ) = p( + ) = 1
b r b a
y
1 1 1 1
q( + ) = q( + ) = 1.
c r c a
As 1/r
Z
|h(x)|r dx kf kp kgkq ,
R

ltimo es facil ver que f g Lr (m), as


de esto u

kf gkr kf kp kgkq .

5.2. El soporte y la convoluci


on
on 5.15. El soporte de f : Rn R se define
Definici

supp(f ) = {x Rn : f (x) 6= 0}

(siempre es cerrado).
Corolario 5.16. Si x
/ supp(f ), entonces existe un entorno abierto de x
donde f se anula.
Observaci on 5.17. Brezis observo que esta nocion no es adecuada cuando
trabajamos con clases de equivalencia tales como el espacio Lp , en virtud que
con la definicion 5.15 no es cierto que si f = g c.t.p entonces supp(f ) =
supp(g).
En tal sentido, se da la siguiente definicion.
on 5.18. x
Definici / supp(f ) si y solo si existe una abierto V tal que x V
y f = 0 casi en todo punto de V .
Si f es una funcion continua en Rn no es difcil verificar que esta nueva
definicion coincide con la definicion 5.15.
Por otra parte, si f = g c.t.p. en Rn (con la nueva definicion) es claro que
supp(f ) = supp(g). As, podemos hablar del soporte de una funcion f Lp
sin preocuparnos cual representante de la clase escogemos.
5.2 El soporte y la convolucion 169

Proposicion 5.19. Si f y g tienen soporte compacto, entonces f g tiene


soporte compacto. Ademas
supp(f g) supp(f ) + supp(g).
Demostracion. Note que
Z Z
(f g)(x) = f (t)g(x t) dt = f (t)g(x t) dt
Rn supp(f )

pues si t
/ supp(f ), entonces f (t) = 0.
Analogamente si xt supp(g), entonces t xsupp(g), luego g(xt) =
0 si t
/ x supp(g). Se deduce que
Z
(f g)(x) = f (t)g(x t) dt.
supp(f )(xsupp(g))

Si (f g)(x) 6= 0, entonces supp(f ) (x supp(g)) 6= luego existe y


supp(f ) (x supp(g)). Pero como y x supp(g), entonces y = x w con
w supp(g). As pues x = y + w con y supp(f ), w supp(g).
As hemos demostrado que
{(f g)(x) 6= 0} supp(f ) + supp(g),
pero la suma de dos compactos es compacto (no vale para cerrados) por lo
tanto
supp(f g) supp(f ) + supp(g).
Teorema 5.20. Si f L1 (Rn , L, m) y k es acotada y uniformemente conti-
nua en Rn , entonces f k es acotada y uniformemente continua.
Demostracion. Veamos que f k es uniformemente continua. Si f = 0, en-
tonces f k = 0 en cuyo caso no hay nada que demostrar. Supongamos que
f 6= 0. Dado > 0, por ser k uniformemente continua encontramos un > 0
tal que si |x y| < , entonces |k(x) k(y)| < kfk1 , entonces
Z Z

|f k(x) f k(y)| =
f (t)k(x t) dt f (t)k(y t) dt
n Rn
ZR
|f (t)||k(x t) k(y t)| dt
Rn Z

< |f (t)| dt
kf k1 Rn
=
ya que |(xt)(yt)| = |xy| < . Esto muestra que f k es uniformemente
continua. Por el teorema 5.10 sabemos que
kf gk kgk1 kkk .
170 Convolucion

5.3. Convoluci
on con funciones suaves
Definicion 5.21. Para m N denotamos C m la clase de funciones f (x) con
n
x R tales que las derivadas parciales hasta el orden m incluido existen y
son continuas.
El subconjunto de funciones de clase C m con soporte compacto se denota
C0m .
Similarmente C es el conjunto de todas las funciones infinitamente di-
ferenciables y C0 el subconjunto de las de soporte compacto.
Si = (1 , 2 , . . . , n ) es un multi-ndice con i {0, 1, 2, . . .}, podemos
considerar la derivada parcial
|| f
(D f )(x) =
x1 1 x2 2 xnn
Siendo || = 1 + 2 + + n .
Teorema 5.22. Si 1 p , f Lp (Rn , L, m) y k C0m , entonces
f k C0m y ademas

D (f k)(x) = (f D k)(x)

siempre que || = 1 + 2 + + n m.
Demostracion. Primero demostremos que si k es continua con soporte com-
pacto, entonces f k es continua.

|(f k)(x + h) (f k)(x)|


Z Z

= f (t)k(x + h t) dt f (t)k(x t) dt
n Rn
ZR

= f (t)[k(x + h t) k(x t)] dt
Rn

Si u = x t, entonces t = x u, luego
Z

= f (t)[k(u + h) k(u)] du
Rn
Z 1/p Z 1/q
p q
|f (x u)| du |k(u + h) k(u)| du .
Rn Rn

Afirmamos que
Z 1/q
q
lm |k(u + h) k(u)| du = 0.
h0 Rn
5.3 Convolucion con funciones suaves 171

En efecto, como k es continua y tiene soporte compacto, entonces es unifor-


memente continua en Rn , por lo tanto dado > 0 existe un > 0 tal que
para todo u Rn , si |h| < , entonces

|k(u + h) k(u)| < .

Podemos ademas suponer que < 1. En consecuencia si |h| < ,


Z Z
|k(u + h) k(u)| du = |k(u + h) k(u)|q du
q
n
R ZI
< q du
I
= q m(I)

donde I = {x Rn : d(x, supp(k)) 1} (que es compacto y por lo tanto


tiene medida finita).
Sea k C0m (m 1) fijada i con 1 i m y sea ei el i-esimo vector de
la base canonica, entonces
(f k)(x + h ei ) (f k)(x)
 h 
k(x t + h ei ) k(x t)
Z
= f (t) dt
Rn h
Z  
k
= f (t) (x t + h ) dt
Rn xi

por el teorema del valor medio, para alg un h = ei dependiendo de x y t,


donde esta entre 0 y h.
Por lo tanto, cuando |h| 0, x k
i
(x t + h ) converge a x k
i
(x t)
uniformemente en t.
k
Como x i
tiene soporte compacto, se deduce del teorema de la conver-
gencia uniforme que la u ltima integral converge a
Z  
k
f (t) (x t) dt.
Rn xi
 
Por lo tanto, x i (f k)(x) existe y es igual a f x k
i
(x) que es continua
por lo antes demostrado. Esto demuestra el teorema para el caso m = 1, la
demostracion para cualquier m es inmediata por induccion (aplicando el caso
m = 1). Se sigue que f k C si f Lp (1 p ) y k C0 .
Ademas por el teorema 5.20 f k tiene soporte compacto.
172 Convolucion

5.4. Aproximaciones de la identidad


on 5.23. Dada una funcion k : Rn R y > 0 definamos
Definici
x
n
k (x) = k .

Por ejemplo, si k = B donde B = {x Rn : kxk 1}, entonces tenemos
que (
n , si |x| <
k (x) =
0, si |x| >
es decir, k (x) = n B (0) donde B (0) = {x Rn : |x| < }. Vemos que a
medida que hacemos peque no a , k es una funcion con un pico mas elevado
y un soporte mas peque no.
Para cualquier funcion k positiva ocurre lo mismo.
Lema 5.24. Si k L1 (Rn , L, m) y > 0, entonces
R R
1. Rn k (x) dx = Rn k(x) dx.

2. Para cada > 0 fijo


Z
|k (x)| dx 0 cuando 0.
|x|>

Demostracion. 1. Basta hacer el cambio y = x , luego


Z Z x
n
k (x) dx = k dx
Rn n
ZR
= k(y) dy.
Rn
x
(y =
es un cambio lineal de coordenadas.)

2. Fijemos un > 0 y hacemos el mismo cambio de variable, entonces


Z Z x
n
|k (x)| dx = k dx
|x|> |x|>
Z
= |k(y)| dy.
|y|>/

Pero / + cuando 0. Como k L1 (Rn , L, m) la u


ltima
integral tiende a cero a medida que 0.
5.4 Aproximaciones de la identidad 173

Observaci on 5.25. Si k 0, la primera propiedad significa que el area bajo


las graficas k y k es la misma, mientras que la segunda significa que para
pequeno el area bajo el grafico de k esta concentrada en una region sobre un
pequeno entorno del origen.
Para cualquier k L1 (Rn , L, m) consideremos
Z
(f k )(x) = f (x t)k (x) dt,
Rn

Ahora, vamos a demostrar que (f k )(x) f (x) puntualmente cuando


0, bajo hipotesis adecuadas sobre f y k.
Definicion 5.26. Una familia {k : > 0} de n
ucleos para la cual f k f
en alg
un sentido, se llama una aproximacion de la identidad.
R
Teorema 5.27. Sea f = f k donde k L1 (Rn ) y Rn k(x) dx = 1. Si
f Lp (Rn , L, m) con 1 p < , entonces kf f kp 0 cuando 0.
Demostracion. Por la parte 1 del lema anterior tenemos
Z Z
f (x) = f (x) k (t) dt = f (x)k (t) dt.
Rn Rn

Por lo tanto
Z

|f (x) f (x)| =
[f (x t) f (x)]k (t) dt
n
ZR
|f (x t) f (x)||k (t)| dt
Rn
Z
= |f (x t) f (x)||k (t)|1/p |k (t)|1/q dt
Rn

donde q es tal que p1 + 1q = 1.


Aplicando la desigualdad de Holder con exponentes p y q obtenemos que
Z 1/p Z 1/q
p
|f (x) f (x)| |f (x t) f (x)| |k (t)| dt |k (t)| dt
Rn Rn

entonces
Z Z Z  Z p/q
p p
|f (x) f (x)| dx |f (x t) f (x)| |k (t)| dt |k (t)| dt dx
Rn Rn Rn Rn
Z Z 
p
= kkk1 |f (x t) f (x)| |k (t)| dt dx.
Rn Rn
174 Convolucion

Cambiando el orden de integracion (lo cual esta justificado por el teorema


de Fubini-Tonelli, pues el integrando es no negativo) obtenemos
Z Z 
p p/q p
kf f kp kkk1 |f (x t) f (x)| |k (t)| dx dt
Rn Rn
Z Z 
p/q p
= kkk1 |k (t)| |f (x t) f (x)| dx dt
Rn Rn

R
haciendo (t) = Rn
|f (x t) f (x)|p dx = kf ( t) f ()kpp tenemos
Z
p p/q
kf f kp kkk1 |k (t)|(t) dt.
Rn

Para > 0 escribimos


Z
I = |k (t)|(t) dt = A, + B,
Rn

donde Z
A, = |k (t)|(t) dt
|t|<
y Z
B, = |k (t)|(t) dt.
|t|

Dado > 0, podemos elegir > 0 tan peque no que (t) < si |t| < , ya
que (t) 0 cuando |t| 0 pues f Lp (m)
Z
A, n |k (t)| dt nkkk1 ,
|t|<

para todo > 0. Por otra parte, por la desigualdad de Minkowski es una
funcion acotada, de hecho
Z 1/p
1/p p
[(t)] = |f (x t) f (x)| dx
Rn
Z 1/p Z 1/p
p p
|f (x t)| dx + |f (x)| dx
Rn Rn
= 2kf kp ,

de modo que
kk (2kf kp )p = C
5.4 Aproximaciones de la identidad 175

luego
Z
B, = |k (t)|(t) dt
|t|
Z
C |k (t)| dt 0.
|t|

Cuando 0. Por lo tanto


I 0 cuando 0
y esto demuestra el teorema.
Corolario 5.28. Para 1 p < , C0 (Rn ) es denso en Lp (Rn , L, m).
Demostracion. Sea f Lp (m) (1 p < ). Dado > 0 escribimos f = g+h
R g tiene soporte compacto y khkp < . Elijamosun nn
donde ucleo k C0 tal
que Rn k(x) dx = 1 y sea g = g k , entonces g C0 (R ) y kg gkp 0
cuando 0.
Por los resultados anteriores, elijamos tal que kg gkp < , entonces
kf g kp khkp + kg g kp < 2.
Esto demuestra el corolario.
R
Teorema 5.29. Sea f = f k donde f L (Rn , L, m) y Rn k(x) dx = 1,
entonces f f en cada punto de continuidad de f cuando 0.
Demostracion. Tenemos que
Z Z

|f (x) f (x)| =
f (x t)k (t) dt f (x)k (t) dt
n Rn
ZR
|f (x t) f (x)||k (t)| dt.
Rn

Si f es continua en x, dado > 0, existe > 0 tal que si |t| < , entonces
|f (x t) f (x)| < y obtenemos
Z Z
|f (x) f (x)| |f (x t) f (x)||k (t)| dt + |k (t)| dt
|t| |t|<
Z
2kf k |t| |k (t)| dt + kkk1 .
R
Como |t|
|k (t)| dt 0 cuando 0 para un fijo, se sigue que

|f f (x)| 0 cuando 0
176 Convolucion

Ejercicios

1. Sean

a) f : R R dada por f = [1,1] .


b) f : R2 R dada por f = B(0,1) .

En cada caso hallar (f f )(x).

2. Supongase que f Lp ((Rn , L, m)) y g Lq ((Rn , L, m)) con p1 + 1q = 1.


demostrar que para cada > 0 existe R > 0 tal que

(f g)(x) < |x| > R.

3. Sean
( )
X
l1 (L )(Rn ) = f Lloc, : kf k,1 = kf kL (Qk ) <
kZn
 
n
l (L1 )(R ) = f Lloc,1 : kf k1, = sup kf kL1 (Qk ) <
kZn

donde Qk = Q0 + k y Q0 = [ 12 , 21 ]n . Si f l1 (L )(Rn ) y g
L (L1 )(Rn ) demostrar que f g L (Rn , L, m) y

kf gk 2n kf k,1 kgk1, .

4. Sea : R R una funcion definida por


( 1

(x) = e x2 1 si |x| < 1,


0 si |x| 1.

a) Demostrar que es de clase C con supp = [1, 1],


xa

b) Para > 0 y a R, demuestre que la funcion f (x) =
es de
clase C con supp f = [a , a + ].

5. Sea [a, b] R y > 0 tal que a + < b donde esta definida como
Rb
en el problema 1. Definamos h : R R por h(x) = a tx


dt para
todo x R. Demostrar que:

a) supp h [a , a + ]
b) h(x) = c (funcion constante) para todo x [a + , b ]
5.4 Aproximaciones de la identidad 177

Rb n
c) h es de clase C y h(n) (x) = tx

a xn

dt para todo x R y

d) La funcion f = h/c de clase C satisface 0 R f (x) 1 para todo
x R, f (x) = 1 para todo x [a + , b ] y R |(a,b) f |dm < 4.

6. Sea f : R R una funcion integrable con respecto a la medida de



Lebesgue.
R Dado > 0, demuestre que existe una funcion g de clase C
tal que R |f g|dm < .

7. Consideremos el espacio vectorial de funciones


 Z 

E = f : R R | f es de clase C y f dm = 0
R

Demostrar que para cada 1 < p < , el espacio vectorial E es denso


en Lp (R). Sera E denso en L1 (R)?
178 Convolucion
Captulo 6

Potenciales

6.1. Potencial de Riesz


Las desigualdades que involucran el potencial de Riesz proveen una impor-
tante herramienta que permite estimar funciones en terminos de la norma de
sus derivadas. En lo sucesivo utilizaremos la transformada de Fourier. Sea
f L1 (Rn ); definimos fb por
Z
n
f () = (2)
b eix f (x) dx, Rn .
Rn

La funcion fb, la llamaremos la transformada de Fourier de la funcion f ,


algunas veces la denotaremos por F(f ). Consideremos el laplaciano de f ,
esto es
n
X 2f
f = .
k=1
x2k

Ahora, tomemos la transformada de Fourier de (f ); es decir

n
!
2f
Xd
(f
d )(x) = 2
(x)
k=1
x k
n
X
= (i2xk )2 fb(x)
k=1
Xn
2
= 4 x2k fb(x)
k=1

= 4 |x|2 fb(x).
2

179
180 Potenciales

Ahora, deseamos reemplazar el exponente 2 en |x|2 por un exponente


general y as definir, al menos formalmente, el laplaciano fraccional por

(() 2 f )(x) = (2|x|) fb(x).
d

Con esta idea en mente, junto con el hecho que || para 0 < < n es
localmente integrable, podemos ver al laplaciano fraccional como un elemento
de S 0 (S 0 el dual del espacio de Schwartz S ). Ahora, definamos el siguiente
operador
I : S 0 S 0 ,
donde

I (f ) = () 2 f = F 1 (|| fb()) (6.1)
para 0 < < n, donde F 1 representa la transformada inversa de Fourier
de || (en el sentido de distribuciones). Usando la formula
Z
1 ||2
|| = 2 1 e d,
C 0

donde C = 2 ( 2 ), el Teorema de Fubini nos permite demostrar que
Z
1 f (y)
I f (x) = dy, (6.2)
() Rn |x y|n

donde n
2 2 ( 2 )
() = .
( n2
)
En efecto, dado que || fb(x) es integrable obtenemos

Z  Z 
1 2ix 1
1
||2
F (|| fb(x))(x) = e 2 e d fb() d
C 0
Rn


Z Z
1 ||2
e2ix e fb() d 2 1 d
=
C 0
Rn


Z Z Z
1 ||2
e2ix e e2iy dy d 2 1 d
=
C 0
Rn Rn
6.1 Potencial de Riesz 181


Z Z Z
1 ||2
f (y) e2i(xy) e d dy 2 1 d
=
C 0
Rn Rn


Z Z
1 |xy|2
f (y) n2 e dy 2 1 d
=
C 0
Rn

Z  Z 
1
|xy|2
1
= f (y) e 2 d dy
C 0
Rn

Z
= Cn |x y|n f (y) dy
Rn

de esta forma hemos demostrado (6.2).

Observaci on 6.1. La verificacion de (6.2) la hicimos de manera formal; es


decir, supusimos que (6.2) estaba bien definido. Sin embargo, para ver que
(6.2) esta bien definido, necesitamos estudiar la convergencia de la integral.
Dado que el soporte de f es acotado, no tendremos problemas de integrabili-
dad en el infinito. A pesar que el nucleo del operador tiene una singularidad
en la diagonal x = y, se tiene que

|f (y)|
Z Z
X dy
dy kf k L
B(x,1) |x y|n k=0 2k1 |xy|<2k |x y|n

X m(B(x, 2k+1 ))
kf kL
k=0
2(k+1)(n)

X
22n kf kL 2k < .
k=0

De esta manera, hemos demostrado que la integral que define al operador I


es localmente uniformemente integrable; es decir I L1loc,u .
A (6.2), lo llamaremos el potencial de Riesz de f . Con la ayuda de (6.1)
podemos ver que
f (x)|() = || ().
|Id
182 Potenciales

Lema 6.2. Sea 0 < < n, entonces


(i) La transformada de Fourier de la funcion |x|n es la funcion
()(2) |x| ,
en el sentido siguiente
Z Z
|x| (x) dx = ()(2) |x| (x)
n d dx,
Rn Rn

para S .
(ii) La identidad
b
(Id
f )(x) = (2|x|) f (x),
se obtiene en el siguiente sentido
Z Z
I (f )(x)g(x) dx = fb(x)(2|x|) gb(x) dx,
Rn Rn

para f, g S .

Demostracion. (I) Ver [?].


(II) En virtud de (I) tenemos

(2)
Z Z
f (x y)|y|n
dy = |y| fb(x y)dy,
()
Rn Rn

pero
d+ x)() = e2ixy fb(y).
y)() = f (y
f (xd
Entonces
(2)
Z Z
n
f (x y)|y| dy = |y| fb(y)e2ixy dy
()
Rn Rn


Z Z
(2) f (x y)|y|n dy g(x)dx =
()
Rn Rn

Z Z
|y| fb(y)e2ixy dy g(x)dx,
Rn Rn
6.1 Potencial de Riesz 183

luego

Z Z Z
(2) I f (x)g(x) dx = |y| fb(y) g(x)e2ixy dx dy
Rn Rn Rn
Z Z
(2) I f (x)g(x) dx = |y| fb(y)b
g (y)dy.
Rn Rn

Dado que b
g (y) = gb(y), entonces
Z Z
(2)
I f (x)g(x) dx = |y| fb(y)b
g (y)dy
Rn Rn
Z Z
I f (x)g(x) dx = (2|x|) fb(x)b
g (x)dy.
Rn Rn

Lema 6.3. Si f S , entonces


(i) I (I f ) = I+ (f ) donde > 0, > 0, + < n.
(ii) (I f ) = I (f ) = I2 (f ) con n > 3, n 2.

Demostracion. (i) Al aplicar la transformada de Fourier obtenemos


(I )]() = (2|x|) (I
[Id d )()

= (2|x|) (2|x|) fb()


= (2|x|)(+) fb()
= (Id
+ )().

As, tomando la transformada inversa de Fourier, se tiene


I (I f ) = I+ (f ).

(ii)
n
!
2 I f (x)
X d
((I
d f ))() = ()
k=1
x2k
n
X
= 4 2 x2k (Id
f )()
k=1

= (2|x|)2 (2|x|) fb()


= (I2
df )(). (6.3)
184 Potenciales

Por otra parte,

(f ))() = (2|x|) (f
(Id d)()

= (2|x|) (2|x|)2 fb()


= (2|x|)2 fb()
= (I2
df )(). (6.4)

De (6.3) y (6.4), se obtiene el resultado.

En lo sucesivo, sera de utilidad la siguiente definicion.

on 6.4. Sea S , el funcional delta se define por la ecuacion


Definici

() = h, i = (0),

Teorema 6.5 (Identidad de aproximacion). Sea f una funcion puntualmente


continua, tal que
Z Z
|f (x)| dx < y f (x) dx = 1.

Si fa (x) = af (ax), entonces

lm hfa , i = h, i S .
a

Demostracion. Por supuesto, la convergencia es en el sentido de distribucion.


La idea es que cuando a , fa (x) se transforme en un pulso angosto
(estrecho), con su altura creciendo en la misma proporcion que su ancho. Sea
cualquier funcion de prueba, para eso, escojamos > 0; tal que |(x)
(0)| <  para |x| < , escojamos t de modo que

Z t Z
|f (x)| dx + |f (x)| dx < .
t
6.1 Potencial de Riesz 185

R
Finalmente, sea a < t , M =
|f (x)| dx y K = max |(x)|, entonces
x

|hfa (), i h, i|
= |haf (a), i h, i|
Z h u i

= f (u) (0) du
a


Z
t h u i Z h u i
= f (u) (0) du + f (u) (0) du

t a a
{|u>t|}
Z  u 
|f (u)| du max (0)

|ut| a

Z  u 
+ |f (u)| du max (0) .

|u/t| a
|u|>t


Ahora, dado que |u| t y a > t , entonces ua < y as
Z  u 
|f (u)| du max (0) < M ,

|ut| a

ademas
Z  u 
|f (u)| du max (0) < 2K,

a

|ut|
|u|>t

luego
t
|hfa (), i h, i| < (M + 2K) si a> ,

lo que significa que
lm hfa , i = h, i.
a

Ejercicios

1. Demuestre que si (0) = h, i


R = () entonces h, i no se puede
escribir como (0) = h, i = (x)(x) dx, siempre que sea lo-
calmente integrable.
186 Potenciales

6.2. Potenciales en Lp
En la Seccion 2.1, consideramos el potencial de Riesz desde un punto de vista
formal. En particular operamos con funciones suaves que se comportan bien
en el infinito. Dado que el potencial de Riesz es un operador integrable, es
natural estudiar su accion en los espacios Lp (Rn ). Por esta razon, formulamos
el siguiente problema; dado con 0 < < n para cuales valores de p y q el
operador
I : Lp (Rn ) Lq (Rn )
es acotado? En otras palabras, cuando se tiene la siguiente desigualdad

kI (f )kq ckf kp ? (6.5)

Para resolver esta pregunta, consideremos el operador dilatacion definido


por
(f )(x) = f (x), > 0.
Por un lado, tenemos
Z
k (f )kp = |f (x)|p (6.6)
Rn
n
= p kf kp ,

por otra parte, obtenemos

1 (I ( ))(x) = I( )(1 x)

Z
1 (f )(y)
= dy
() |1 x y|n
Rn

= I f (x).

As

kI (f )kq = k1 (I ( (f )))kq
n
= q kI ( f )kq . (6.7)

Si
kI ( f )kq ckf kq ,
6.2 Potenciales en Lp 187

entonces por (6.5) y (6.6), se tiene que


n n
+ q kI ( f )kq q ck (f )kp .

As n n
kI ( f )kq ( p q ) ck (f )kp ,
donde si np nq 6= 0, entonces 0 o .
Si obtenemos la desigualdad deseada, necesariamente tendremos
1 1
= .
q p n

Veremos que esta condicion es suficiente para dos casos excepcionales. Estos
n
suceden cuando p = 1, el cual implica que q = n y cuando q = , entonces
n
p = . Consideremos la funcion
(
1, si |x| < 21 ,
(x) =
0, si |x| 21 .

Note que Z
(x) dx = 1.
Rn

Ahora, definamos la siguiente sucesion

k (x) = 2k n (kx),

observe que  
1
supp k = B 0,
2k
y Z
k (x) dx = 1.
Rn

Ahora, consideremos la siguiente situacion; si (6.5) fuese valido, se tiene

kk k n
n ckk k1 .

Puesto que kk k1 = 1. se tendra que


Z
n n
|I (x)| n dx c n
Rn
188 Potenciales

n
Z Z n
(y) n
dy
|x y|n
dx C n .
n
n
R R
Si
1
g(y) = ,
|x y|n
notemos que g S , entonces con la ayuda del Teorema 6.5, obtenemos
Z Z
lm g(y)k (y)dy = g(y)(y)dy = g(0).
k
Rn Rn

El Lema de Fatou nos permite obtener


n n
Z Z n Z Z n

g(y)(y) dy
dx l
m inf g(y)k (y) dy
dx

Rn Rn Rn Rn
Z
n n
|g(0)| n dx C n
Rn
Z
dx n

n
C n .
|x|
Rn
Esto ultimo nos conduce a una contradiccion. El segundo caso atpico ocurre
cuando q = , de nuevo la desigualdad del tipo (6.5), no se puede obtener.
Una razon inmediata es que este es el caso dual del caso p = 1; sin embargo,
este caso (q = ) lo podemos ver directamente considerando la siguiente
funcion, sea
  n (1+)
|x| log |x|
1
si |x| 21
f (x) =

si |x| > 21 .

0
con n (1 + ) donde  es un numero positivo suficientemente peque
no. Ahora
n
queremos demostrar que f L (R ), en efecto
n

Z Z  (1+)
n
n 1
|f (x)| dx = |x| log dx
|x|
Rn |x| 21
Z 1
2 dr
= n (1+)
0 r log 1r
< .
6.2 Potenciales en Lp 189

Sin embargo, I (f ) es esencialmente no acotado cerca del origen. Esto es


Z   n (1+)
1 n 1
I (f )(0) = |y| log dy
() |x|
|y| 12
Z 1
2 dr
= C() 
1 n (1+)
0 r log r
= .

Ahora bien, si tomamos un subconjunto Rn ( < n), tal que 0 < m() <
, obtenemos el siguiente resultado.

Lema 6.6. Sea Rn un conjunto medible con 0 < m() < y 0 < <
n, entonces existe una constante C > 0, tal que

kI( )k Ck k n .

Demostracion. Podemos suponer que x = 0, y hallar un R > 0, para


el cual |B(0, R)| = ||; denotemos B = B(0, R); luego, si y \ B,
entonces |y| > R como n < 0 :
Z
|y|n dy Rn | \ B|
\B

= Rn |B \ |
Z Z
n
|y| dy |y|n dy. (6.8)
\B B\

Por otra parte


Z Z Z
n n
|y| dy = |y| dy + |y|n dy,
\B B

y por (6.8), se tiene que


Z Z Z
n n
|y| dy |y| dy + |y|n dy,
B\ B

de donde
Z Z
n
|y| dy |y|n dy.
B
190 Potenciales

Como
Z Z R
n
|y| dy = n |B(0, 1)| rn1 rn dr
B 0
n
= |B(0, 1)|R

= R |B(0, 1)||B(0, 1)|/n |B(0, 1)|/n
= |B(0, 1)|1/n (Rn |B(0, 1)|)/n
= |B(0, 1)|1/n |B(0, R)|/n .

Finalmente obtenemos
Z
|y|n dy |B(0, 1)|1/n |B(0, R)|/n ;

es decir
kI ( )k Ck k n .

on 6.7. I f es semicontinua cuando f 0.


Proposici

Demostracion. Si xn x, entonces |xn y|n f (y) |x y|n f (y);


por el Lema de Fatou obtenemos
Z Z
n
lmnf |xn y| f (y)dy lmnf |xn y|n f (y)dy
Rn n
Z ZR
|x y|n f (y)dy lmnf |xn y|n f (y)dy
Rn Rn
I f (x) lmnf I f (xn ).

Lema 6.8. Sea la medida signada de Radon en Rn , x Rn y < n,


entonces Z Z
d
n
= (n ) rn1 (B(x, r))dr,
Rn |x y| 0

siempre que sea no negativa, 0


Z
d||(y)
< .
Rn |x y|n
6.2 Potenciales en Lp 191

Demostracion. En virtud que toda medida de Radon es una medida de Le-


besgue y gracias al corolario 2.72, podemos escribir
Z Z  
d(y) 1
n
= {x : > } d
Rn |x y| 0 |x y|n
Z  
1  n
1
= {x : |x y| < d
0
Z  
1  n
1 
= B x, d,
0

haciendo el siguiente cambio de variable


1
  n
1
= r,

resulta
1
!!
Z   n Z
1
rn1 B(x, r) dr.

B x, d = (n )
0 0

Finalmente
Z Z
d(y)
rn1 B(x, r) dr.

= (n )
Rn |x y|n 0

Teorema 6.9. Sea 1 p < . Sea un conjunto de medida finita y sea


f Lp (), entonces
kI f kLp () Ckf kLp () (6.9)
donde C = |B(0, 1)|1/n .

Demostracion. Por el Lema 6.6 con = 1, tenemos


Z
|x y|n dy C||/n .

Entonces, si p 1, utilizando la desigualdad de Holder, obtenemos


Z Z 1/p Z 1/q
n p n n
|f (y)||x y| dy |f (y)| |x y| dy |x y| dy

192 Potenciales

Z Z 1/p
n 11/p p n
|f (y)||x y| dy C |f (y)| |x y| dy

Z p Z 
n p1 p n
|f (y)||x y| dy C |f (y)| |x y| dy

Z Z Z 
p p1 p n
|I f (x)| dx C |f (y)| |x y| dy dx

Z Z
p1 p
|x y|n dx

=C |f (y)| dy
Z

C p |f (y)|p dy.

As,

kI f kLp() Ckf kLp() .

Ahora, estimaremos la norma del potencial de Riesz de manera mas general.

Teorema 6.10. Si 0 < < n, > 0 y > 0, entonces para x Rn

|f (y)|
Z
n
dy C M f (x),
B(x,) |x y|
donde C = n |B(0, 1)|.

Demostracion. para x Rn y > 0 empleamos el Lema 6.8, as obtenemos


|f (y)|
Z
n
dy
B(x,) |x y|

Z Z 
dr
= (n ) |f (y)|dy
0 B(x,r)B(x,) rn+1
 Z Z 
dr dr
(n ) |B(0, 1)| M f (x)rn + |B(0, 1)| M f (x) n
0 rn+1 rn+1
n
= |B(0, 1)|M f (x) .

6.2 Potenciales en Lp 193

Teorema 6.11 (Desigualdad de Hedberg). Sea 0 < < n y f Lp (Rn ),


entonces para 1 p < n se tiene la siguiente desigualdad
p p
|I f (x)| kf kpn (M f (x))1 n .
Demostracion. Para x Rn y > 0 se tiene que
|f (y)| |f (y)|
Z Z
|I f (x)| dy + dy
|x y|n |x y|n
B(x,) Rn \B(x,)

por el Teorema 6.10, obtenemos




|f (y)|
Z Z Z
dr
dy = (n ) |f (y)|dy

|x y|n rn+1

0
B(x,) B(x,r)B(x,)
 Z 
n dr
(n ) m(B(0, 1)) M f (x)r n+1
0 r
Z
dr
+ m(B(0, 1)) M f (x) n n+1
0 r
n
= m(B(0, 1))M f (x) . (6.10)

Ahora bien, para > 0 la desigualdad de Holder implica que
1q
|f (y)|
Z Z
dy kf kp |x y|(n)q dy

n
Rn \B(x,) |x y|
B(x,)
 Z  1q
n1q(n)
= kf kp nm(B(0, 1)) r dr

nm(B(0, 1)) n
kf kp p . (6.11)
n q(n )
Finalmente de (6.10) y (6.11), se tiene que

Z
|f (y)| nm(B(0, 1))  n

|x y|n dy n q(n ) M f (x) + kf kp . (6.12)
p

n
R
  np
Si escogemos = Mkffk(x)
p
, entonces (6.12) se transforma en
nm(B(0, 1)) p p
|I f (x)| (M f (x))1 n kf kpn .
n q(n )
194 Potenciales

Teorema 6.12. Sea 0 < < n.


n np
(a) Si 1 < p <
yq= np
, entonces

I : Lp Lq ;

es decir, kI f kq Ckf kp .
n
(b) Si q = n
, entonces
I : L1 L(1, n
n
)

esto es, n
  n
kf k1
m({x : I f (x) > }) c .

Demostracion. (a) Observe que la desigualdad de Hedberg junto con el Teo-


rema 4.8, implica que
1q 1q
Z p
Z
p
|I f (x)|q dx Ckf kpn |M f (x)|q(1 n ) dx
Rn Rn
p1
p
Z
= Ckf kpn |M f (x)|p dx
Rn
Ckf kp

(b) Para el caso p = 1 la desigualdad de Hedberg se transforma en



|I f (x)| kf k1n (M f (x))1 n . (6.13)

Observe que por el Teorema 4.6 y (6.13) se tiene


n
! n

m({x : I f (x) > }) m x : M f (x) >

ckf k1n
  n
kf k1 n
c ,

as  n
 n
kf k1
m({x : I f (x) > }) c .

6.2 Potenciales en Lp 195

Ejercicios

1. La integral de Poisson y la integral de Gauss-Weierstrass vienen dadas


respectivamente por:
R
a) Pt (x) = Rn P (y, t)(x y) dy t > 0.
R
b) Wt (x) = Rn W (y, t)(x y) dy.

Donde
Cn t
P (y, t) =
(|y|2 + t2 )(n+1)/2
 
(n+1)/2 n+1
Cn =
2
y
|x|2
W (x, t) = (4t)n/2 e 4t .
Sea Lp (Rn , L, m) con 1 < p < n/. Demostrar que
Z
1
I (x) = 1 t1 Pt (x) dt
2 () 0
Z
1
= t/21 Wt (x) dt.
(/2) 0
196 Potenciales
Captulo 7

Espacios Lp() con 0 < p < 1

7.1. Nociones b
asicas
Teorema 7.1. Lp () con 0 < p < 1 es un espacio completo

Demostracion. Sea {fn }nN una sucesion de Cauchy en Lp () con 0 < p < 1,
entonces Z
d(fn , fm ) = lm |fn (x) fm (x)|p d = 0,
n X
m
de aqu se sigue que para cada n umero natural k existe un menor n
umero
natural nk tal que
Z
1
|fn (x) fm (x)|p d < k para m nk , n nk ,
X 3
en particular, podemos tomar n = nk+1 y m = nk de modo que
Z
1
|fnk +1 (x) fnk (x)|p d < k para k = 1, 2, 3, . . . (7.1)
X 3
Definamos  
1
Ek = x X : |fnk +1 fnk (x)| > k/p ,
2
entonces,
Z Z  p
p 1 1
|fnk +1 (x) fnk (x)| d d = (Ek )
Ek Ek 2k/p 2k
luego por (7.1) se tiene que
1 1
k
(Ek ) < k
2 3
197
198 Espacios Lp () con 0 < p < 1

es decir,
 k
2
(Ek ) < .
3
Consideremos ahora
 
[ [ 1
Ek = x X : |fnk +1 fnk (x)| > k/p . (7.2)
k=N k=n
2

Si x no pertenece a este conjunto es claro que


1
|fnN +1 fnN (x)| ,
2N/p
1
|fnN +2 fnN +1 (x)| ,
2(N +1)/p
..
.
de modo que la serie

X
|fnk +1 (x) fnk (x)| (7.3)
k=1

converge, pero la medida de (7.2) es


 k  N
X X 2 2
(Ek ) < =3
k=N k=N
3 3

la cual tiende a cero cuando N tiende a infinito. Esto muestra que el conjunto
de todas la x para las cuales (7.3) no converge es de medida cero, es decir,
la serie (7.3) converge casi en todas partes, entonces tambien lo hace la serie

X
(fnk +1 (x) fnk (x))
k=1

ya que una serie absolutamente convergente es convergente. Escribiendo


k1
X
fnk (x) = fn1 (x) + (fnj +1 (x) fnj (x))
j=1

se sigue que

X
lm fnk (x) = fn1 (x) + (fnj +1 (x) fnj (x))
k
j=1

existe, denotando este lmite por f (x) vemos que


7.1 Nociones basicas 199

lm fnk = f (x) casi en todas partes.


k

A demostrar que {fn }nN converge a f . Para ello observamos que por el Lema
de Fatou
Z Z
p
|fnk (x) f (x)| d = lm inf |fnk (x) fnj (x)|p d
X X j Z
lm inf |fnk (x) fnj (x)|p d
j X
p
<  /2.
Finalmente,
Z Z Z
p p
|fnk (x) f (x)| d |fn (x) fnk (x)| d + |fnk (x) f (x)|p d
X X X
< p
as, (fn f ) Lp () y f = (f fn ) + fn Lp ().
Proposici
on 7.2. Sea
 Z 1 
Lp = f : [0, 1] R : f es L medible y p
|f (t)| dt < .
0

Entonces,
R1
a) d(f, g) = 0
|f (t) g(t)|p dt es una metrica en Lp .
b) d es invariante por traslacion y as (Lp , d) es un espacio vectorial topologi-
co.
c) Si A es convexo abierto no vaco, entonces A = Lp .
d) f : Lp Y es lineal e Y es localmente convexo. Entonces f es constante.
e) (Lp ) = {0}.

Demostracion. |f g|p 0 para toda f, g Lp as,


Z 1
|f g|p dt 0,
0

entonces d(f, g) 0.

Si d(f, g) = 0, entonces Z 1
|f g|p dt = 0,
0
200 Espacios Lp () con 0 < p < 1

luego |f g|p = 0 c.t.p, lo que implica que |f g| = 0 c.t.p, de donde f = g


c.t.p.

Por otra parte si f = g para f, g Lp , entonces f g = 0 de donde


|f g|p = 0 y as,
Z 1
|f g|p dt = 0,
0

es decir, d(f, g) = 0.

Para f y g en Lp notemos que


Z 1 Z 1
p
d(f, g) = |f g| dt = |g f |p dt = d(g, f )
0 0

es decir,
d(f, g) = d(g, f ).

Finalmente, si a y b son reales positivos, entonces a + b b y a + b a,


como 0 < p < 1, entonces p 1 < 0, luego

(a + b)p1 bp1

y
(a + b)p1 ap1
de donde,
b(a + b)p1 bp
y
a(a + b)p1 ap ,
sumando

a(a + b)p1 + b(a + b)p1 ap + bp


(a + b)(a + b)p1 ap + bp
(a + b)p ap + bp

Ahora, apoyandonos en esta u ltima desigualdad, podemos demostrar la de-


sigualdad triangular. Sean f , g y h en Lp , entonces

|f g|p = |f h + h g|p |f h|p + |h g|p


7.1 Nociones basicas 201

para 0 < p < 1, luego


Z 1
d(f, g) = |f g| dt
0
Z 1 Z 1
p
|f h| dt + |h g|p dt
0 0
= d(f, h) + d(h, g),

es decir, d(f, g) d(f, h) + d(h, g).

Ahora queremos exhibir que d es invariante por traslacion. En efecto,


Z 1
d(f + h, g + h) = |f + h (g + h)|p dt
Z0 1
= |f g| dt
0
= d(f, g).

Antes de continuar, recordemos la definicion de base local.

Definici on 7.3. Sea p un punto arbitrario de un espacio topologico X , una


clase Bp de conjuntos abiertos que contienen a p es una base local en p, si y
solo si para cada abierto O que contiene a p existe un abierto Op con centro
en p tal que Op O.

Continuando, sea V una vecindad abierta, convexa y no vaca del origen


de Lp [0, 1]. Sea f Lp [0, 1]. Dado que V es abierta, existe una bola Br
(r > 0) centrada en el origen tal que Br V . Sea n N tal que nf Bnr
donde
Bnr = {f Lp : kf kpp < nr}

luego,
Z 1
knf kpp = |nf (t)|p dt < nr
0

es decir,
Z 1
1
knf kpp = |f (t)|p dt < r
n1p 0
202 Espacios Lp () con 0 < p < 1

Ahora, para s [0, 1] definamos gs = [0,s] f y (s) = kgs kpp . Sea s, s0 [0, 1]
tal que s s0 , entonces

|(s) (s0 )| = |kgs kpp kgs0 kpp |


kgs gs0 kpp
= k([0,s] [0,s0 ] )f kpp
= k[0,s]\[0,s0 ] f kpp
= k[s0 ,s] f kpp
Z 1
= |[s0 ,s] (t)f (t)|p dt.
0
Z s
lm |(s) (s0 )| = lm |f (t)|p dt 0
ss0 ss0 s0

As, es continua en [0, 1].

Dado que (0) = 0 y (1) = kf kpp , por el teorema del valor intermedio
existe j (0, 1) j = 1, 2, . . . , n tal que
(1)
(0) (1),
n
kf kpp
0 kf kpp ,
n
1
kgj kpp = (j ) = kf kpp ,
n
Z j Z 1
1
|f (x)|p dx = |f (x)|p dx.
0 n 0
Definamos 
nf si 0 x j ;
hj (x) =
0 en otro caso.
Note que
Z 1 Z
p
|hj (x)| dx = |nf (x)|p dx
0 0
Z 1
p1
=n |f (x)|p dx
Z0 1
1
= 1p |f (x)|p dx
n 0
< r,
7.1 Nociones basicas 203

por lo tanto hj Br y as, hj V .



1X
Note que f = h , de la convexidad de V se concluye que f V , es
n j=1 j
decir Lp [0, 1] V , pero V Lp [0, 1] as, V = Lp [0, 1].

Para demostrar la parte d), sea f : Lp Y una aplicacion lineal y continua,


de donde Y es localmente convexo, entonces existe una base local B cuyos
elementos son convexos. Sea B esta base local convexa de Y . Si B (0) B, en-
tonces f 1 (B (0)) es abierta, convexa y no vaca ( arbitrariamente peque
no).
Por lo tanto,
f 1 (B (0)) = Lp [0, 1],
en consecuencia
f (Lp [0, 1]) B (0),
entonces
f (h) B (0)
para todo h Lp [0, 1], por lo tanto

|f (h)| < 

luego,
lm |f (h)| < lm  = 0,
0 0

as |f (h)| = 0, es decir f (h) = 0 para todo h Lp [0, 1]. En consecuencia el


nico funcional lineal continuo de Lp [0, 1] es f 0.
u

De la parte d) es claro que (Lp [0, 1]) = {0}, con lo cual se obtiene e).

Observaci on 7.4. Despues de analizada esta situacion, podemos preguntar-


nos: El espacio Lp ([0, 1], L, m) sera normable?

El siguiente resultado desafortunadamente nos dice que esto no es posible.

Proposici on 7.5. Sea {fn }nN una sucesion en Lp ([0, 1], L, m). Entonces
no existe una norma k k en Lp (m) tal que si fn 0 en Lp (m) implica que
kfn k 0 cuando n , para cualquier sucesion {fn }nN Lp (m).

Demostracion. Supongamos que tal norma k k existe. Afirmamos que existe


una constante positiva C < tal que kf k Ckf kp f Lp (m). Dado
que la aplicacion f 7 kf k es continua, podemos hallar un > 0 tal que
si kf kp < , entonces kf k 1, luego, para todo f Lp (m) se tiene que
204 Espacios Lp () con 0 < p < 1

f
kf kp
B(0, ) con 0 < || < 1, donde B(0, ) es la bola de centro 0 y radio
. Por lo tanto
f
kf kp 1

implica que
1
kf k kf kp (7.4)

Si 1, entonces (7.4) se cumple para C = 1 . Escojamos

C = nf {K : kf k Kkf kp f Lp (m)}

(Notese que no excluimos la posibilidad de que C = 0). Por el Teorema del


valor intermedio existe c (0, 1) tal que
Z c Z 1 Z 1
p p 1
|f | dm = |f | dm = |f |p dm.
0 c 2 0

Ahora, para g = f [0,c) y h = f (c,1] , tenemos que f = g +h y kgkp = khkp =


1
2 p kf kp , por la desigualdad triangular

kf k kgk + khk
C(kgkp + khkp )
C
= 1/p1 kf kp .
2
Como p (0, 1), entonces

C
C,
21/p1
de donde C = 0, as kf k = 0 f Lp (m) lo que contradice el hecho que
k.k es una norma.

Observacion 7.6. Si Lp (m) fuera normable, entonces el Teorema de Hahn-


Banach es cierto en Lp (m), pero (Lp (m)) = {0}. Contradiccion.

Teorema 7.7. Sean (X, A, ) un espacio medible y p un n umero real tal que
0 < p < 1. Sean f y g funciones de Lp (X), tal que g 6= 0, entonces
1
kf + gkp 2 p 1 (kf kp + kgkp ). (7.5)
7.1 Nociones basicas 205

Demostracion. Sea 0 t < y 0 < p < 1, entonces p 1 < 0 y es claro


que 1 + t t y 1 + t 1, por lo cual

(1 + t)p1 tp1
y
(1 + t)p1 1. (7.6)
De (7.6) obtenemos
t(1 + t)p1 tp , (7.7)
sumando (7.6) y (7.7) se tiene que

(1 + tp ) 1 + tp . (7.8)
|f |
Definamos t = |g|
, sustituyendo en (7.8) resulta

(|f | + |g|)p |f |p + |g|p ,

pero |f + g| |f | + |g| y 0 < p < 1, entonces

|f + g|p |f |p + |g|p , . (7.9)


1
Ahora, dado que la funcion f (t) = t p es convexa en [0, ], entonces en virtud
de (7.9) se tiene

kf + gkpp
R R R
|f + g|p d |f |p d + |g|p d
= ,
2 2 2
entonces
R  p1 R R  p1
kf + gkp |f + g|p d |f |p d + |g|p d
1 =
2p 2 2
1
2 p 1 [kf kp + kgkp ]

y esto demuestra (7.5).

Observaci
on 7.8. Este u
ltimo resultado nos indica que
Z 1/p
p
kf kp = |f | d
X

no es norma en Lp (X) con 0 < p < 1 el resultado que sigue nos garantiza
con mas precision esta u
ltima afirmacion.
206 Espacios Lp () con 0 < p < 1

umeros reales con 0 < p < 1 y < q < 0


Teorema 7.9. Sean p y q dos n
tales que
1 1
+ = 1,
p q
sean f y g funciones positivas tales que f p y g q son integrables y, ademas,
supongamos que f g es integrable, entonces
Z
kf kp kgkq f g d.
X

Demostracion. Definamos r = 1/p y s = q/p, verifiquemos que r y s son


conjugados  
1 1 p 1
+ =p =p 1 = 1,
r s q q
ademas, observe que s > 1, ahora escribamos

f p = f p g p g p = (f g)p g p ,

a demostrar que (f g)p Lr (X) y g p Ls (X), en efecto


Z Z Z
p r p 1/p
[(f g) ] d = [(f g) ] d = f g d < ,
X X X

lo que prueba que (f g)p Lr (X). Por otra parte,


Z Z Z
p s p q/p
(g ) d = (g ) d = g q d < ,
X X X

as g p Ls (X), luego en virtud de la desigualdad de Holder se tiene que


Z Z 1/r Z 1/s
p p r p s
f d [(f g) ] d (g ) d
X X X
Z p Z p/q
p 1/p p q/p
= [(f g) ] d (g ) d
X X
Z p Z p/q
q
= f g d g d ,
X X

de donde
Z 1/p Z  Z 1/q
p q
|f | d f g d |g| d ,
X X X
7.1 Nociones basicas 207

de aqu
Z 1/p Z 1/q Z 
p q
|f | d |g| d f g d ,
X X X

por lo tanto Z
kf kp kgkq f g d.
X

Corolario 7.10. Sea 0 < p < 1 y sean f y g funciones positivas de Lp (X),


entonces f + g Lp (X) y

kf + gkp kf kp + kgkp .

Demostracion. En virtud del teorema 7.7 f + g Lp (X), por otra parte


escribamos
(f + g)p = f (f + g)p1 + g(f + g)p1 , (7.10)
consideremos ahora
Z 1/q

(f + g)p1 = |f + g| q(p1)
d
q
X
Z 1/q
p
= |f + g| d
X
= kf + gkp/q
p
h 1 ip/q
2 p 1 (kf kp + kgkp ) ,

lo que prueba que (f + g)p1 Lq (X). Retomando (7.10) y empleando el


Teorema 7.9, resulta
Z "Z 1/p Z 1/p # Z 1/q
p p p q(p1)
(f + g) d f d + g d (f + g) d
X X X X
"Z 1/p Z 1/p # Z 1/q
p p p
= f d + g d (f + g) d ,
X X X

de aqu
Z 1 1q Z 1/p Z 1/p
p p p
(f + g) d f d + g d ,
X x X
208 Espacios Lp () con 0 < p < 1

as Z 1/p Z 1/p Z 1/p


p p p
(f + g) d f d + g d ,
X x X

finalmente,
kf + gkp kf kp + kgkp .

Observaci on 7.11. El resultado anterior nos indica que la desigualdad de


Minkowski queda sin efecto en Lp (X) cuando 0 < p < 1, por tal motivo, la
funcion k kp no define una norma en Lp (X) con 0 < p < 1.

Ejercicios

1. Para 0 < p < 1, demostrar que



Xn n
1p X
fj N kfj kp ,
p


j=1 p j=1

1p
ademas demuestre que N p es la mejor constante.
Captulo 8

Lp es un espacio uniformemente
convexo

8.1. Convexidad Uniforme


Definicion 8.1. Un espacio de Banach (R, V, +, , k k) se dice que es uni-
formemente convexo si para todo  > 0 existe un numero > 0 tal que para
todo x, y V las condiciones
kxk = kyk = 1, kx yk 
implican
x + y
2 1 .

El n
umero
 
x + y
() = nf 1
: kxk = kyk = 1, kx yk 
2
se le llamara el modulo de convexidad. Notese que si 1 < 2 , entonces (1 ) <
(2 ) y (0) = 0 pues x = y si  = 0.
El ejemplo mas inmediato de un espacio uniformemente convexo es un
espacio de Hilbert ya que su norma satisface la ley del paralelogramo, es
decir,
x + y 2 1 x y 2

2 2
2 = 2 (kxk + kyk ) 2 ,

en tal sentido si kxk = kyk = 1 y kx yk , entonces


x + y 2 2

1  ,
2 4

209
210 Lp es un espacio uniformemente convexo

de aqu obtenemos r
2 x + y
1 1 1
,
4 2
de donde r
2
() 1 1 .
4
Lema 8.2. Si p 2 entonces
1/2
(|a + b|p + |a b|p )1/p |a + b|2 + |a b|2
para todo a, b R.

Demostracion. Consideremos la funcion


f (t) = (1 + tp )1/p (1 + t2 )1/2
para todo t [1, 1]; es claro que f es derivable en [1, 1], calculemos la
primera derivada de f
1 1 1
f 0 (t) = (1 + tp ) p 1 ptp1 (1 + t2 )1/2 (1 + t2 )3/2 2t(1 + tp )1/p
p 2
1
1
= (1 + tp ) p (1 + t2 )3/2 t tp2 (1 + t2 ) (1 + tp )
 
1
= (1 + tp ) p 1 (1 + t2 )3/2 t(tp2 1),
de aqu se tiene que f 0 (1) = 0, f 0 (0) = 0, tambien observamos que f 0 (t) < 0
si t (0, 1) y f 0 (t) > 0 si t (1, 0), lo que significa que la funcion f alcanza
un maximo en t = 0. Sea t 6= 0, entonces
f (t) f (0),
es decir, f (t) 1 pues f (0) = 1, en consecuencia
(1 + tp )1/p (1 + t2 )1/2 1,
as
(1 + tp )1/p (1 + t2 )1/2 ,
escojamos
|a b|
t= ,
|a + b|
entonces 1/2
(|a + b|p + |a b|p )1/p |a + b|2 + |a b|2 .
8.1 Convexidad Uniforme 211

Lema 8.3. Si p 2 para todo a, b R, entonces

|a + b|p + |a b|p 2p1 (|a|p + |b|p ).

Demostracion. En virtud del Lema 8.2 y la ley del paralelogramo podemos


escribir

(|a + b|p + |a b|p )1/p (|a + b|2 + |a b|2 )1/2


= [2(|a|2 + |b|2 )]1/2

2(|a|2 + |b|2 )1/2 . (8.1)

Por la desigualdad de Holder para


2 p2
+ = 1,
p p
tenemos
p2
|a|2 + |b|2 (|a|p + |b|p )2/p (1 + 1) p

p2
2 p (|a|p + |b|p )2/p ,

de aqu obtenemos
p2
2(|a|2 + |b|2 )1/2 22 2p (|a|p + |b|p )1/p
p1
=2 p (|a|p + |b|p )1/p ,

por (8.1) tenemos


p1
(|a + b|p + |a b|p )1/p 2 p (|a|p + |b|p )1/p ,
por lo tanto
|a + b|p + |a b|p 2p1 (|a|p + |b|p ).

Lema 8.4. Sean p, q R tales que 1 < p < y q 2, entonces para


a, b R se cumple que
1/q 1/p
|a + b|q |a b|q |a|p + |b|p
 
+ .
2 2 2
212 Lp es un espacio uniformemente convexo

Demostracion. Consideremos la funcion


q/p  q  q
1 + |t|p

1+t 1t
f (t) =
2 2 2

con t [0, 1], entonces


 q
p p 1
 q1  q1
0 q p1 t 1 + |t| q 1+t q 1t
f (t) = p|t| +
p 2|t| 2 2 2 2 2
" q #
 p p
 1   q1   q1
q 1 + |t| 1 + t 1 t
= t|t|p2 + .
2 2 2 2

Notese que
f 0 (0) = f 0 (1) = f (0) = f (1) = 0
y que f 0 (t) 0 para todo t [0, 1], lo que significa que f es creciente en
[0, 1], luego si 0 t, entonces f (0) f (t), de donde
q/p q q
1 + |t|p
  
1+t 1t
0 ,
2 2 2

de aqu
q q q/p
1 + |t|p
  
1+t 1t
+ ,
2 2 2
por lo cual
q q 1/q 1/p
1 + |t|p
  
1+t 1t
+ , (8.2)
2 2 2

hacemos t = b/a, entonces 0 b/a 1, reemplazando t en (8.2) resulta


1/q 1/p
|a + b|q |a b|q |a|p + |b|p
 
+ .
2 2 2
8.1 Convexidad Uniforme 213

Teorema 8.5. Si 1 < p < . El espacio Lp es uniformemente convexo.

Demostracion. Distingamos dos casos:

Caso 1 p 2. En virtud del Lema 8.3 tenemos que

|a + b|p + |a b|p 2p1 (|a|p + |b|p ).

para a, b R. Ahora bien, sean f y g funciones de Lp , entonces

|f + g|p + |f g|p 2p1 (|f |p + |g|p ),

integrando tenemos
Z Z Z Z 
p p p1 p p
|f + g| d + |f g| d 2 |f | d + |g| d ,

es decir,
kf + gkpp + kf gkpp 2p1 (kf kpp + kgkpp ).
Ahora bien, si kf kp = kgkp = 1 y kf gkp , entonces

kf + gkpp 2p1 (kf kpp + kgkpp ) kf gkpp

implica que

kf + gkpp 2p1 2 p
= 2p p ,

por lo tanto
f + g p

1  ,
 

2 2
p

de donde
f + g
1 
  1/p   p

2 1 ,
p 2 2
de esto u
ltimo obtenemos que
  p
f + g
1
,
2 2 p

as   p
Lp () .
2
214 Lp es un espacio uniformemente convexo

Caso 2 1 < p < 2. Del Lema 8.3 tenemos


1/q  p 1/p
|a + b|q |a b|q |a| + |b|p

+
2 2 2
para a, b R, q 2 y 1 < p < . Escojamos q = 2 y sean f, g Lp
con 1 < p < , entonces
1/2  p 1/p
|f + g|2 |f g|2 |f | + |g|p

+ ,
2 2 2
de aqu " 1/2 #p
|f + g|2 |f g|2 |f |p + |g|p
+
2 2 2
e integrando
Z " 1/2 #p
|f + g|2 |f g|2
Z  p
|f | + |g|p

+ d d,
2 2 2

as
|f + g|2 |f g|2 1/2 p kf kp + kgkp

p p
+ ,

2 2 2


p

de donde 
|f + g|2 |f g|2 1/2
+ 1.

2 2


p

En virtud de la observacion 7.8 tenemos


 1/2
f + g 2 f g 2

f + g f g
+ + 1,

2 2
2 2
p p

p

en consecuencia
f + g 2
2
1 f g


2 2
p p
  2
1 ,
2
por lo tanto
f + g 2
r
1  1  .
 2  2

2 2 2
p
8.1 Convexidad Uniforme 215

Observaci on 8.6. La convexidad uniforme es una propiedad geometrica de


la norma, una norma equivalente no necesariamente es uniformemente con-
vexa. Por otra parte, la reflexividad es una propiedad topologica, es decir, un
espacio reflexivo sigue siendo reflexivo para sus normas equivalentes. El Teo-
rema de Milman-Pettis (ver Brezis) nos provee de una herramienta inusual,
nos dice que una propiedad geometrica implica una propiedad topologica. En
otras palabras, todo espacio de Banach uniformemente convexo es reflexivo.
Antes de hablar del recproco de este resultado analicemos el siguiente ejem-
plo. Consideremos en R2 las normas Eucldea k(x, y)k2 = (|x|2 + |y|2 )1/2 y la
norma l1 k(x, y)k1 = |x| + |y|, no es difcil demostrar que
1
k(x, y)k1 k(x, y)k2 2k(x, y)k1
2
es decir, las normas k(x, y)k1 y k(x, y)k2 son equivalentes, ademas R2 con la
norma Eucldea es uniformemente convexo mientras que con la norma l1 no
lo es, sin embargo R2 es reflexivo con respecto a ambas normas. El siguiente
resultado nos dice que el recproco del Teorema de Milman-Petti no es cierto.

Teorema 8.7 (M.M. Day). Existen espacios de Banach los cuales son sepa-
rables, reflexivos y estrictamente convexos, pero no son isomorfos a ning
un
espacio uniformemente convexo.

M.M. Day, Reflexive Banach space not isomorphic to uniformly convex


spaces, Bull. Amer. Math. Soc. volume 47, n4 (1941), 313-317.

Ejercicios

1. Consideremos en R2 las normas k(x, y)k2 = (|x|2 +|y|2 )1/2 y k(x, y)k1 =
|x| + |y|. Demuestre que estas normas son equivalentes, es decir
1
k(x, y)k1 k(x, y)k2 2k(x, y)k1 .
2
216 Lp es un espacio uniformemente convexo
Captulo 9

Isometra en Lp

Definici on 9.1. El operador lineal T : Lp () Lp () se dice que es una


isometra si y solo si

kT (f )kp = kf kp (f Lp ()).

umeros reales. Entonces si 2 p <


Lema 9.2. Sean y dos n

| + |p + | |p 2(||p + ||p ),

y
| + |p + | |p 2(||p + ||p ).
para p < 2. Si p 6= 2, la igualdad ocurre si o es cero.
Demostracion. Si p = 2, tenemos la igualdad para todo y . Si 2 < p < ,
entonces 1 p/2, luego con los exponentes conjugados p/2 y p/(p 2)
aplicamos la desigualdad de Holder a 2 + 2 (ver Lema 2.33) as
2 p2
2 + 2 (p + r ) p (1 + 1) p

de donde se obtiene que


2p p
p + p 2 2 (2 + 2 ) 2 . (9.1)

Si 0 < p < 2, reemplazamos p por 4/p en (9.1), se tiene que


4 4 p2 2
p + p 2 p (2 + 2 ) p .
p p
Si reemplazamos por 2 y por 2 esta u
ltima desigualdad se transforma
en p2 2
2 + 2 2 p (p + p ) p

217
218 Isometra en Lp

o
2p p
p + p 2 2 (2 + 2 ) 2 . (9.2)
Dado que
2
0 1
2 + 2
y

2
0 1
2 + 2
para 2 < p resulta
||2(p2)
0 2 1
( + 2 )p2
y

||2(p2)
0 1
( 2 + 2 )p2
lo que es equivalente a

||p2
0 p2 1
( 2 + 2 ) 2

y
||p2
0 p2 1.
( 2 + 2 ) 2

As,
||p ||1
0 p
( 2 + 2 ) 2 2 + 2
y
||p ||1
0 2 p .
( + 2 ) 2 2 + 2
Sumando estas dos u
ltimas desigualdades obtenemos
p
||p + ||p ( 2 + 2 ) 2 si 2 < p. (9.3)

De manera similar se obtiene


p
||p + ||p ( 2 + 2 ) 2 si p < 2. (9.4)
219

Note que la igualdad se da en (9.3) y (9.4) si y solo si = 0 o = 0.


Finalmente si p > 2, reemplazamos y en (9.1) por | + | y .
Entonces
2p p
| + |p + | |p 2 p (| + |2 + | |2 ) 2
2p p
=2 p (2[||2 + ||2 ]) 2
p
= 2( 2 + 2 ) 2
2(||p + ||p ).
As, obtenemos la primera desigualdad. La segunda desigualdad se obtiene
de manera similar usando (9.2) y (9.4). Observamos que en cualquier caso la
igualdad ocurre si = 0 o = 0.
Como una consecuencia del lema anterior la siguiente version integral.
Lema 9.3. Sea 1 p < , p 6= y supongamos que f, g Lp (). Entonces

kf + gkpp + kf gkpp = 2(kf kpp + kgkpp )

si y solo si f g = 0 c.t.p. [].


Demostracion. Si f.g = 0 c.t.p. [], entonces (supp f supp g) = 0. Enton-
ces
Z Z Z
p p p
kf + gkp = |f + g| d + |f + g| d + |f + g|p d
supp(g) supp g X\(supp f supp g)

= kf kpp + kgkpp
(9.5)
Similarmente obtenemos

kf gkpp = kf kpp + kgkpp (9.6)

Sumando (9.5) y (9.6) se tiene

kf + gkpp + kf gkpp = 2(kf kpp + kgkpp )


) Ahora, si kf + gkpp + kf gkpp = 2(kf kpp + kgkpp ), entonces
Z Z Z Z 
p p p p
|f + g| d + |f g| d 2 |f | d + |g| d = 0
X X X X

luego Z
[|f + g|p + |f g|p 2(|f |p + |g|p )] d = 0.
X
220 Isometra en Lp

Ahora bien en virtud del Lema 9.2 tenemos

|f + g|p + |f g|p 2(|f |p + |g|p ) 0

o
|f + g|p + |f g|p 2(|f |p + |g|p ) 0.
En ambos casos, por un resultado conocido de la teora de integracion de
Lebesgue resulta

|f + g|p + |f g|p 2(|f |p + |g|p ) = 0


c.t.p.[]. Esto claramente implica que para casi todo x X, f (x) = 0 cuando
g(x) 6= 0 y g(x) = 0 cuando f (x) 6= 0, o alternativamente (supp(g)
supp(g)) = 0.

Teorema 9.4 (Lamberti). Sea el operador lineal T : Lp (m) Lp (m) una


isometra en Lp (m) donde 1 p < , p 6= 2. Entonces existe una funcion
Lebesgue medible : R R y una u nica funcion Lebesgue medible h definida
c.t.p. [m] tal que
T (f )(x) = h(x)f ((x)).
La funcion esta definida c.t.p.[m] en supp(h) y para cada conjunto E
Lebesgue medible con m(E) < , se tiene
Z
m(E) = |h(t)|p dt.
1 (E)

Demostracion. Para cada conjunto A R Lebesgue medible tal que m(A) <
, definimos (A) por

(A) = supp(T (A )).

Si A B = , entonces por el Lema 9.3

kA + B kpp + kA B kpp = 2(kA kpp + kB kpp ).

Del hecho que T es una isometra en Lp (m) y en virtud del Lema 9.3 se sigue
que
T (A ) T (B ) = 0, c.t.p.[] en R.
Ademas A S B = A + B y T (A S B ) = T (A ) + T (B ), luego
[ [
(A B) = (A) (B), si A B = .
221

Dado que para todo conjunto A y B se tiene que


[
(A) = (A\(A B)) (A B),

y [ [
(A B) = (A\(A B)) (B)
es claro que para conjuntos A y B Lebesgue medibles tal que m(A) < y
m(B) < [ [
(A B) (A B) = (A) (B).
Por otra parte, sea {Xj }jN una sucesion creciente
S de subconjuntos de R
S que m(Xj ) < para j = 1, 2, . . . y R = j=1 Xj , ademas sea E =
tal
j=1 (Xj ). Entonces enva subconjuntos medibles en R en subconjuntos
medibles de E y (R\A) = E\(A).
Tambien, si {Aj }jN es una sucesion de subconjuntos
S de R Lebesgue
medibles con m(Aj ) < , j = 1, 2, . . . y si A = j=1 Aj entonces A =
lmk kj=1 Aj , de la continuidad y acotamiento de T , se tiene que
P

k
X
T (A ) = lm T (Aj ).
k
j=1

Sin embargo, la sucesion {supp T (Aj )}jN es una clase disjunta, as



!
[ [
= (Aj ).
j=1 j=1

Esto nos muestra que es un -homomorfismo, ahora, invocando el Teo-


rema B.2 podemos hallar una funcion : E R Lebesgue medible tal que

(A) = 1 (A)

para cualquier conjunto de Borel A. Luego, dado que el conjunto (A) es


u
nico c.t.p.[], entonces la funcion es u
nica c.t.p.[m]. Dado un conjunto C
Lebesgue medible con m(C) < , definamos

hC = T (X ),

as 1
kT (c )kp = khC kp = (m(C)) p ,
luego, para cada conjunto medible A, tenemos

C = CA + CAc ,
222 Isometra en Lp

de aqu, resulta
hC = T (CA ) + T (CAc ).
Sin embargo, arriba hemos demostrado que

(supp(T (AC )) (supp T (CAc ))) = .

As,
T (CA ) = hC (CA)
= hC CA (()).
Para la sucesion {Xj }iN considerada una linea arriba, definamos una funcion
h Lebesgue medible dada por

h = lm hXj .
j

Entonces, dado cualquier subconjunto A de R medible con m(A) < , se


sigue de la acotacion y continuidad de T que

T (A ) = lm T (Xj A )
j

= hA (()).
As, para cada funcion f simple y medible se tiene que

T (f ) = hf (()).

Sea f Lp (m), entonces en virtud del Lema 2.47 existe una sucesion {sn }nN
de funciones simples tal que

kf sn kp 0 cuando n .

Ahora

kT (f ) hSn (())kp = kT (f Sn )kp = kf Sn kp


Entonces por el Lema 2.28 con g() = p resulta

m({x R | |T (f )(x) hSn ((x))| > }) p kT (f ) hSn (())kpp


= p kf Sn kpp .

As, que m({x R | |T (f )(x) hSn ((x))| > }) 0 cuando n


ltimo se deduce que existe una subsucesion {Snk }jN tal
( > 0), de esto u
que para casi todo x R
223

T (f )(x) = lm h(x)Snk ((x))


j

= h(x)f ((x)).

Finalmente, para cada conjunto A Lebesgue medible con m(a) < , tenemos

Z Z
p
m(A) = (A (x)) dm = |T (A )(x)|p dm
ZR R

= |h(x)|p A ((x))dm
ZR
= |h(x)|p dm.
1 (A)

nica c.t.p.[m], en efecto sea h0 otra funcion tal que T (f ) =


La funcion es u
h f (()) dado que f es arbitraria tenemos h = h0 c.t.p.[m].
0
224 Isometra en Lp
Ap
endice A

El teorema del residuo

Teorema A.1 (Teorema del residuo). Sea f una funcion analtica dentro
y sobre una curva simple cerrada excepto en los puntos z1 , . . . , zn que se
encuentra en el interior de esta. Entonces
I Xn
f (z)dz = 2i Res(f, z = zk ).
k=1

Lema A.2. Sea p > 1, entonces


1

x p
Z

dx = .
0 1+x sin(/p)

Demostracion. Consideremos la integral de variable compleja dada por


1
z p
I
dz
C 1+z
donde C es la region compuesta de la siguiente manera

y
D

r A B
H G
E J x

225
226 El teorema del residuo

Los segmentos AB y GH son paralelos entre si y con el eje real positivo.


Sea
1
z p
f (z) = .
1+z
No es difcil deducir que f posee un polo simple en z = 1 dentro de la
region C.
Si z = 1, entonces

z = cos() + i sin() = ei .

Luego

Res(f, z = 1) = lm (z + 1)f (z)


z1
1
z p
= lm (z + 1)
z1 1+z
p1
= lm z
z1
i
=e p .

En virtud del teorema del residuo se tiene que


1
z p
I
i
dz = 2ie p .
C 1+z

Por otra parte, de acuerdo a las integrales de caminos se tiene que


Z Z Z Z
i
e p = f (z)dz + f (z)dz + f (z)dz + f (z)dz
AB BDEF G GH HJA
1 1
x p (Rei ) p
Z R Z 2
= dx + iRei d
r 1+x 0 1 + Rei
1 1
(xe2i ) p (Rei ) p
Z r Z 0
+ 2i
dx + i
iRei d.
R 1 + xe 2 1 + Re

Esto ultimo se obtuvo al hacer el cambio z = xe2i en la tercera integral,


ademas se debe tener presente que el argumento de z aumenta 2 al dar una
vuelta alrededor del crculo BDEF G.
Ahora bien, si r 0 y R podemos observar que la segunda y
tercera integral del resultado anterior se anulan.
227

En efecto,

1 1
!
2
(Rei ) p 2
(Rei ) p
Z Z
lm iRei d = lm iRei d
R 0 1 + Rei 0 R 1 + Rei

2
iRei
Z
= lm 1 d
0 R
(Rei ) p (1 + Rei )
Z 2
i
= lm 1 d
0 R (Rei ) p
Z 2
= 0d
0
=0.

Para calcular el otro lmite se procedera de manera similar, previo a ello,


1
hacemos el siguiente cambio de variable u = r p donde u 0. As

1
! !
0
(rei ) p i 0
iei
Z Z
r
lm ire d = i lm 1 d
r0 2 1 + rei 2 e p r0 r p (1 + rei )
0
iei up
Z  
= i lm i
d
2 e p u0 u(1 + ue )
0
iei
Z
= i (0)d
2 ep
=0.

Finalmente, resulta

1

x p 0
(xe2i )f rac1p
Z Z
i
2ie p = dx + dx
0 1+x 1 + xe2i
p1 1 2i

x p e p
Z Z
x
= dx dx
0 1+x 0 1 + xe2i
p1 1

x p
Z Z
x 2i
= dx e p dx
0 1+x 0 1 + x(cos(2) + i sin(2))
Z p1
2i x
=(1 e p ) dx.
0 1+x
228 El teorema del residuo

Por lo tanto
1 i

x p 2ie p
Z
dx = 2i
0 1+x 1 e p 
2i 1
= i 1 2
ep ep

=  i i

p p
e e
2i


=  .

sin p
Ap
endice B

-homomorfismo

on B.1. Sea (X, A) un espacio medible. Si f es una funcion real


Definici
A-medible en X. Entonces la funcion de conjuntos definida por
(A) = f 1 (A) = {x X : f (x) A}
es una funcion definida de la -algebra L de R en la -algebra A tal que
[ [
(A B) = (A) (B) si A B = (B.1)
y
(A\B) = (A)\(B) A, B L. (B.2)
Una funcion que satisface las condiciones (B.1) y (B.2) se dice que es un
homomorfismo de L en A. El homomorfismo se dice que es un -homomorfismo
si (R) = X y para toda sucesion {Aj }jN de conjuntos disjuntos en L se
cumple que

!
[ [
= (Aj ).
j=1 j=1

Teorema B.2 (Sikorki). Sea (X, A) un espacio de medida y un -homomorfismo


definido de la -algebra de Borel en A. Entonces existe una u
nica funcion f
1
A-medible tal que (A) = f (A) para cualquier conjunto de Borel A.
Demostracion. Para cada n
umero real r, sea Ar = ([, r]), note que
A = X y Ar1 Ar2 si r1 r2 .i
Ahora bien, para cada x X definamos
f (x) = nf{r R | x Ar }.
Note que f : X R y para cada t R
[ [
{x X | f (x) t} = Ar = {AS | s t, s Q}
rt

229
230 -homomorfismo

donde Q representa el conjunto de los n umeros racionales. As es claro que f


es A-medible, dado que todo conjunto de Borel puede expresarse como union
e interseccion contable de subintervalos de R abiertos y cerrados, entonces

Ar = f 1 ([, r]) .

Finalmente, en virtud que es un -homomorfismo es facil ver que (A) =


f 1 (A).
Ap
endice C

Alfabeto griego

1. -alpha

2. -beta

3. , -gamma

4. , -delta

5. -epsilon

6. -zeta

7. -eta

8. , -theta

9. -iota

10. -kappa

11. , -lambda

12. -mu

13. -nu

14. , -xi

15. o-omicron

16. , -pi

17. -rho

231
232 Alfabeto griego

18. , -sigma

19. -tau

20. , -upsilon

21. , -phi

22. -chi

23. , -psi

24. , -omega
Bibliografa

[1] Y.A. Abramovich and C.D. Aliprantis. An Invitation to Operator


Theory, volume 50 of Graduate Studies in Mathematics. American Mat-
hematical Society, USA, 2002.

[2] Charalambos D. Aliprantis and Burkinshaw. Principles of Real Analysis.


Academic Press, New York, third edition, 1998.

[3] Haim Brezis. Functional Analysis, Sobolev Spaces and Partial Differen-
tial Equations. Springer, New York, 2010.

[4] Emmanuelle DiBenedetto. Real Analysis. Birkhauser Advanced Texts,


Boston, 2002.

[5] Gerald Folland. Real Analysis, Modern Tecniques and Their Applica-
tions. John Wiley and Sons, Inc., New York, second edition, 1999.

[6] Frank Jones. Lebesgue integration on Euclidean Space. Jones and


Bartlett Publishers, Sudbury, Massachusetts, revised edition, 2001.

[7] Serge Lang. Real Analysis. Addison-Wesley Publising Company,


Reading, Massachusetts, 1983.

[8] John McDonald and Neil Weiss. A Course in Real Analysis. Academic
Press, New York, 1999.

[9] Ole A. Nielsen. An Introduction to Integration and Measure Theory.


Canadian Mathematical Society Series of Monographs and Advanced
text, A Wiley-Interscience publication, Jhon Wiley and Sons, Inc., New
York, 1996.

[10] George Okikiolu. Aspect of the theory of Bounded Integral Operators in


Lp -spaces. Academic Press, New York, 1971.

[11] H.L. Royden. Real Analysis. Prentice Hall, New Jersey, third edition,
1988.

233
234 BIBLIOGRAFIA

[12] Walter Rudin. Real and complex analysis, volume 693. Mc Graw-Hill
Book Company, New York, second edition, 1987.

[13] Elias M. Stein. Singular integrals and differentiability properties of fun-


ctions, volume 30 of Princeton Mathematical Series. Princeton Univer-
sity Press, USA, 1970.

[14] Alberto Torchinski. Real Variables. Addison-Wesley Publishing Com-


pany, Inc., USA, 1988.
Indice alfab
etico

Aplicacion lineal conjugada, 121 Espacio


completo, 119
Cantor
de Banach, 36, 42, 102
diagonalizacion de, 94
de Hilbert, 118121
Complemento ortogonal, 119
de las funciones localmente inte-
Conjunto
grables, 137
de Borel, 100
dual, 86, 115, 155
Convergencia
dual del espacio de Schwartz, 180
debil, 89
metrico
Convexidad Uniforme, 209
-compacto, 107
Convolucion, 157, 162
normado, 116
propiedades, 162
reflexivo, 117, 121
Desigualdad separable, 43, 44
de Cauchy-Schwarz, 120, 125 -finito, 124
de Hardy, 113 uniformemente convexo, 209, 213
en lp , 45 modulo de convexidad, 209
de Hedberg, 193, 194 vectorial, 118
de Hilbert, 112 Espacios
en lp , 47, 49 isometricamente isomorfos, 115, 117
de Holder, 30, 38, 108110, 112,
163, 191, 193 Fatou
generalizada, 32 lema de, 72, 188, 190
de Jensen, 10, 12 Funcion
de Kolmogorov, 148 Borel medible, 101
de Markov, 35, 147 convexa, 5, 6, 9
de Minkowski, 28, 39, 41, 109 delta, 184
de Paley-Zygmund, 126 escalonada, 63
de Rogers, 54 esencialmente acotada, 52
de Young, 16 gamma, 131
integral de Minkowski, 109, 114 localmente integrable, 137
Dimension maximal de Hardy-Littlewood, 138,
finita, 102 146, 156
Distribuciones, 180 medible, 111

235
236 INDICE ALFABETICO

semicontinua inferiormente, 139 Norma


simple, 57, 58, 96 del supremo, 102
Funcional en lp , 41
acotado, 67, 114
continuo, 120 Operador
lineal, 67, 120122 acotado, 127
acotado cerca del origen, 189
Identidad de aproximacion, 184 adjunto del operador de Hardy,
Inmersion natural, 117 114
Integral compacto, 107, 109
de Gauss-Weierstrass, 195 de Hardy, 113, 128, 164, 165
de Lebesgue-Stieltjes, 99 de Volterra, 133
de Poisson, 195 dilatacion, 186
Isometra, 121 espectro de un, 127
en Lp , 217 identidad, 127
Isomorfismo, 115 integral, 107, 108
isometrico, 117 maximal, 137
no compacto, 128
l , 50 no invertible, 127
Laplaciano, 179 semicontinuo, 190
fraccional, 180 sublineal, 147, 150, 151
Ley del paralelogramo, 118, 119, 126, debil (p, q), 147
209, 211 de tipo fuerte (p, q), 147
Localmente uniformemente integrable,
Potencial
181
de Riesz, 179, 181, 186
Lp , 21
norma del, 192
aproximaciones en, 57
Potenciales, 179
debil, 141
en Lp , 186
es separable, 64
Producto interior, 118120
L2 , 118
norma en, 21, 27 Semigrupo de Gauss-Weierstrass, 164
lp , 38 -homomorfismo, 229
Lp con 0 < p < 1, 197 Subconjunto
denso, 44
M.M. Day, 215 Subespacio
Medida cerrado, 120
de Borel, 99 propio, 120
de Lebesgue, 104, 191 Supremo esencial, 22
de Radon, 191
con signo, 190 Teorema
-finita, 124 de Arzela-Ascoli, 107, 109
INDICE ALFABETICO
237

de Bolzano-Weierstrass, 93
de Cotlar, 154
de cubrimiento de Vitali, 138
de descomposicion de Hahn, 121
de diferenciacion de Lebesgue, 142
de Fubini, 108110, 112, 114, 180
de Hahn-Banach, 115
de la convergencia dominada, 86
de Lamberti, 220
de Radon-Nikodym, 121, 124
de representacion de Riesz, 74, 79
de Riesz, 120122
de Sikorki, 229
de Tonelli, 159, 163
de W.H. Young, 166
del residuo, 225
calculo, 129
Interpolacion de Marcinkiewick, 151
Transformada
de Fourier, 179
inversa de Fourier, 180
Traslacion
continuidad de, 103

Urysohn
lema de, 59

Valor propio, 127


Von Neumann, 121

Young, 18, 19

Você também pode gostar